Sei sulla pagina 1di 244

Introducci on

Este libro est a dirigido a estudiantes de las Olimpiadas de Matem aticas que quieren prepararse en el estudio de desigualdades, tema ahora frecuente en los concursos de matem aticas de distintos niveles. En este volumen se presentan las desigualdades cl asicas y desde luego las m as u tiles para enfrentar y resolver problemas de optimizaci on. Para la presentaci on de los temas, el libro se ha dividido en cuatro cap tulos. El cap tulo 1 de desigualdades num ericas contiene las desigualdades b asicas. La mayor a de ellas son desigualdades num ericas que en general carecen de una interpretaci on geom etrica, sin embargo cuando es posible darla esta se incluye. Se hace notar la importancia de algunas de ellas, por ejemplo, la desigualdad entre la media geom etrica y la media aritm etica, la desigualdad de Cauchy-Schwarz, la desigualdad del reacomodo, la desigualdad de Jensen, la desigualdad de Muirhead, entre otras. Para todas ellas, adem as de su justicaci on, se presentan varios ejemplos que muestran c omo utilizarlas en problemas del tipo de olimpiadas de matem aticas. Tambi en se hace notar c omo la estrategia de sustituci on se utiliza para deducir varias desigualdades. En el cap tulo 2 de desigualdades geom etricas, se ejemplica el uso de las desigualdades num ericas b asicas del cap tulo 1 para resolver problemas geom etricos. Se trabaja tambi en, en esta parte, con desigualdades que tienen un fuerte contenido geom etrico, iniciando con hechos b asicos como la desigualdad del tri angulo y la desigualdad de Euler. Introducimos ejemplos donde el car acter sim etrico en las variables ayuda a resolver algunos problemas, se hace tambi en enfasis en c omo los cambios de variable se utilizan para deducir varias de ellas. Resaltamos entre estos la transformaci on de Ravi y la correspondencia entre una desiguladad en t erminos de las longitudes de los lados de un tri angulo a, b, c y las desigualdades correspondientes en t erminos de s, r y R, el semiper metro, el

II

Introducci on

inradio y el circunradio del tri angulo, respectivamente. Se incluyen tambi en varios problemas geom etricos cl asicos haciendo hincapi e en los m etodos utilizados para resolverlos. En el cap tulo 3 se presentan ciento veinte problemas de desigualdades que han aparecido en concursos recientes, cubriendo todos los niveles desde olimpiadas nacionales, regionales hasta competencias internacionales. En el cap tulo 4, se dan las soluciones a cada uno de los doscientos diez ejercicios de los cap tulos 1 y 2, as como a los problemas presentados en el cap tulo 3. La mayor a de las soluciones de los ejercicios o problemas que han aparecido en competencias internacionales de matem aticas se extrajeron de las soluciones ociales de cada uno de los concursos. Esta es la raz on por la cual no damos cr editos individuales por ellas. Una gran parte de los ejercicios y problemas de desigualdades se pueden resolver utilizando distintas t ecnicas, es por ello que encontrar a algunos ejercicios repetidos, pero en diferentes secciones. Esto le indicar a que puede encontrar, con la t ecnica desarrollada en la secci on correspondiente, una manera de resolver el ejercicio utilizando dicha herramienta. El material presentado en este libro ha sido acumulado durante los u ltimos quince a nos, principalmente durante las sesiones de trabajo con estudiantes que han ganado el concurso nacional de la Olimpiada Mexicana de Matem aticas; estos estudiantes se estaban preparando para las competencias internacionales en las que M exico participa. Quisieramos agradecer a Rafael Mart nez Enr quez, Leonardo Ignacio Mart nez Sandoval, David Mireles Morales, Jes us Rodr guez Viorato y Pablo Sober on Bravo por su cuidadosa revisi on del texto y sus valiosos comentarios, lo cual permiti o mejorar la presentaci on del libro. Radmila Bulajich Manfrino Jos e Antonio G omez Ortega Rogelio Valdez Delgado

Contenido

Introducci on 1. Desigualdades Num ericas 1.1. El orden en los n umeros reales . . . 1.2. La funci on cuadr atica ax2 + 2bx + c. 1.3. Una desigualdad fundamental, media geom etrica-media aritm etica . 1.4. Una desigualdad maravillosa, la desigualdad del reacomodo . . . . 1.5. Funciones convexas . . . . . . . . . 1.6. Una desigualdad u til . . . . . . . . . 1.7. La estrategia de sustituci on . . . . . 1.8. Teorema de Muirhead . . . . . . . .

III

. . . . . . . . . . . . . . . . . . . . . . . . . . . . . . . . . . . . . . . . . . . . . . . . . . . . . . . . . . . . . . . . . . . . . . . . . . . . . . . . . . . . . . . . . . . . . . . . . . . . . . . . . . . . . . . . . . . . . . . .

1 1 5 7 15 24 38 46 51

2. Desigualdades Geom etricas 2.1. Dos desigualdades b asicas . . . . . . . . . . . . . . 2.2. Desigualdades entre los lados de un tri angulo . . . 2.3. Uso de desigualdades en la geometr a del tri angulo 2.4. La desigualdad de Euler y algunas aplicaciones . . . 2.5. Funciones sim etricas de a, b y c . . . . . . . . . . . 2.6. Desigualdades con areas y per metros . . . . . . . 2.7. Teorema de Erd os-Mordell . . . . . . . . . . . . . 2.8. Problemas de optimizaci on . . . . . . . . . . . . . 3. Problemas Recientes de Desigualdades

. . . . . . . .

. . . . . . . .

. . . . . . . .

. . . . . . . .

. . . . . . . .

. . . . . . . .

59 . 59 . 63 . 68 . 77 . 82 . 87 . 93 . 102 115

IV

Contenido

4. Soluciones a los Ejercicios y Problemas 135 4.1. Soluciones a los ejercicios del cap tulo 1 . . . . . . . . . . . . . 135 4.2. Soluciones a los ejercicios del cap tulo 2 . . . . . . . . . . . . . 161 4.3. Soluciones a los problemas del cap tulo 3 . . . . . . . . . . . . . 187 Notaci on Bibliograf a Indice 235 237 239

Cap tulo 1

Desigualdades Num ericas

1.1.

El orden en los n umeros reales

Los n umeros reales tienen la importante propiedad de poseer un orden. El orden en los n umeros reales nos permitir a comparar dos n umeros y decidir cual de ellos es mayor o bien si son iguales. A n de evitar justicaciones tediosas, asumiremos que en los n umeros reales hay un conjunto P que llamaremos el conjunto de n umeros positivos, y simb olicamente escribiremos x > 0, para decir que un n umero x est a en P . Aceptaremos tambi en las tres propiedades siguientes. Propiedad 1.1.1 Cada n umero real x tiene una y s olo una de las siguientes caracter sticas: (i) x = 0. (ii) x P (esto es x > 0). (iii) x P (esto es x > 0). Propiedad 1.1.2 Si x,y P , entonces x + y P (en s mbolos x > 0, y > 0 x + y > 0). Propiedad 1.1.3 Si x, y P , entonces xy P (en s mbolos x > 0, y > 0 xy > 0). Si tenemos a la recta real como representaci on geom etrica de los n umeros reales, es decir, una recta dirigida donde se ha localizado el cero 0, el cual

Desigualdades Num ericas

divide en dos partes a la recta, los n umeros positivos son la parte que contiene al uno 1. Por lo general el 1 est a colocado a la derecha de 0 en el sentido que tiene la recta. Hacemos notar que el 1 es positivo, ya que si fuera negativo, como cumple con la propiedad 1 x = x para cada x, tendr amos que cualquier n umero x = 0 cumplir a con x P y x P , lo cual contradice la propiedad 1.1.1. Ahora podemos denir la relaci on, a es mayor que b, si a b P (en s mbolos a > b). An alogamente, a es menor que b, si b a P (en s mbolos a < b). Observemos que a < b es equivalente a b > a. Denimos tambi en a es menor o igual que b, si a < b o a = b, (en s mbolos a b). Denotamos a los n umeros reales por R y al conjunto P de n umeros reales positivos por R+ . Ejemplo 1.1.4 (i) Si a < b y c es cualquier n umero, entonces a + c < b + c. (ii) Si a < b y c > 0, entonces ac < bc. En efecto, para mostrar (i) tenemos que a + c < b + c (b + c) (a + c) > 0 b a > 0 a < b. Para ver (ii), tenemos que: a < b b a > 0 y como c > 0, resulta que (b a)c > 0, luego bc ac > 0 y entonces ac < bc. Ejercicio 1.1 Dados dos n umeros a y b, una y s olo una de las siguientes armaciones se cumple, a = b, a > b o a < b. Ejercicio 1.2 Muestre las siguientes armaciones: (i) a < 0, b < 0 ab > 0. (ii) a < 0, b > 0 ab < 0. (iii) a < b, b < c a < c. (iv ) a < b, c < d a + c < b + d. (v ) a < b b < a. 1 (vi) a > 0 > 0. a 1 (vii) a < 0 < 0. a a (viii) a > 0, b > 0 > 0. b (ix) 0 < a < b, 0 < c < d ac < bd. (x) a > 1 a2 > a. (xi) 0 < a < 1 a2 < a.

1.1 El orden en los n umeros reales

Ejercicio 1.3 (i) Si a > 0, b > 0 y a2 < b2 , entonces a < b. (ii) Si b > 0, olo si a > b. tenemos que a b > 1 si y s Para un n umero real x se dene el valor absoluto, el cual se denota por |x|, como |x| = x x para x 0 para x < 0.

Geom etricamente, |x| es la distancia del n umero x (en la recta real) al origen 0. Tambien, |a b| es la distancia entre los n umeros reales a y b en la recta real. Ejercicio 1.4 Para cualesquiera n umeros reales x, a y b, se tiene que: (i) |x| 0, y es igual a cero solamente cuando x = 0. (ii) |x| = |x|. (iii) |x|2 = x2 . (iv ) |ab| = |a| |b|. |a| a , con b = 0. = (v ) b |b|

Proposici on 1.1.5 (Desigualdad del tri angulo) Para dos n umeros reales a y b, siempre se tiene que |a + b| |a| + |b| . Adem as la igualdad ocurre solamente cuando ab 0. Demostraci on. Como ambos lados de la desigualdad son n umeros positivos; 2 por el ejercicio 1.3 bastar a entonces vericar que |a + b| (|a| + |b|)2 . |a + b|2 = (a + b)2 = a2 + 2ab + b2 = |a|2 + 2ab + |b|2 |a|2 + 2 |ab| + |b|2 = |a|2 + 2 |a| |b| + |b|2 = (|a| + |b|)2 . En las relaciones anteriores hay una sola desigualdad y esta es inmediata ya que ab |ab|. Notemos que cuando ab 0 se tiene que ab = |ab| = |a| |b|, y entonces se da la igualdad. La forma general de la desigualdad del tri angulo, para n umeros reales x1 , x2 , ... , xn , es |x1 + x2 + . . . + xn | |x1 | + |x2 | + . . . + |xn |.

Desigualdades Num ericas

La igualdad se tiene cuando todos los xi tienen el mismo signo. Esta se demuestra de manera similar, o bien por inducci on. Otra versi on de la desigualdad anterior que su usa muy a menudo es |x1 x2 . . . xn | |x1 | + |x2 | + . . . + |xn |. Ejercicio 1.5 Sean x, y , a, b n umeros reales muestre que: (i) |x| b b x b. (ii) ||a| |b|| |a b|. (iii) x2 + xy + y 2 0. (iv ) x > 0, y > 0 x2 xy + y 2 > 0. Ejercicio 1.6 Para n umeros reales a, b, c muestre que |a| + |b| + |c| |a + b| |b + c| |c + a| + |a + b + c| 0.

Ejercicio 1.7 Sean a, b n umeros reales con 0 a b 1, muestre que: ba 1, (i) 0 1 ab a b (ii) 0 + 1, 1+b 1+a 1 (iii) 0 ab2 ba2 . 4 Ejercicio si n, m son n umeros enteros positivos, entonces que 1.8 Muestre m+2n m 2 si y s olo si 2 < m+n . n < Ejercicio 1.9 Si a b, x y , entonces ax + by ay + bx. Ejercicio 1.10 Si x, y > 0, entonces
x2 y

y2 x

x + y.

Ejercicio 1.11 (Rep ublicas Checa y Eslovaca, 2004) Sean a, b, c, d R con a + d = b + c, muestre que (a b)(c d) + (a c)(b d) + (d a)(b c) 0.

1.2 La funci on cuadr atica ax2 + 2bx + c.

Ejercicio 1.12 Sea f (a, b, c, d) = (a b)2 + (b c)2 + (c d)2 + (d a)2 . Para a < b < c < d, muestre que f (a, c, b, d) > f (a, b, c, d) > f (a, b, d, c).

Ejercicio 1.13 (IMO, 1960) Para qu e valores de x se cumple la desigualdad 4x2 < 2x + 9? (1 1 + 2x)2 Ejercicio 1.14 Muestre que para cualquier entero positivo, la parte fraccionaria de 4n2 + n es menor que 1 4. Ejercicio 1.15 (Lista corta IMO, 1996). Sean a, b, c n umeros positivos que satisfacen abc = 1, muestre que a5 ab bc ca + 5 + 5 1. 5 5 + b + ab b + c + bc c + a5 + ca

1.2.

La funci on cuadr atica ax2 + 2bx + c.

Una desigualdad muy u til en los n umeros reales es x2 0, la cual es v alida para cualquier n umero real x (basta ver las propiedades 1.1.1 y 1.1.3 y el ejercicio 1.2 de la secci on anterior). De esta se deducen muchas otras desigualdades, en particular la usaremos para maximizar o minimizar una funci on cuadr atica ax2 +2bx + c. Estas funciones cuadr aticas aparecen con frecuencia en problemas de optimizaci on o en desigualdades. Un ejemplo com un consiste en probar que si a > 0, la funci on cuadr atica b2 b 2 nimo es c a . En ax + 2bx + c tendr a un m nimo en x = a y el valor m efecto, b2 b ax2 + 2bx + c = a x2 + 2 x + 2 a a b = a x+ a
2

+c b2 , a

b2 a

+c

6
2

Desigualdades Num ericas

b como x + a 0 y como el valor m nimo de esta u ltima expresi on es cero 2 b cuando x = a , tenemos que el valor m nimo de la cuadr atica es c ba . b y Si a < 0, la funci on cuadr atica ax2 + 2bx + c, tendr a un m aximo en x = a

el valor m as grande de esta u ltima puesto que a x + 2 expresi on es cero, luego la funci on cuadr atica siempre es menor o igual a c ba b . y toma ese valor en x = a Ejemplo 1.2.1 Si x, y son n umeros positivos con x + y = 2a, el producto xy es m aximo cuando x = y = a. Si x + y = 2a, entonces y = 2a x. Por lo que, xy = x(2a x) = x2 + 2ax = (x a)2 + a2 tiene un valor m aximo cuando x = a y entonces y = x = a. Esto se puede interpretar geom etricamente como: de los rect angulos de per metro jo, el de mayor area es el cuadrado. Ya que si x, y son los lados del rect angulo, el per metro es 2(x + y ) = 4a y su area es xy , que es m axima cuando x = y = a. Ejemplo 1.2.2 Si x, y son n umeros positivos con xy = 1, la suma x + y es m nima cuando x = y = 1. + 2, luego, 1 x + y es m nimo cuando x x = 0, esto es, cuando x = 1. Por lo tanto, x = y = 1. Tambi en podemos interpretar geom etricamente esto de la siguiente forma, de los rect angulos de area 1, el cuadrado es el de menor per metro. En efecto, si x, y son los lados del rect angulo, tenemos que su area es xy = 1 y su per metro 2 1 1 as, el per metro es 2(x + y ) = 2 x + x = 2 x x + 2 4. Adem 1 es 4 si y s olo si x = 0, esto es, cuando x = y = 1. x Si xy = 1, entonces y = As , x + y = x + = Ejemplo 1.2.3 Para cualquier n umero positivo x, se tiene que x +
1 x 1 x. 1 x

su valor ah es c

b2 2 a . En efecto, como ax 2 b 0 (ya que a < 0), a

+ 2bx + c = a x +

b 2 a

+c

b2 a

1 x

2.

2 1 1 + 2 2. Adem as, la igualdad se logra Basta observar que x + x x = x 1 si y s olo si x x = 0, esto es cuando x = 1.

Ejemplo 1.2.4 Si a, b > 0 entonces a = b.

a b

b a

2, y la igualdad se da si y s olo si

1.3 Media geom etrica-media aritm etica Basta aplicar el ejemplo anterior con x = a b.

Ejemplo 1.2.5 Dados a, b, c > 0, se puede construir un tri angulo con lados de longitud a, b y c si y s olo si pa2 + qb2 > pqc2 , para cualesquiera p, q con p + q = 1. Recordemos que a, b y c son las longitudes de los lados de un tri angulo si y s olo si a + b > c, a + c > b, b + c > a. Sea Q = pa2 + qb2 pqc2 = pa2 + (1 p)b2 p(1 p)c2 = c2 p2 + (a2 b2 c2 )p + b2 . Q es una funci on cuadr atica1 de p y Q > 0 = a2 b2 c2
2

[a + b + c] [a b c] [a b + c] [a + b c] < 0 [b + c a][c + a b][a + b c] > 0. Ahora [b + c a][c + a b][a + b c] > 0, si los tres factores son positivos o si uno es positivo y dos de ellos negativos. Esto u ltimo es imposible, pues si por ejemplo, [b + c a] < 0 y [c + a b] < 0, tendr amos al sumar estas desigualdades que c < 0, lo cual es falso. As los tres factores son necesariamente positivos. Ejercicio 1.16 Supongamos que el polinomio ax2 + bx + c satisface que: a > 0, a + b + c 0, a b + c 0, a c 0 y b2 4ac 0. Muestre que sus ra ces son reales y se encuentran en el intervalo 1 x 1. Ejercicio 1.17 Si a, b, c son n umeros positivos, entonces no suceden simult a1 1 , b(1 c) > 1 , c (1 a ) > . neamente las desigualdades a(1 b) > 4 4 4

a2 (b + c)2

a2 b2 c2 2bc

4b2 c2 < 0

a2 (b c)2 < 0

a2 b2 c2 + 2bc < 0

1.3.

Una desigualdad fundamental, media geom etrica-media aritm etica

La primera desigualdad que consideramos fundamental en los problemas de optimizaci on, es la desigualdad entre la media geom etrica y la media aritm etica
Una funci on cuadr atica ax2 + bx + c con a > 0, es positiva cuando su discriminante 2 4acb2 ` 2 + 4a . = b 4ac es negativo. En efecto, esto se sigue de que ax2 + bx + c = a x + 2b a b b2 4ac Recuerde que las ra ces son , y son reales cuando 0, en caso contrario no 2a hay ra ces reales, por lo que ax2 + bx + c toma un mismo signo, esta expresi on ser a positiva si a > 0.
1

Desigualdades Num ericas

de dos n umeros no negativos a y b, que est a dada por a+b . (M G M A) ab 2 Adem as la igualdad ocurre si y s olo si a = b. a+b etrica y la media Los n umeros ab y 2 se conocen como la media geom aritm etica de a y b, respectivamente. Para demostrar la desigualdad basta observar que, 2 a + b 2 ab a+b 1 a b 0. ab = = 2 2 2 Desde luego la igualdad se logra si y s olo si a = b, es decir, cuando a = b. Ejercicio 1.18 Para x 0, 1 + x 2 x. Ejercicio 1.19 Para x > 0, x +
1 x

2.

Ejercicio 1.20 Para x, y R+ , x2 + y 2 2xy . Ejercicio 1.21 Para x, y R+ , 2(x2 + y 2 ) (x + y )2 . Ejercicio 1.22 Para x, y R+ ,
1 x

1 y

b x

4 x +y .

Ejercicio 1.23 Para a, b, x R+ , ax + Ejercicio 1.24 Para a, b > 0,


a b

2 ab.
2

b a

2.
a+b 2

Ejercicio 1.25 Para 0 < b a,

1 (ab)2 8 a

ab

1 (ab) 8 b

Presentamos ahora una demostraci on geom etrica y visual de las siguientes desigualdades, para x, y > 0,
1 x

2 +

1 y

x+y . xy 2

(1.1)

A h E O y C

g B x D

1.3 Media geom etrica-media aritm etica

Construyamos una semicircunferencia de di ametro BC = x + y y sean x = BD, y = DC . Sea A el punto donde la perpendicular a BC por D intersecta a la semicircunferencia y sea E el pie de la perpendicular desde D al radio AO. Denotemos por g = AD, h = AE . Por ser ABD y CAD tri angulos rect angulos semejantes, tenemos que x g = , y g luego g= xy.

Tambi en, por ser AOD y ADE tri angulos rect angulos semejantes, tenemos xy h 2 2xy . = luego h= = x +y , 1 xy x+y +1 2
x y

Finalmente, la geometr a nos dice que en un tri angulo rect angulo un cateto x +y siempre es menor que la hipotenusa. Por lo que, h g 2 , que reescribimos como x+y 2 xy . 1 1 2 x + y El n umero
1 1 +y x

se conoce como la media arm onica de x y y . La desigualdad

de la izquierda en la ecuaci on (1.1) se conoce como la desigualdad entre la media arm onica y la media geom etrica. Algunas desigualdades se pueden demostrar a trav es de repetir la aplicaci on de una desigualdad sencilla, adem as del uso de una buena idea para dividir el problema, esto se puede ver resolviendo los siguientes ejercicios. Ejercicio 1.26 Para x, y , z R+ , (x + y )(y + z )(z + x) 8xyz . Ejercicio 1.27 Para x, y , z R, x2 + y 2 + z 2 xy + yz + zx. Ejercicio 1.28 Para x, y , z R+ , xy + yz + zx x yz + y zx + z xy . Ejercicio 1.29 Para x, y R, x2 + y 2 + 1 xy + x + y . Ejercicio 1.30 Para x, y , z R+ , Ejercicio 1.31 Para x, y , z R+ ,
1 x

1 y

+
yz x

1 z

zx y

1 xy

1 yz

1 . zx

xy z

Ejercicio 1.32 Para x, y , z R, x2 + y 2 + z 2 x y 2 + z 2 + y x2 + z 2 . La desigualdad entre la media geom etrica y la media aritm etica se puede extender para m as n umeros. Por ejemplo, podemos ver la siguiente desigualdad entre

x + y + z.

10

Desigualdades Num ericas

la media aritm etica y la media geom etrica para cuatro n umeros no negativos a, 4 c +d abcd , de la siguiente manera: b, c, d, que arma que a+b+ 4 1 a+b+c+d = 4 2 a+b c+d + 2 2 1 ab + cd 2 4 ab cd = abcd.

Observemos que usamos tres veces la desigualdad M G M A para dos n umeros, en cada caso: con a y b, con c y d, y con ab y cd. M as a un, la igualdad se da si y s olo si a = b, c = d y ab = cd, esto es cuando los n umeros satisfacen a = b = c = d. Ejercicio 1.33 Para x, y R, x4 + y 4 + 8 8xy . Ejercicio 1.34 Para a, b, c, d R+ , (a + b + c + d) Ejercicio 1.35 Para a, b, c, d R+ ,
a b 1 a

1 b

1 c

1 d

16.

b c

c d

d a

4.

Hay un muy buen truco para ver que la desigualdad tambi en es verdadera para 3 b+c tres n umeros no negativos a, b y c, esto es, que abc a+3 . Considere 3 la desigualdad es los siguientes cuatro n umeros a, b, c y d = abc. Como 4 3 4 a+b+c+d verdadera para cuatro n umeros, tenemos abcd = d d = d. 4 b+c 3 a+b+c Luego, a+4 d 1 d = d , por lo que, d = 3 abc. 4 4 3 Estas ideas se pueden usar para justicar la versi on general de la desigualdad para n n umeros no negativos. Si a1 , a2 , ... , an son n n umeros no negativos, tomamos los n umeros A y G denidos como A= a1 + a2 + + an n y G= n a1 a2 an .

Estos n umeros se conocen como la media aritm etica y la media geom etrica de los n umeros a1 , a2 , ... , an , respectivamente. Teorema 1.3.1 (Desigualdad entre las medias geom etrica y aritm etica) n a1 a2 an a1 + a2 + + an . n

Primera demostraci on. (de Cauchy) Llamemos Pn a la armaci on G A, para n n umeros. La demostraci on la haremos por inducci on sobre n, pero una inducci on del siguiente tipo: (1) Se muestra que la armaci on vale para 2 n umeros, esto es que P2 es cierta.

1.3 Media geom etrica-media aritm etica (2) Se muestra que la armaci on: Pn Pn1 . (3) Se muestra que la armaci on: Pn P2n .

11

Cuando es v alido (1), (2) y (3), todas las armaciones Pn con n 2 son verdaderas. Demostramos ahora estas armaciones: (1) Este apartado lo hemos hecho en la primera parte. (2) Sean a1 , a2 , ... , an n umeros no negativos y sea g = n1 a1 an1 . Con este n umero agregado a los n umeros a1 , ... , an1 obtenemos n n umeros a los que aplicamos Pn , a1 + + an1 + g n a1 a2 an1 g = n
n

gn1 g = g.
a1 ++an1 n1

Luego, a1 + + an1 + g ng y entonces se sigue que por lo que Pn1 es verdadera. (3) Sean a1 , a2 , ... , a2n n umeros no negativos, entonces

g,

a1 + a2 + + a2n = (a1 + a2 ) + (a3 + a4 ) + + (a2n1 + a2n ) 2 ( a1 a2 + a3 a4 + + a2n1 a2n ) 1 2n ( a1 a2 a3 a4 a2n1 a2n ) n = 2n (a1 a2 a2n ) 2n . Hemos aplicado primero varias veces la armaci on P2 que sabemos es cierta y despu es la armaci on Pn a los n umeros a1 a2 , a3 a4 , ... , a2n1 a2n .
+an Segunda demostraci on. Sea A = a1 + . Tomemos dos de los n umeros n ai , uno menor que A y otro mayor que A, si es que existen. Sin p erdida de generalidad, supongamos que son a1 = A h y a2 = A + k con h, k > 0. Cambiaremos a1 y a2 por otros dos n umeros que incrementen el producto y que dejen ja la suma, denimos a 1 = A, a2 = A + k h. Como a 1 + a2 = A + A + k h = A h + A + k = a1 + a2 , es claro que, a1 + a2 + a3 + + an = a1 + a2 + a3 + + an , pero a 1 a2 = A(A + k h) = 2 2 A + A(k h) y a1 a2 = (A + k)(A h) = A + A(k h) hk, luego, a 1 a2 > a1 a2 y entonces a1 a2 a3 an > a1 a2 a3 an .
1

12

Desigualdades Num ericas

Si A = a 1 = a2 = a3 = = an , ya no hay nada que demostrar (se da la igualdad), en caso contrario hay dos elementos, uno mayor que A y otro menor que A, y repetimos el argumento. Como cada vez que realizamos esta operaci on se crea un n umero igual a A, el proceso no puede ir m as all a de n pasos.

Ejemplo 1.3.2 Encuentre el valor m aximo de x(1 x3 ) para 0 x 1. La idea de la prueba es transformar el producto en uno cuya suma sea constante. Si y = x(1 x3 ), es claro que el lado derecho de 3y 3 = 3x3 (1 x3 )(1 x3 )(1 x3 ), visto como el producto de cuatro n umeros 3x3 , (1 x3 ), (1 x3 ) y (1 x3 ), tiene suma constante igual a 3. La desigualdad entre la media geom etrica y la media aritm etica para los cuatro n umeros nos garantiza que 3y luego, y
3 . Adem as, 434 1 x= 3 . 4 3

3x3 + 3(1 x3 ) 4

3 4

esto es, si

el valor m aximo se alcanza cuando 3x3 = 1 x3 ,

Ejercicio 1.36 Sean xi > 0, i = 1, ..., n. Muestre que ( x1 + x 2 + + x n ) 1 1 1 + + + x1 x2 xn n2 .

Ejercicio 1.37 Si {a1 , ..., an } es una permutaci on de {b1 , ..., bn } R+ , entonces an b1 b2 bn a1 a2 + + + n y + + + n. b1 b2 bn a1 a2 an Ejercicio 1.38 Si a > 1, entonces an 1 > n a
n+1 2

n1 2

Ejercicio 1.39 Si a, b, c > 0 y (1 + a)(1 + b)(1 + c) = 8, entonces abc 1. Ejercicio 1.40 Si a, b, c > 0 entonces Ejercicio 1.41 Si a, b, c 0, entonces a2 b2 + b2 c2 + c2 a2 abc(a + b + c).
a3 b

b3 c

c3 a

ab + bc + ca.

1.3 Media geom etrica-media aritm etica

13

Ejercicio 1.42 Si a, b, c > 0, entonces a2 b + b2 c + c2 a ab2 + bc2 + ca2 9a2 b2 c2 .

Ejercicio 1.43 Sean a, b, c > 0 tales que abc = 1. Muestre que 1 + ab 1 + bc 1 + ac + + 3. 1+a 1+b 1+c

Ejercicio 1.44 Si a, b, c > 0, muestre que 9 2 a+b+c 1 1 1 + + a+b b+c c+a 1 1 1 + + . a b c

Ejercicio 1.45 Si Hn = 1 +

1 2
1

1 + + n , muestre que

n(n + 1) n < n + Hn , para

n 2.
1 1+x1 1 1+xn

Ejercicio 1.46 Si x1 , x2 , ... , xn > 0, son tales que Muestre que x1 x2 xn (n 1)n .

+ +

= 1.

Ejercicio 1.47 (Lista corta IMO, 1998) Sean a1 , a2 , ... , an n umeros positivos con a1 + a2 + + an < 1, muestre que 1 a1 a2 an [1 (a1 + a2 + + an )] n+1 . (a1 + a2 + + an ) (1 a1 ) (1 a2 ) (1 an ) n Ejercicio 1.48 Sean a1 , a2 , ... , an n umeros positivos, tales que 1 1+an = 1. Muestre que a1 + + an (n 1) 1 1 + + a1 an .
1 1+a1

+ +

14

Desigualdades Num ericas

Ejercicio 1.49 (APMO, 1991) Sean a1 , a2 , ... , an , b1 , b2 , ... , bn n umeros positivos, tales que a1 + a2 + + an = b1 + b2 + + bn . Muestre que a2 1 a2 n 1 + + (a1 + + an ). a1 + b1 an + bn 2

Ejercicio 1.50 Sean a, b, c n umeros positivos, muestre que 1 1 1 1 + + . a3 + b3 + abc b3 + c3 + abc c3 + a3 + abc abc

Ejercicio 1.51 Sean a, b, c n umeros positivos con a + b + c = 1, muestre que 1 +1 a 1 +1 b 1 +1 c 64.

Ejercicio 1.52 Sean a, b, c n umeros positivos con a + b + c = 1, muestre que 1 1 a 1 1 b 1 1 c 8.

Ejercicio 1.53 (Rep ublicas Checa y Eslovaca, 2005). Sean a, b, c n umeros positivos que satisfacen abc = 1, muestre que a b c 3 + + . (a + 1)(b + 1) (b + 1)(c + 1) (c + 1)(a + 1) 4

Ejercicio 1.54 Sean a, b, c n umeros positivos que satisfacen 1, muestre que abc 8. Ejercicio 1.55 Sean a, b, c n umeros positivos, muestre que 2ab 2bc 2ca + + a + b + c. a+b b+c c+a

1 1 1 1+a + 1+b + 1+c

1.4 Una desigualdad maravillosa

15

Ejercicio 1.56 Sean a1 , a2 , ... , an , b1 , b2 , ... , bn n umeros positivos, muestre que n n 1 (ai + bi )2 4n2 . ai bi
i=1 i=1

Ejercicio 1.57 (Rusia, 1991) Sean x, y , z n umeros reales no negativos, muestre que (x + y + z )2 x yz + y zx + z xy. 3

Ejercicio 1.58 (Rusia, 1992) Para n umeros reales positivos x, y , z , muestre que x4 + y 4 + z 2 8xyz.

Ejercicio 1.59 (Rusia, 1992) Muestre que para cualesquiera n umeros reales x, y > 1, se tiene x2 y2 + 8. y1 x1

1.4.

Una desigualdad maravillosa, la desigualdad del reacomodo

Considere dos colecciones de n umeros reales ordenados en forma creciente, a1 a2 an y b1 b2 b n .

Para cualquier permutaci on (a 1 , a2 , ..., an ) de (a1 , a2 , ..., an ), se tiene que a1 b1 + a2 b2 + + an bn a 1 b1 + a2 b2 + + an bn

(1.2) (1.3)

an b1 + an1 b2 + + a1 bn

Adem as, la igualdad en (1.2) es cierta si y s olo si (a 1 , a2 , ..., an ) = (a1 , a2 , ..., an ). Y la igualdad en (1.3) es cierta si y s olo si (a1 , a2 , ..., an ) = (an , an1 , ..., a1 ). A la desigualdad (1.2) se le conoce como la desigualdad del reacomodo.

16

Desigualdades Num ericas

Corolario 1.4.1 Para cualquier permutaci on (a 1 , a2 , ..., an ) de (a1 , a2 , ..., an ), se tiene que 2 2 a2 1 + a2 + + an a1 a1 + a2 a2 + + an an . Corolario 1.4.2 Para cualquier permutaci on (a 1 , a2 , ..., an ) de (a1 , a2 , ..., an ), se tiene que a a a 1 + 2 + + n n. a1 a2 an

Demostraci on (de la desigualdad del reacomodo) Supongamos que b1 b2 bn . Sean S S = a1 b1 + a2 b2 + + ar br + + as bs + + an bn

= a1 b1 + a2 b2 + + as br + + ar bs + + an bn .

La diferencia entre S y S es que los coecientes de br y bs , donde r < s, est an intercambiados. Por lo tanto, S S = ar br + as bs as br ar bs = (bs br )(as ar ). Entonces podemos armar que S S si y s olo si as ar . Repitiendo este proceso tenemos que la suma S es la mayor cuando a1 a2 an . Ejemplo 1.4.3 (IMO, 1975) Considere dos colecciones de n umeros x1 x2 xn , y1 y2 yn y una permutaci on (z1 , z2 , ..., zn ) de (y1 , y2 , ..., yn ). Muestre que (x1 y1 )2 + + (xn yn )2 (x1 z1 )2 + + (xn zn )2 . Elevando al cuadrado y reacomodando la desigualdad anterior, tenemos que es equivalente a
n i=1 n n n 2 yi i=1 n n

x2 i 2
n 2 i=1 yi

xi y i +
i=1 n 2 i=1 zi , n i=1 i=1

x2 i 2

xi zi +
i=1 i=1

2 zi ,

pero como = es equivalente a demostrar

entonces la desigualdad que tenemos que probar


n

xi zi

xi yi ,
i=1

que es la desigualdad (1.2) del teorema.

1.4 Una desigualdad maravillosa

17

Ejemplo 1.4.4 (IMO, 1978) Sean x1 , x2 , ... , xn enteros positivos distintos, muestre que 1 xn 1 1 x1 x2 + 2 + + 2 + + + . 12 2 n 1 2 n Sea (a1 , a2 , . . . , an ) una permutaci on de (x1 , x2 , . . . , xn ) con a1 a2 1 1 1 1 an y sea (b1 , b2 , . . . , bn ) = n2 , (n1)2 , , 1 2 , esto es bi = (n+1i)2 para los ndices i = 1, . . . , n. Considere la permutaci on (a 1 , a2 , . . . , an ) de (a1 , a2 , . . . , an ) denida por ai = xn+1i , para i = 1, . . . , n. Utilizando la desigualdad (1.3) tenemos que xn x1 x2 + 2 + + 2 2 1 2 n
= a 1 b1 + a2 b2 + + an bn

an b1 + an1 b2 + + a1 bn

= a1 bn + a2 bn1 + + an b1 an a1 a2 + 2 + + 2. = 2 1 2 n

Como 1 a1 , 2 a2 , ... , n an , tenemos que x1 x2 1 xn a1 a2 an 1 2 n 1 1 + + + 2 2 + 2 + + 2 2 + 2 + + 2 = + + + . 12 22 n 1 2 n 1 2 n 1 2 n Ejemplo 1.4.5 (IMO, 1964) Sean a, b, c las longitudes de los lados de un tri angulo. Muestre que a2 (b + c a) + b2 (a + c b) + c2 (a + b c) 3abc. Como la expresi on es una funci on sim etrica en a, b y c, podemos suponer sin perder generalidad que c b a. En tal caso, a (b + c a) b (a + c b) c (a + b c). Por ejemplo, la primera desigualdad se justica como sigue a (b + c a) b (a + c b) ab + ac a2 ab + bc b2 (a b) (a + b c) 0. (a b) c (a + b) (a b)

Por la ecuaci on (1.3) de la desigualdad del reacomodo, tenemos que a2 (b+ca)+b2 (c+ab)+c2 (a+bc) ba(b+ca)+cb(c+ab)+ac(a+bc) Por lo tanto, 2 a2 (b + c a) + b2 (c + a b) + c2 (a + b c) 6abc. a2 (b+ca)+b2 (c+ab)+c2 (a+bc) ca(b+ca)+ab(c+ab)+bc(a+bc).

18

Desigualdades Num ericas

Ejemplo 1.4.6 (IMO, 1983) Sean a, b, c los lados de un tri angulo, muestre que a2 b(a b) + b2 c(b c) + c2 a (c a) 0. Consideremos el caso c b a, (los otros casos se desarrollan de manera similar). Como en el ejemplo anterior, tenemos que a(b+ca) b(a+cb) c(a+bc) 1 1 1 y como adem as a b c , por la desigualdad (1.2) obtenemos 1 1 1 a(b + c a) + b(c + a b)+ c(a + b c) a b c 1 1 1 a(b + c a) + b(c + a b) + c(a + b c). c a b Por lo tanto, a (b a) b(c b) c (a c) + + + a + b + c. c a b a(b a) b(c b) c(a c) De donde tenemos que + + 0. Multiplicando por c a b abc, obtenemos a+b+c a2 b (a b) + b2 c(b c) + c2 a(c a) 0. Ejemplo 1.4.7 (Desigualdad de Cauchy-Schwarz ) Para cualesquiera n umeros reales x1 , . . ., xn , y1 , . . ., yn , la siguiente desigualdad se cumple
n 2 n n

xi yi
i=1

x2 i
i=1 i=1

2 yi

La igualdad se da si y s olo si existe una R con xi = yi , para cada i = 1, 2, ..., n. Si x1 = x2 = = xn = 0 o y1 = y2 = = yn = 0 el resultado es evidente.
n n 2 2 De lo contrario, sean S = i=1 xi y T = i=1 yi , claramente S = 0 y yi xi T = 0. Denamos ai = S y an+i = T , para i = 1, 2, ..., n. Usando el corolario 1.4.1, n

2 =
i=1

x2 i + S2

n i=1

2 yi = T2

2n

a2 i
i=1

a1 an+1 + a2 an+2 + + an a2n + an+1 a1 + + a2n an = 2 x1 y1 + x2 y2 + + xn yn . ST

1.4 Una desigualdad maravillosa

19

La igualdad ocurre si y s olo si ai = an+i , para i = 1, 2, . . . , n, y esto es cierto S yi , para i = 1, 2, . . . , n. si y s olo si xi = T Podemos dar otra demostraci on de la desigualdad de Cauchy-Schwarz utilizando la identidad de Lagrange
n 2 n n

xi y i
i=1

=
i=1

x2 i
i=1

2 yi

1 2

i=1 j =1

(xi yj xj yi )2 .

La importancia de la desigualdad de Cauchy-Schwarz ser a evidente a lo largo del libro. Como veremos, es una herramienta u til para resolver un gran n umero de ejercicios y problemas. Ejemplo 1.4.8 (Desigualdad de Nesbitt) Para a, b, c R+ , se tiene que a b c 3 + + . b+c c+a a+b 2 Sin perder generalidad, podemos suponer que a b c, de esta hip otesis se 1 1 1 . sigue que a + b c + a b + c y b+ c c +a a+b Usando la desigualdad del reacomodo (1.2) dos veces, tenemos b c b c a a + + + + b+c c+a a+b b+c c+a a+b a b c c a b + + + + . b+c c+a a+b b+c c+a a+b Por lo tanto, 2 b c a + + b+c c+a a+b b+c c+a a+b + + b+c c+a a+b = 3.

Otra forma, de demostrar la desigualdad es usando la desigualdad (1.3) dos veces, c+a a+b b+c + + b+c c+a a+b a+b b+c c+a + + b+c c+a a+b 3 3.

2b+c+a 2c+a+b +b+c Luego, al sumar las dos expresiones obtenemos 2ab +c + c+a + a+b 6, y entonces 2b 2c 2a + + 3. b+c c+a a+b

20

Desigualdades Num ericas

Ejemplo 1.4.9 (IMO, 1995) Sean a, b, c R+ , con abc = 1. Muestre que a3 (b 1 1 1 3 + 3 + 3 . + c) b (c + a) c (a + b) 2

Sin perder generalidad podemos suponer c b a. Sean x = z=1 c , entonces S = = a3 (b 1 1 1 + 3 + 3 + c) b (c + a) c (a + b) + y3 1 1 + z x + z3 1 1 + x y

1 a,

y =

1 b

x3 1 1 + y z

y2 z2 x2 + + . y+z z+x x+y

Como x y z , tenemos que x + y z + x y + z y tambi en que y z x . Por la desigualdad del reacomodo (1.2), obtenemos y +z z +x x +y x2 y2 z2 + + y+z z+x x+y x2 y2 z2 + + y+z z+x x+y xy yz zx + + y+z z+x x+y

xz yx zy + + , z+x z+x x+y que a su vez nos lleva a 2S x + y + z 3 3 xyz = 3. Por lo tanto, S 3 2. Ejemplo 1.4.10 (APMO, 1998) Sean a, b, c R+ , muestre que 1+ Observemos que 1+ 1+ a b 1+ b c 1+ + a+b+c c 2 1+ 3 a abc + a+b+c abc 2 1+ 3 abc abc a b 1+ b c 1+ a+b+c c 2 1+ 3 a abc .

a b c + + b c a

a c b + + c b a

2(a + b + c) a b c a c b + + + + + . 3 b c a c b a abc

1.4 Una desigualdad maravillosa Deniendo a = x3 , b = y 3 , c = z 3 . Tenemos ahora que mostrar que 2 x3 + y 3 + z 3 x3 y 3 z3 x3 z 3 y3 + + + + + . y3 z3 x3 z3 y 3 x3 xyz Pero, si consideramos (a1 , a2 , a3 , a4 , a5 , a6 ) =
(a 1 , a2 , a3 , a4 , a5 , a6 ) = y z x z y x z, x, y, y, x, z x2 y2 y2 z2 x2 x2 z2 z2 y2 y2 x y z x z y y , z , x, z , y, x

21

y (b1 , b2 , b3 , b4 , b5 , b6 ) =

, z2 ,

, x2 , tenemos que

x3 y 3 z3 x3 z 3 y3 x2 y y 2 z z 2 x x2 z z2 y y2 x + + + + + + + + + + y3 z3 x3 z3 y 3 x3 y 2 z z 2 x x2 y z 2 y y 2 x x2 z = = x2 y2 z2 x2 z2 y2 + + + + + yz zx xy zy yx xz 2 x3 + y 3 + z 3 . xyz

Ejemplo 1.4.11 (Desigualdad de Tchebyshev) Sean a1 a2 an y b1 b2 bn , entonces a1 + a2 + + an b1 + b2 + + bn a1 b1 + a2 b2 + + an bn . n n n Aplicando varias veces la desigualdad del reacomodo obtenemos a1 b1 + + an bn = a1 b1 + a2 b2 + + an bn

a1 b1 + + an bn a1 b3 + a2 b4 + + an b2 . . . . . . . . .

a1 b1 + + an bn a1 b2 + a2 b3 + + an b1

a1 b1 + + an bn a1 bn + a2 b1 + + an bn1 , al sumar todas las expresiones, obtenemos n (a1 b1 + + an bn ) (a1 + + an ) (b1 + + bn ) . Notemos que la igualdad ocurre solamente cuando a1 = a2 = = an o b1 = b2 = = bn . a3 + b3 + c3 a2 b + b2 c + c2 a.

Ejercicio 1.60 Cualesquiera tres n umeros reales positivos a, b y c cumplen que

22

Desigualdades Num ericas

Ejercicio 1.61 Cualesquiera tres n umeros reales positivos a, b y c, con abc = 1, satisfacen que a3 + b3 + c3 + (ab)3 + (bc)3 + (ca)3 2(a2 b + b2 c + c2 a).

Ejercicio 1.62 Cualesquiera tres n umeros reales positivos a, b y c cumplen que c2 b c a a2 b2 + + + + . 2 2 2 b c a a b c

Ejercicio 1.63 Cualesquiera tres n umeros reales positivos a, b y c cumplen que 1 1 1 a+b+c 2 + 2 + 2. abc a b c

Ejercicio 1.64 Si a, b y c son la longitudes de los lados de un tri angulo, muestre que a b c + + 3. b+ca c+ab a+bc Ejercicio 1.65 Si a1 , a2 , ... , an R+ y s = a1 + a2 + + an , entonces a2 an n a1 . + + + s a1 s a2 s an n1

Ejercicio 1.66 Si a1 , a2 , ... , an R+ y s = a1 + a2 + + an , entonces s s n2 s . + + + s a1 s a2 s an n1 Ejercicio 1.67 Si a1 , a2 , ... , an R+ y a1 + a2 + + an = 1, entonces a2 an n a1 . + + + 2 a1 2 a2 2 an 2n 1

1.4 Una desigualdad maravillosa

23

Ejercicio 1.68 (Desigualdad media cuadr atica-media aritm etica) Sean x1 , x2 , ... , xn R+ , entonces
2 2 x1 + x 2 + + x n x2 1 + x2 + + x n . n n

Ejercicio 1.69 Para n umeros reales positivos a, b, c, que satisfacen a+b+c = 1, muestre que 1 ab + bc + ca . 3 Ejercicio 1.70 (Media arm onica, geom etrica y aritm etica) Sean x1 , x2 , ... , xn R+ , muestre que
1 x1

1 x2

n + +

1 xn

x1 + x2 + + xn n . x1 x2 xn n

Donde las igualdades se dan si y s olo si x1 = x2 = = xn . Ejercicio 1.71 Sean a1 , a2 , ... , an n umeros positivos con a1 a2 an = 1. Muestre que
n1 n1 n1 a1 + a2 + + an

1 1 1 + + + . a1 a2 an

umeros positivos, tales que Ejercicio 1.72 (China, 1989) Sean a1 , a2 , ... , an n a1 + a2 + + an = 1. Muestre que a1 an 1 ( a1 + + an ). + + 1 a1 1 an n1

Ejercicio 1.73 Sean a, b y c n umeros positivos tales que a + b + c = 1. Muestre que: (i) 4a + 1 + 4b + 1 + 4c + 1 < 5, (ii) 4a + 1 + 4b + 1 + 4c + 1 21.

24

Desigualdades Num ericas

Ejercicio 1.74 Sean a, b, c, d R+ , con ab + bc + cd + da = 1, muestre que b3 c3 d3 1 a3 + + + . b+c+d a+c+d a+b+d a+b+c 3

Ejercicio 1.75 Sean a, b, c n umeros positivos, con abc = 1, muestre que a b c + + a + b + c. b c a

Ejercicio 1.76 Sean x1 , x2 , ... , xn (n > 2) n umeros reales tales que la suma de cualesquiera n 1 de ellos es mayor que el elemento que quedo fuera de la suma. Sea s = n k =1 xk . Muestre que
n k =1

x2 s k . s 2xk n2

1.5.

Funciones convexas

Una funci on f : [a, b] R es convexa en el intervalo I = [a, b], si para cada t [0, 1] y para a x < y b, se tiene la desigualdad f (ty + (1 t)x) tf (y ) + (1 t)f (x). (1.4) Geom etricamente, la desigualdad anterior se interpreta diciendo que la gr aca de f entre x y y queda por debajo del segmento que une a los puntos (x, f (x)) y (y, f (y )).

(x, f (x))

(y, f (y ))

1.5 Funciones convexas En efecto, la ecuaci on de la recta que pasa por (x, f (x))y (y, f (y )) es L(s) = f (x) + f (y ) f (x) (s x). yx

25

Luego, al evaluar en s = ty + (1 t)x, obtenemos que L(ty + (1 t)x) = f (x) + f (y ) f (x) (t(y x)) = f (x) + t(f (y ) f (x)) yx = tf (y ) + (1 t)f (x).

Por lo que, la desigualdad (1.4) es equivalente a f (ty + (1 t)x) L(ty + (1 t)x). Proposici on 1.5.1 (1) Si f es convexa en el intervalo [a, b], entonces es convexa en cualquier subintervalo [x, y ] [a, b] . (2) Si f es convexa en [a, b], entonces, para cualesquiera x, y [a, b], se tiene f x+y 2 1 (f (x) + f (y )). 2 (1.5)

(3) (Desigualdad de Jensen) Si f es convexa en [a, b], entonces para cualesquiera t1 , ... , tn [0, 1] con n i=1 ti = 1 y para x1 , ... , xn [a, b], se tiene f (t1 x1 + + tn xn ) t1 f (x1 ) + + tn f (xn ). (4) En particular, para x1 , ... , xn [a, b], podemos armar que f x1 + + x n n 1 (f (x1 ) + + f (xn )) . n

Demostraci on. (1) Se deja como ejercicio al lector. (2) Basta tomar t =
1 2

en la ecuaci on (1.4).
tn1 1tn xn1 )

t1 (3) f (t1 x1 + + tn xn ) = f ((1 tn ) ( 1 tn x1 + +

(1 tn ) f

(1 tn ) = t1 f (x1 ) + + tn f (xn ).

+ tn xn ) tn1 + + 1tn xn1 + tn f (xn ), por convexidad tn1 t1 on 1tn f (x1 ) + + 1tn f (xn1 ) + tn f (xn ), por inducci
t1 1tn x1

1 . (4) Solamente tenemos que aplicar (3) con t1 = t2 = = tn = n

26

Desigualdades Num ericas

Observaciones 1.5.2 (i) Podemos ver que (4) es v alida bajo la u nica suposif (x)+f (y ) y , para cualesquiera x, ci on de que f satisfaga la relaci on f x+ 2 2 y [a, b]. (ii) Podemos ver que (3) es v alida para t1 , . . . , tn [0, 1] n umeros racionales, f (y ) x +y bajo la u nica condici on de que f satisfaga la relaci on f 2 f (x)+ para 2 cualesquiera x, y [a, b]. La demostraci on de (i), la haremos por inducci on. Llamemos Pn a la armaci on f x1 + + x n n 1 (f (x1 ) + + f (xn )) , n

para x1 , . . ., xn [a, b] . Es claro que son v alidas las armaciones P1 y P2 . Ahora, mostremos que Pn Pn1 . +xn1 . Entonces, por ser cierto Pn , Sean x1 , . . ., xn [a, b] y sea y = x1 + n1 tenemos f x1 + + xn1 + y n 1 1 1 f (x1 ) + + f (xn1 ) + f (y ). n n n

Pero el lado izquierdo es f (y ), por lo que n f (y ) f (x1 )+ + f (xn1 )+ f (y ), luego 1 f (y ) (f (x1 ) + + f (xn1 )) . n1 Finalmente veamos que Pn P2n . Sea D = f
xn+1 ++x2n . n x1 ++xn +xn+1 ++x2n 2n

=f

u+v 2

, donde u =

x1 ++xn n

yv =

Como f D

u+v 2

1 2 (f (u) + f (v )), tenemos que

x1 + + x n xn+1 + + x2n 1 1 (f (u) + f (v )) = f +f 2 2 n n 1 (f (x1 ) + + f (xn ) + f (xn+1 ) + + f (x2n )) , 2n

donde hemos usado dos veces la armaci on Pn . Para demostrar (ii), nuestro punto de partida ser a considerar la armaci on +x n 1 f x1 + ( f ( x ) + + f ( x )) , para x , . . . , x [ a, b ] y n N . 1 n 1 n n n n r1 rn , n umeros racionales en [0, 1] con , . . . , tn = s Sean t1 = s i=1 ti = 1. n 1 pi Si m es el m nimo com un m ultiplo de las si , entonces ti = m con pi N y

1.5 Funciones convexas


n i=1 pi

27

= m, luego pn p1 x1 + + xn m m

f (t1 x1 + + tn xn ) = f

1 = f (x1 + + x1 ) + + (xn + + xn ) m
p1 t erminos pn t erminos

p1 pn = f (x1 ) + + f (xn ) m m = t1 f (x1 ) + + tn f (xn ).

1 (f (x1 ) + + f (x1 )) + + (f (xn ) + + f (xn )) m


p1 t erminos pn t erminos

Observaci on 1.5.3 Si f : [a, b] R es una funci on continua2 en [a, b] y cumple el inciso (2) de la proposici on, entonces f es convexa. Hemos visto que si f cumple (2), entonces f (qx + (1 q )y ) qf (x) + (1 q )f (y ), para cualesquiera x, y [a, b] y q [0, 1] un n umero racional. Como cualquier real t se puede aproximar por una sucesi on de n umeros racionales qn , tenemos que si estos qn est an en [0, 1], entonces f (qn x + (1 qn )y ) qn f (x) + (1 qn )f (y ). Ahora bien, la continuidad de f nos garantiza que al tomar el l mite obtenemos f (tx + (1 t)y ) tf (x) + (1 t)f (y ). Decimos que f : [a, b] R es c oncava si f es convexa. Observaci on 1.5.4 Una funci on f [a, b] R es c oncava en [a, b] si y s olo si f (ty + (1 t)x) tf (y ) + (1 t)f (x), para 0 t 1 y a x < y b. Veamos ahora algunos criterios para decidir si una funci on es convexa.
2 Una funci on f : [a, b] R es continua en el punto c [a, b] si l mxc f (x) = f (c), y f es continua en [a, b] si es continua para todo punto en el intervalo. Equivalentemente, f es continua en c, si para toda sucesi on de puntos {cn } que converge a c, la sucesi on {f (cn )} converge a f (c).

28

Desigualdades Num ericas

Criterio 1.5.5 Una funci on f : [a, b] R es convexa si y s olo si el conjunto {(x, y ) | a x b, f (x) y } es convexo3 . Demostraci on. Supongamos que f es convexa y, sean A = (x1 , y1 ) y B = (x2 , y2 ) dos puntos del conjunto U = {(x, y ) | a x b, f (x) y }. Para mostrar que tB + (1 t)A = (tx2 + (1 t)x1 , ty2 + (1 t)y1 ) pertenece a U , bastar a ver que a tx2 + (1 t)x1 b y f (tx2 + (1 t)x1 ) ty2 + (1 t)y1 . La primera condici on es inmediata de que tanto x1 como x2 pertenecen a [a, b]. Para la segunda condici on, como f es convexa, sucede que f (tx2 + (1 t)x1 ) tf (x2 ) + (1 t)f (x1 ) adem as, como f (x2 ) y2 y f (x1 ) y1 , se tiene que f (tx2 + (1 t)x1 ) ty2 + (1 t)y1 . Ahora, veamos que f es convexa si U es convexo. Sean x1 , x2 [a, b] y, consideremos A = (x1 , f (x1 )) y B = (x2 , f (x2 )). Claramente, A y B est an en U , y por ser U convexo tambi en est a en U el segmento que los une, es decir, los puntos de la forma tB + (1 t)A para t [0, 1]. Esto es, (tx2 + (1 t)x1 , tf (x2 ) + (1 t)f (x1 )) U , pero esto implica que f (tx2 + (1 t)x1 ) tf (x2 ) + (1 t)f (x1 ), por lo que f es convexa.

Criterio 1.5.6 Una funci on f : [a, b] R es convexa si y s olo si para cada f (x)f (x0 ) es no-decreciente para x0 [a, b] se cumple que la funci on P (x) = xx0 x = x0 . Demostraci on. Supongamos que f es convexa. Para probar que P (x) es nodecreciente tomemos x < y y mostremos que P (x) P (y ). Podemos tener las siguientes tres situaciones: x0 < x < y , x < x0 < y o x < y < x0 .
3 Un subconjunto C del plano es convexo si para cada par de puntos A, B en C se tiene que el segmento de recta que estos determinan est a contenido en C . Como el segmento entre A y B son los puntos de la forma tB + (1 t)A, con 0 t 1, lo que se pide es que este pertenezca a C .

1.5 Funciones convexas

29

Consideremos el primer caso, los otros dos se demuestran de manera semejante. Notemos primero que P (x) P (y ) f (y ) f (x0 ) f (x) f (x0 ) x x0 y x0 (f (x) f (x0 ))(y x0 ) (f (y ) f (x0 ))(x x0 )

f (x)(y x0 ) f (y )(x x0 ) + f (x0 )(y x) yx x x0 + f (x0 ) f (x) f (y ) y x0 y x0 x x0 yx yx x x0 y+ x0 f (y ) + f (x0 ) . f y x0 y x0 y x0 y x0 De aqu , el resultado ya es evidente.

Criterio 1.5.7 Si la funci on f : [a, b] R es derivable4 con derivada nodecreciente, entonces f es convexa. En particular, si f es dos veces derivable y f (x) 0 entonces la funci on es convexa. Demostraci on. Desde luego f (x) 0, para x [a, b], garantiza que f (x) es no-decreciente. Vemos que f (x) no-decreciente implica que la funci on es convexa. Sea x = tb + (1 t)a, un punto de [a, b]. Por el teorema del valor medio5 , existen c (a, x) y d (x, b) tales que f (x) f (a) = (x a)f (c) = t(b a)f (c) y Luego, por ser f (x) no-decreciente, tenemos que (1 t) (f (x)f (a)) = t(1t)(ba)f (c) t(1t)(ba)f (d) = t(f (b)f (x)) que reacomodando los t erminos nos da f (x) tf (b) + (1 t)f (a).
) f ( c ) Una funci on f : [a, b] R es derivable en un punto c [a, b] si l mxc f (xx = f (c) c existe, y es derivable en A [a, b] si lo es en cada punto de A. 5 Teorema del valor medio: Para una funci on continua f : [a, b] R, que tambi en es derivable en (a, b) existe un n umero x (a, b) tal que f (x)(b a) = f (b) f (a). Consultar [21], p ag. 266. 4

f (b) f (x) = (b x)f (d) = (1 t)(b a)f (d).

30

Desigualdades Num ericas

Veamos, ahora, una interpretaci on geom etrica de la convexidad (y la concavidad). Sean x, y , z puntos en el intervalo [a, b] con x < y < z . Si los v ertices del tri angulo XY Z tienen coordenadas X = (x, f (x)), Y = (y, f (y )), Z = (z, f (z )), entonces el area del tri angulo est a dada por 1 = det A, donde 2 1 x f (x) A = 1 y f (y ) . 1 z f (z )

El area puede ser positiva o negativa, dependiendo si el tri angulo XY Z se recorre en sentido positivo (contrario al avance de las manecillas del reloj) o en sentido negativo. Para una funci on convexa tendremos que > 0 y para una funci on c oncava < 0, como muestran las gr acas siguientes.

(y, f (y ))

(x, f (x)) (z, f (z )) (y, f (y )) (x, f (x))

(z, f (z ))

En efecto, > 0 det A > 0 (z y )f (x) (z x)f (y ) + (y x)f (z ) > 0 yx zy f (x) + f (z ). f (y ) < zx zx
x Si tomamos t = y z x , tenemos que 0 < t < 1, 1 t = y f (tz + (1 t)x) < tf (z ) + (1 t)f (x). z y z x ,

y = tz + (1 t)x

Ahora veamos, una serie de ejemplos, donde resaltamos el uso de las funciones convexas para establecer desigualdades. Ejemplo 1.5.8 La funci on f (x) = xn , n 1 es convexa en R+ y la funci on n f (x) = x , con n par, es convexa en R.

1.5 Funciones convexas

31

Lo anterior se sigue de que, f (x) = n(n 1)xn2 0, en las respectivas situaciones. Como aplicaciones de este hecho obtenemos lo siguiente:
a +b b a+b b a+ (i) Ya que a+ 2 2 , tenemos que 2 2 , que es la desigualdad entre la media aritm etica y la media cuadr atica. 2
2 2 2 2

(ii) Como

a+b n 2

1, se cumple

an +bn tenemos que, 2 1 que an + bn n1 . 2

para a y b positivos tales que a + b =

(iii) Tenemos que, para n umeros positivos a y b, 1 + Esto se sigue de que, 2n = f (2) f
a+b a

a n + b

1+

b n a

2n+1 . b a
n

+ 2

a+b b

1 a +f f 1+ 2 b 1 2 1+ a b
n

1+ b a

+ 1+

Ejemplo 1.5.9 La funci on exponencial f (x) = ex es convexa en R, ya que x f (x) = e > 0, para todo x R. Veamos varias aplicaciones de esta propiedad: (i) (Desigualdad M G M A con pesos) Si x1 , ... , xn , t1 , ... , tn son n umeros positivos y n t = 1 , entonces i=1 i
tn 1 xt 1 xn t1 x1 + + tn xn . ti log xi y ex es convexa, tenemos que i En efecto, como xt i =e tn 1 xt 1 xn

= et1 log x1 etn log xn = et1 log x1 ++tn log xn

t1 elog x1 + + tn elog xn = t1 x1 + + tn xn .

1 , para 1 i n, obtenemos otra En particular, si tomamos ti = n demostraci on de la desigualdad entre la media geom etrica y la media aritm etica para n n umeros. (ii) (Desigualdad de Young) Sean x, y n umeros reales positivos. Si a, b > 0 1 1 a 1 b x +1 satisfacen la condici on a + b = 1, entonces xy a by .

Solamente es necesario aplicar la parte (i) como sigue, xy = (xa ) a y b


1 1 b

1 a 1 b x + y . a b

32

Desigualdades Num ericas

(iii) (Desigualdad de H older) Sean x1 , x2 , ... , xn , y1 , y2 , ... , yn , n umeros 1 +1 = 1 , entonces positivos y a, b > 0 tales que a b
n i=1 n 1/a n b yi i=1 1/b

xi y i

xa i
i=1

Supongamos primero que Por la parte (ii), xi yi


n i=1

n n a b i=1 xi = i=1 yi 1 b 1 a a xi + b yi , luego n

= 1.

xi yi

1 a

xa i +
i=1 n a i=1 xi

1 b

n b yi = i=1

1 1 + = 1. a b = B . Tomemos x i =
n b i=1 yi

Supongamos ahora que xi i y = By 1/b . Como A1/a i


n i=1 a x i

=A y
n

n b i=1 yi

n a i=1 xi

=1 y
i=1

yi

= 1,

tenemos que
n n x i yi i=1 n i=1 xi yi

1 Por lo tanto,

=
i=1

1 xi yi = 1/a 1/b 1 /a 1 /b A B A B

xi y i .
i=1

Como un caso particular, cuando a = b = 2, se obtiene la desigualdad de Cauchy-Schwarz. Veamos ahora una consecuencia de la desigualdad de H older, la cual es una generalizaci on de la desigualdad del tri angulo. Ejemplo 1.5.10 (Desigualdad de Minkowski) Sean a1 , a2 , ... , an , b1 , b2 , ... , bn n umeros positivos y sea p > 1, entonces
n
1 p

A1/a B 1/b .

1 p

1 p

(ak + bk )p
k =1

(ak )p
k =1

+
k =1

(bk )p

Observemos que (ak + bk )p = ak (ak + bk )p1 + bk (ak + bk )p1

1.5 Funciones convexas luego,


n n n

33

(ak + bk )p =
k =1 k =1

ak (ak + bk )p1 +
k =1

bk (ak + bk )p1 .

(1.6)

Usemos la desigualdad de H older en cada t ermino de la suma de la derecha en 1 + = 1 , para obtener (1.6), con q tal que 1 p q
n k =1 n k =1 n
1 p

1 q

ak (ak + bk )p1 bk (ak + bk )p1

(ak )p
k =1 n
1 p

(ak + bk )q(p1)
k =1 n
1 q

(bk )p
k =1 k =1

(ak + bk )q(p1)

Sustituyendo estas desigualdades en (1.6), y observando que q (p 1) = p, obtenemos la desigualdad requerida. La desigualdad de Minkowski es una igualdad si permitimos p = 1. Para 0 < p < 1, la desigualdad se invierte. Ejemplo 1.5.11 (Lista corta IMO, 1998) Si r1 , ... , rn son n umeros reales mayores que 1, muestre que 1 n 1 . + + n r r + 1 1 + r1 1 + rn 1 n Notemos primero que la funci on f (x) = f (x) =
e x (1+ex )2 ex (ex 1) (ex +1)3 ri = exi

y f (x) =

Ahora, si ri > 1, entonces convexa, tenemos que 1 e por tanto, n


x ++x ( 1 n n)

para alguna xi > 0. Como f (x) = 1 1 + + x 1+e 1 1 + exn

1 es convexa para R+ , ya que 1 + ex 0, para x > 0.


1 1+ex

es

+1

1 n

1 1 n + + . r1 r n + 1 1 + r1 1 + rn

Ejemplo 1.5.12 (China, 1989) Para cualesquiera n n umeros x1 , ... , xn (0, 1) y con n x = 1 , se cumple que i=1 i n n x i=1 xi i . 1 xi n1 i=1

34 Usaremos el hecho que la funci on f (x) = debe a que f (x) > 0. 1 n


n i=1 x 1x

Desigualdades Num ericas es convexa en (0, 1), esto se

xi 1 = n 1 xi

n i=1

f (xi ) f

i=1

1 xi n

=f

1 n

1 = , n n1

por lo tanto,
n i=1

xi n . 1 xi n1
n i=1 xi

Bastar a ahora ver que de Cauchy-Schwarz,

n i=1

n xi i=1

xi

n, pero esto se sigue de la desigualdad n n. i=1 1 =

Ejemplo 1.5.13 (Hungr a-Israel, 1999) Sean k y l dos enteros positivos dados, y sean aij , 1 i k y 1 j l, kl n umeros positivos dados. Muestre que si q p > 0 entonces
l j =1 k
q p

ap ij
i=1

1 q

k i=1

l j =1

1 p p q aq . ij

p Denamos bj = k i=1 aij , para j = 1, 2, . . . , l, y denotemos la parte izquierda de la desigualdad a probar por L y su parte derecha por R. Entonces

Lq =
j =1 l

q p bj

=
j =1 k

bj p
l

q p

ap ij
i=1
q p p

=
i=1

bj
j =1

. ap ij

1.5 Funciones convexas Usando la desigualdad de H older obtenemos


k

35

Lq

i=1 k

l j =1 l j =1
q

bj p

q p

q q p

q p
q

l j =1

=
i=1

j =1

bj

q p

q p
q

p bj

q p
q

j =1 l j =1

k i=1

p q q aij

p q q p (ap ij )

La desigualdad L R se sigue dividiendo ambos lados de Lq Lqp Rp entre Lqp y tomando la ra z p- esima. Ejercicio 1.77 (i) Para a, b R+ , con a + b = 1, muestre que a+ 1 a
2

p q q aij = Lq p R p .

+ b+

1 b

25 . 2

(ii) Para a, b, c R+ , con a + b + c = 1, muestre que 1 a+ a


2

1 + b+ b

1 + c+ c

100 . 3

Ejercicio 1.78 Para 0 a, b, c 1, muestre que a b c + + + (1 a)(1 b)(1 c) 1. b+c+1 c+a+1 a+b+1

Ejercicio 1.79 (Rusia, 2000) Demuestre que para n umeros reales x, y con 0 x, y 1, se tiene que 1 + 1 + x2 1 1+ y2 2 . 1 + xy

36

Desigualdades Num ericas

Ejercicio 1.80 Muestre que la funci on f (x) = sen x es c oncava en el intervalo [0, ]. Utilice lo anterior para vericar que los a ngulos A , B , C de un tri angulo 3 3. cumplen con sen A + sen B + sen C 2 Ejercicio 1.81 Si A, B , C , D son angulos en el intervalo [0, ], entonces: (i) sen A sen B sen2
A+B 2

y la igualdad se cumple si y s olo si A = B .


A+B +C +D 4

(ii) sen A sen B sen C sen D sen4 (iii) sen A sen B sen C sen3

A+B +C 3

Si adem as A, B , C son los a ngulos internos de un tri angulo, (iv ) sen A sen B sen C 3 8 3.
B C 1 (v ) sen A 2 sen 2 sen 2 8 . B C (vi) sen A + sen B + sen C = 4 cos A 2 cos 2 cos 2 .

Ejercicio 1.82 (Desigualdad de Bernoulli) (i) Para todo n umero real x > 1 y todo entero positivo n, se cumple (1 + x)n 1 + nx. (ii) Use la desigualdad para dar otra demostraci on de la desigualdad M G M A. Ejercicio 1.83 (Desigualdad de Sch ur) Si x, y , z son reales positivos y n es un entero positivo, se cumple xn (x y )(x z ) + y n (y z )(y x) + z n (z x)(z y ) 0. Para el caso n = 1, la desigualdad puede tomar una de las siguientes formas: (a) x3 + y 3 + z 3 + 3xyz xy (x + y ) + yz (y + z ) + zx(z + x). (b) xyz (x + y z )(y + z x)(z + x y ). (c) Si x + y + z = 1, 9xyz + 1 4(xy + yz + zx). Ejercicio 1.84 (Canad a, 1992) Cualesquiera tres n umeros reales no negativos x, y y z cumplen x(x z )2 + y (y z )2 (x z )(y z )(x + y z ).

1.5 Funciones convexas Ejercicio 1.85 Si a, b y c son reales positivos, pruebe que b c 9 a + + . 2 2 2 (b + c) (c + a) (a + b) 4(a + b + c)

37

Ejercicio 1.86 Sean a, b y c n umeros reales positivos, pruebe que 1+ 6 3 . ab + bc + ca a+b+c

M as a un, si abc = 1, pruebe que 1+ 3 6 . a+b+c ab + bc + ca

Ejercicio 1.87 (Desigualdad entre medias potenciales) Sean x1 , x2 , . . . , xn n umeros reales positivos y sean t1 , t2 , . . . , tn n umeros reales positivos tales que su suma sea 1. Sean r y s dos n umeros reales positivos distintos de cero tal que r > s. Muestre que
s s s r r (t1 xr 1 + + tn xn ) (t1 x1 + + tn xn ) ,
1 1

la igualdad se da si y s olo si x1 = x2 = = xn . Ejercicio 1.88 (Dos extensiones de la desigualdad de H older.) Sean x1 , x2 , . . . , xn , y1 , y2 , . . . , yn , z1 , z2 , . . . , zn n umeros reales positivos. 1 1 +1 (i) Si a, b, c son n umeros reales positivos tales que a b = c , entonces
n
1 c

1 a

1 b

(xi yi )c
i=1

xi a
i=1 i=1

yi b

. = 1, entonces

(ii) Si a, b, c son n umeros reales positivos tales que


n i=1 n
1 a

1 1 1 a+b+c n
1 c

1 b

xi yi zi

xi a
i=1 i=1

yi b
i=1

zi c

38

Desigualdades Num ericas

Ejercicio 1.89 (Desigualdad de Popoviciu) Si I es un intervalo y f : I R es una funci on convexa, entonces, para a, b, c I , muestre que 2 f 3 a+b 2 +f b+c 2 +f c+a 2 f (a) + f (b) + f (c) +f 3

a+b+c 3

Ejercicio 1.90 Sean a, b, c n umeros reales no negativos, muestre que: 3 (i) a2 + b2 + c2 + 3 a2 b2 c2 2(ab + bc + ca). (ii) a2 + b2 + c2 + 2abc + 1 2(ab + bc + ca). Ejercicio 1.91 Sean a, b, c n umeros reales positivos, muestre que b+c c+a a+b + + a b c 4 a b c + + b+c c+a a+b .

1.6.

Una desigualdad util

Primero, estudiemos dos identidades algebraicas muy u tiles, que se pueden deducir al considerar un factor especial de a3 + b3 + c3 3abc. Sea P el polinomio c ubico P (x) = x3 (a + b + c)x2 + (ab + bc + ca)x abc. Como a, b, c satisfacen la ecuaci on P (x) = 0, se tiene que a3 (a + b + c)a2 + (ab + bc + ca)a abc = 0, b3 (a + b + c)b2 + (ab + bc + ca)b abc = 0, Sumando estas tres igualdades, obtenemos a3 + b3 + c3 3abc = (a + b + c)(a2 + b2 + c2 ab bc ca). (1.7) c3 (a + b + c)c2 + (ab + bc + ca)c abc = 0.

La identidad anterior implica el siguiente resultado, si a + b + c = 0, entonces a3 + b3 + c3 = 3abc.

1.6 Una desigualdad u til Tambi en tenemos que la expresi on a2 + b2 + c2 ab bc ca puede ser escrita como 1 a2 + b2 + c2 ab bc ca = [(a b)2 + (b c)2 + (c a)2 ]. 2 De esta forma, obtenemos otra versi on de la identidad (1.7), 1 a3 + b3 + c3 3abc = (a + b + c)[(a b)2 + (b c)2 + (c a)2 ]. 2

39

(1.8)

(1.9)

Esta presentaci on de la identidad implica una demostraci on m as corta de la desigualdad M G M A para tres variables. De (1.9), es claro que si a, b, c son n umeros positivos, entonces a3 + b3 + c3 3abc. Ahora, si x, y , z son n umeros 3 3 3 positivos, tomando a = x, b = y y c = z , podemos deducir que x+y+z 3 xyz , 3 con igualdad si y s olo si x = y = z . Observe que se puede utilizar la identidad (1.8) para dar otra demostraci on del ejercicio 1.27. Ejercicio 1.92 Para n umeros reales x, y , z , muestre que x2 + y 2 + z 2 |xy + yz + zx|.

Ejercicio 1.93 Para n umeros positivos reales a, b, c, muestre que a2 + b2 + c2 1 1 1 + + . a b c abc

Ejercicio 1.94 Si x < y < z son n umeros reales, muestre que (x y )3 + (y z )3 + (z x)3 > 0.

40

Desigualdades Num ericas

Ejercicio 1.95 Sean a, b, c las longitudes de los lados de un tri angulo, muestre que 3 3 3 3 a + b + c + 3abc m ax {a, b, c}. 2 Ejercicio 1.96 (Rumania, 2007) Para n umeros reales no-negativos x, y , z , muestre que x3 + y 3 + z 3 3 xyz + |(x y )(y z )(z x)|. 3 4 Ejercicio 1.97 (UK, 2008) Encuentra el m nimo de x2 + y 2 + z 2 , donde x, y , z son n umeros reales que satisfacen que x3 + y 3 + z 3 3xyz = 1. Una desigualdad simple, que puede ser utilizada para demostrar un gran n umero de desigualdades algebraicas, es la siguiente. Teorema 1.6.1 (Una desigualdad u til) Si a, b, x, y son n umeros reales y x, y son positivos, entonces se tiene la siguiente desigualdad (a + b)2 a2 b2 + . x y x+y (1.10)

Demostraci on. La demostraci on es muy simple. Simplicando, podemos expresar la desigualdad como a2 y (x + y ) + b2 x(x + y ) (a + b)2 xy , la cual se reduce a la desigualdad (ay bx)2 0 que es obvia. Vemos que la a b igualdad se tiene si y s olo si ay = bx, esto es, si x =y . Otra forma de justicar este resultado es utilizando la desigualdad de CauchySchwarz de la siguiente manera, (a + b)2 = b a x+ y x y
2

a2 b2 + x y

(x + y ).

Usando el teorema dos veces, podemos extender la desigualdad para tres pares de n umeros, (a + b)2 c2 (a + b + c)2 a2 b2 c2 + + + , x y z x+y z x+y+z

1.6 Una desigualdad u til y un simple argumento inductivo muestra que a2 a2 (a1 + a2 + + an )2 a2 1 + 2 + + n , x1 x2 xn x1 + x2 + + xn

41

(1.11)

para todos los n umeros reales a1 , a2 , ... , an y x1 , x2 , ... , xn > 0, con igualdad si y s olo si a1 a2 an = = = . x1 x2 xn La desigualdad (1.11) es conocida tambi en como la desigualdad de CauchySchwarz en la forma de Engel o el Principio del m nimo de Arthur Engel. Como una primera aplicaci on de esta desigualdad, damos otra demostraci on de la desigualdad de Cauchy-Schwarz,
2 2 a2 1 + a2 + + an = 2 a2 b2 a2 b2 a2 1 b1 + 22 2 + + n2 n , 2 bn b1 b2

entonces
2 2 2 a2 a2 a2 (a1 b1 + a2 b2 + + an bn )2 n bn 2 b2 1 b1 + + + . 2 2 b2 b2 b2 b2 n 1 2 1 + b2 + + b n

Luego, podemos concluir


2 2 2 2 2 2 (a2 1 + a2 + + an )(b1 + b2 + + bn ) (a1 b1 + a2 b2 + + an bn )

y la igualdad se tiene si y s olo si a1 a2 an = = = . b1 b2 bn Existen otras formas de la desigualdad de Cauchy-Schwarz a la Engel, como las del siguiente ejemplo. Ejemplo 1.6.2 Sean a1 , ... , an , b1 , ... , bn n umeros reales positivos. Muestre que: (i) (ii) an (a1 + + an )2 a1 + + . b1 bn a1 b1 + + an bn 1 an a1 + + 2 b a + + an b2 1 n 1

a1 an + + b1 bn

42

Desigualdades Num ericas

Estas dos desigualdades se siguen de la desigualdad u til aplicada de las siguientes maneras: an a2 a2 (a1 + + an )2 a1 + + = 1 + + n . (i) b1 bn a1 b1 an bn a1 b1 + + an bn a1 an 1 b2 b2 (ii) 2 + + 2 = 1 + + n bn a1 an a1 + + an b1
a2 1 a2 n

a1 an + + b1 bn

Ejemplo 1.6.3 (APMO, 1991) Sean a1 , ... , an , b1 , ... , bn , n umeros reales positivos, tal que a1 + a2 + + an = b1 + b2 + + bn . Muestre que a2 1 a2 n 1 + + (a1 + + an ). a1 + b1 an + bn 2 Observemos que (1.11) implica a2 (a1 + a2 + + an )2 a2 n 1 + + a1 + b1 an + bn a1 + a2 + + an + b1 + b2 + + bn = (a1 + a2 + + an )2 2(a1 + a2 + + an )

1 = (a1 + a2 + + an ). 2 El siguiente ejemplo es una demostraci on de la desigualdad entre la media cuadr atica y la media aritm etica. Ejemplo 1.6.4 (Desigualdad media cuadr atica-media aritm etica) Para n umeros reales positivos x1 , ... , xn , se tiene que
2 2 x1 + x2 + + xn x2 1 + x2 + + x n . n n

Observemos que usando (1.11), se tiene que


2 2 x2 (x1 + x2 + + xn )2 1 + x2 + + x n n n2

y esta implica la desigualdad de arriba. En algunos casos los numeradores no son cuadrados, pero un simple truco nos permite escribirlos como cuadrados y entonces podemos aplicar la desigualdad. El siguiente ejemplo es una primera aplicaci on de este truco, la cual nos permite dar una demostraci on m as corta del ejemplo 1.4.9.

1.6 Una desigualdad u til

43

Ejemplo 1.6.5 (IMO, 1995) Sean a, b, c n umeros positivos tal que abc = 1. Demuestre que a3 (b Observemos que 1 1 1 1 1 1 2 2 2 a b c + + = + + a3 (b + c) b3 (c + a) c3 (a + b) a(b + c) b(c + a) c(a + b)
1 2 1 +1 (a ab + bc + ca b + c) = 2(ab + bc + ca) 2abc

1 1 3 1 + 3 + 3 . + c) b (a + c) c (a + b) 2

3 3 (abc)2 3 = , 2 2

donde la primera desigualdad se sigue de (1.11) y la segunda es una consecuencia de la desigualdad M G M A. Otro ejemplo del uso de la desigualdad (1.11) es la siguiente demostraci on de la desigualdad de Nesbitt. Ejemplo 1.6.6 (Desigualdad de Nesbitt) Para a, b, c R+ , se tiene que b c 3 a + + . b+c c+a a+b 2
b c Multipliquemos los tres t erminos de la izquierda de la desigualdad por a a, b, c, respectivamente, y ahora usemos la desigualdad (1.11) para obtener

b2 c2 (a + b + c)2 a2 + + . a(b + c) b(c + a) c(a + b) 2(ab + bc + ca) De la ecuaci on (1.8), obtenemos a2 + b2 + c2 ab bc ca 0 y entonces (a + b + c)2 3(ab + bc + ca). Por lo tanto a b c (a + b + c)2 3 + + . b+c c+a a+b 2(ab + bc + ca) 2 Ejemplo 1.6.7 (Rep ublicas Checa y Eslovaca, 1999) Para n umeros reales positivos a, b y c, muestre la desigualdad b c a + + 1. b + 2c c + 2a a + 2b

44 Observemos que

Desigualdades Num ericas

a b c a2 b2 c2 + + = + + . b + 2c c + 2a a + 2b ab + 2ca bc + 2ab ca + 2bc Entonces usando (1.11) tenemos que b2 c2 (a + b + c)2 a2 + + 1, ab + 2ca bc + 2ab ca + 2bc 3(ab + bc + ca) donde la u ltima desigualdad se sigue como en el ejemplo anterior. Ejercicio 1.98 (Sud africa, 1995) Para n umeros positivos a, b, c, d, muestre que 1 1 4 16 64 + + + . a b c d a+b+c+d Ejercicio 1.99 Sean a y b n umeros reales positivos, muestre que 8(a4 + b4 ) (a + b)4 .

Ejercicio 1.100 Sean x, y , z n umeros reales positivos, muestre que 2 2 9 2 + + . x+y y+z z+x x+y+z

Ejercicio 1.101 Sean a, b, x, y , z n umeros reales positivos, muestre que x y z 3 + + . ay + bz az + bx ax + by a+b

Ejercicio 1.102 Sean a, b, c n umeros reales positivos, muestre que a2 + b2 b2 + c2 c2 + a2 + + a + b + c. a+b b+c c+a

Ejercicio 1.103 (i) Sean x, y , z n umeros reales positivos, muestre que y z 1 x + + . x + 2y + 3z y + 2z + 3x z + 2x + 3y 2

1.6 Una desigualdad u til

45

(ii) (Moldavia, 2007) Sean w, x, y , z n umeros reales positivos, muestre que x y z 2 w + + + + . x + 2y + 3z y + 2z + 3w z + 2w + 3x w + 2x + 3y 3 Ejercicio 1.104 (Croacia, 2004) Sean x, y , z n umeros reales positivos, muestre que y2 z2 3 x2 + + . (x + y )(x + z ) (y + z )(y + x) (z + x)(z + y ) 4 Ejercicio 1.105 Para n umeros reales positivos a, b, c, d, muestre que a b c d + + + 2. b+c c+d d+a a+b Ejercicio 1.106 Sean a, b, c, d, e n umeros reales positivos, muestre que a b c d e 5 + + + + . b+c c+d d+e e+a a+b 2 Ejercicio 1.107 (i) Muestre que, para todos los n umeros reales positivos a, b, c, x, y , z con a b c y z y x, se tiene la siguiente desigualdad (a + b + c)3 a3 b3 c3 + + . x y z 3(x + y + z ) (ii) (Belorusia, 2000) Muestre que, para todos los n umeros reales a, b, c, x, y , z , se sigue la siguiente desigualdad (a + b + c)3 a3 b3 c3 + + . x y z 3(x + y + z ) Ejercicio 1.108 (Grecia, 2008) Para x1 , x2 , . . . , xn n umeros enteros positivos, muestre que
2 2 x2 1 + x2 + + x n x1 + x2 + + xn
kn t

x1 x2 x n ,

donde k = m ax {x1 , x2 , . . . , xn } and t = m n {x1 , x2 , . . . , xn }. Bajo que condiciones es v alida la igualdad?

46

Desigualdades Num ericas

1.7.

La estrategia de sustituci on

Una estrategia u til para resolver problemas de desigualdades es el uso de sustituciones. Es decir, sustituir unas variables por otras de manera que el problema tenga una expresi on m as amable para trabajar. Con una sustituci on conveniente se puede lograr, por ejemplo, que cambien un poco los t erminos dif ciles de manejar, simplicar expresiones o bien reducir t erminos. A un cuando no hay recetas para hacer esto, en esta secci on se dan algunas ideas de lo que puede hacerse, como siempre lo mejor es trabajar con algunos ejemplos. Una primera sugerencia es que para problemas donde hay una condici on extra, hay que intentar utilizarla para simplicar la desigualdad. En el siguiente ejemplo con la condici on extra se eliminan denominadores buscando hacer m as f acil el problema. Ejemplo 1.7.1 Si a, b, c son n umeros reales positivos y menores que 1, con a + b + c = 2, se tiene que a 1a b 1b c 1c 8.

Despu es de hacer la sustituci on x = 1 a, y = 1 b, z = 1 c, tenemos que x + y + z = 3 (a + b + c) = 1, a = 1 x = y + z , b = z + x, c = x + y . Por lo tanto, la desigualdad es ahora equivalente a y+z x y esta, a su vez, es equivalente a (x + y )(y + z )(z + x) 8xyz. Pero esta u ltima desigualdad es inmediata. Basta aplicar tres veces la desigual dad M G M A de la siguiente manera (x + y ) 2 xy (ver el ejercicio 1.26). z+x y x+y z 8,

Puede suceder que la condici on debe usarse simplemente para dirigirse a la soluci on, como en los siguientes dos ejemplos. Ejemplo 1.7.2 (M exico, 2007) Si a, b, c son n umeros reales positivos que satisfacen a + b + c = 1, muestre que a + bc + b + ca + c + ab 2.

1.7 La estrategia de sustituci on. Usando la condici on a + b + c = 1, se tiene que a + bc = a(a + b + c) + bc = (a + b)(a + c), luego, por la desigualdad M G M A, a + bc = (a + b)(a + c) 2a + b + c . 2

47

An alogamente, 2b + c + a 2c + a + b c + ab y . 2 2 Por lo que, al sumar las tres desigualdades, se obtiene b + ca a + bc + b + ca + c + ab 2a + b + c 2b + c + a 2c + a + b 4a + 4b + 4c + + = = 2. 2 2 2 2 La igualdad se alcanza cuando a + b = a + c, b + c = b + a y c + a = c + b, lo que nos lleva a que, a = b = c = 1 3. Ejemplo 1.7.3 Si a, b, c son n umeros reales positivos con ab + bc + ca = 1 muestre que, a b c 3 + + . 2 2 2 2 a +1 b +1 c +1 Notemos que (a2 + 1) = a2 + ab + bc + ca = (a + b)(a + c), y, an alogamente, b2 + 1 = (b + c)(b + a) y c2 + 1 = (c + a)(c + b). Ahora la desigualdad, que estamos considerando, es equivalente a a + (a + b)(a + c) b + (b + c)(b + a) c 3 . 2 (c + a)(c + b)

Por la desigualdad M G M A aplicada a cada sumando del lado izquierdo, tenemos que a b c + + (a + b)(a + c) (b + c)(b + a) (c + a)(c + b) a b c a b c 1 1 1 + + + + + 2 a+b a+c 2 b+c b+a 2 c+a c+b

3 . 2

Los problemas muchas veces sugieren qu e sustituci on debe hacerse. En el siguiente ejemplo la sustituci on nos permite convertir al menos a uno de los t erminos de la desigualdad en una expresi on m as sencilla.

48

Desigualdades Num ericas

Ejemplo 1.7.4 (India, 2002) Si a, b, c son n umeros reales positivos, muestre la siguiente desigualdad c+a a+b b+c a b c + + + + . b c a c+b a+c b+a
b c Al hacer, x = a b , y = c , z = a , el lado izquierdo de la desigualdad es ahora muy sencillo, x + y + z. Veamos como queda el lado derecho. El primer sumando se modica as , b 1+ a 1+ a 1 + xy c+a 1x c bc = = = =x+ . b b c+b 1 + y 1+y 1+ c 1+ c

An alogamente, 1y a+b =y+ a+c 1+z y b+c 1z =z+ . b+a 1+x

Ahora, la desigualdad es equivalente a tener x1 y1 z1 + + 0. 1+y 1+z 1+x Solamente que, ahora, contamos con una condici on extra xyz = 1. La desigualdad anterior la podemos rescribir como (x2 1)(z + 1) + (y 2 1)(x + 1) + (z 2 1)(y + 1) 0 x2 z + y 2 x + z 2 y + x2 + y 2 + z 2 x + y + z + 3. Pero, por la desigualdad M G M A, x2 z + y 2 x + z 2 y 3 3 x3 y 3 z 3 = 3. y +z 1 Tambi en, tenemos x2 + y 2 + z 2 3 (x + y + z )2 = x+3 (x + y + z ) 3 xyz (x + y + z ) = x + y + z , donde la primera desigualdad se obtuvo de aplicar la desigualdad (1.11). Para hacer una sustituci on a veces es necesario preparar un poco el terreno tambi como muestra el siguiente ejemplo. Este en sirve para se nalar que se puede necesitar hacer m as de un proceso de sustituci on. Ejemplo 1.7.5 Sean a, b, c n umeros reales positivos, muestre que se cumple que (a + b)(a + c) 2 abc(a + b + c).

y est a es equivalente a,

1.7 La estrategia de sustituci on. Al dividir entre a2 cada lado de la desigualdad y tomar x = desigualdad se transforma en (1 + x)(1 + y ) 2 xy (1 + x + y ).
b a,

49 y =
c a,

la

1 , s = 1+ 1 Ahora, al dividir entre xy ambos lados y hacer la sustituci on r = 1+ x y, la desigualdad a demostrar ahora es rs 2 rs 1.

Esta u ltima, es equivalente a (rs 2)2 0, que es inmediata. Es una situaci on com un que los problemas tengan varias formas de resolverse, resulta tambi en que pueden aceptar varias sustituciones que ayuden a resolverlo. Esto se muestra en el siguiente ejemplo. Ejemplo 1.7.6 (Corea, 1998) Si a, b, c son n umeros reales positivos, tales que a + b + c = abc, muestre que 1 1 1 3 + + . 2 2 2 2 1+a 1+b 1+c

1 1 , y = 1 on a + b + c = abc, se Bajo la sustituci on x = a b , z = c , la condici traduce en xy + yz + zx = 1 y la desigualdad es equivalente a

x + x2 + 1

y y2 + 1

3 . 2 z2 + 1 z

Pero esta es la tercera desigualdad de esta secci on. Otra posible sustituci on para resolver el ejemplo anterior es considerar la sustituci on a = tan A, b = tan B , c = tan C, como se tiene que tan A + tan B + tan C = tan A tan B tan C entonces A + B + C = (o bien un m ultiplo de ). Pero como 1 + tan2 A = (cos A)2 , la desigualdad es equivalente a 3 que es v alida como se ver a en el ejemplo 2.5.2. cos A + cos B + cos C 2 Note que la desigualdad de Jensen no es posible aplicarla en este caso ya que 1 la funci on f (x) = 1+ no es c oncava en R+ . x2 En el ejemplo anterior vimos que adem as de las sustituciones algebr aicas hay sustituciones trigon ometricas. Esto se muestra tambi en en el siguiente ejemplo. En las secciones 2.2 y 2.5 del siguiente cap tulo trabajaremos con sustituciones geom etricas.

50

Desigualdades Num ericas

Ejemplo 1.7.7 (Rumania, 2002) Si a, b, c son n umeros reales del intervalo (0, 1). Muestre que abc + (1 a)(1 b)(1 c) < 1.
2 Al hacer la sustituci on a = cos cos2 B, c = cos2 C, con A, B, C en el A, b = 2 A = sen A, 1 a = 1 cos 1 b = sen B ) , se tiene que intervalo (0 , 2 y 1 c = senC . Por lo que la desigualdad es equivalente a

cos A cos B cos C + sen A sen B sen C < 1. Para demostrar esta desigualdad bastar a observar que, cos A cos B cos C + sen A sen B sen C < cos A cos B + sen A sen B = cos(A B ) 1. Ejercicio 1.109 Sean x, y , z n umeros reales positivos, muestre que y3 z3 x3 + + 1. x3 + 2y 3 y 3 + 2z 3 z 3 + 2x3

Ejercicio 1.110 (Kazajast an, 2008) Sean x, y , z son n umeros reales positivos que cumplen con xyz = 1, muestre que 1 1 1 3 + + . yz + z zx + x xy + y 2

Ejercicio 1.111 (Rusia, 2004) Si n > 3 y x1 , x2 , ..., xn son n umeros reales positivos, con x1 x2 xn = 1, muestre que 1 1 1 + + + > 1. 1 + x1 + x1 x2 1 + x2 + x2 x3 1 + xn + xn x1

Ejercicio 1.112 (Polonia, 2006) Sean a, b, c n umeros reales positivos que satisfacen ab + bc + ca = abc, muestre que b4 + c4 c4 + a4 a4 + b4 + + 1. ab(a3 + b3 ) bc(b3 + c3 ) ca(c3 + a3 )

1.8 Teorema de Muirhead

51

Ejercicio 1.113 (Irlanda, 2007) Sean a, b, c n umeros reales positivos, muestre que 1 bc ca ca a2 + b2 + c2 a+b+c + + . 3 3 3 a b b Ejercicio 1.114 (Rumania, 2008) Sean a, b, c n umeros reales positivos, con abc = 8, muestre que a2 b2 c2 + + 0. a+1 b+1 c+1

1.8.

Teorema de Muirhead

En 1903, R.F. Muirhead public o un art culo acerca de algunos m etodos algebraicos usados en el estudio de identidades y desigualdades de funciones sim etricas algebraicas de n variables. an 1 a2 consider El o expresiones algebraicas de la forma xa o po1 x2 xn , luego analiz linomios sim etricos que conten an estas expresiones, para dar un ordenen el espacio de n-adas (a1 , a2 , ..., an ) que satisfac an la condici on a1 a2 an . Vamos a suponer que xi > 0 para toda 1 i n. Denotemos por
!

F (x1 , ..., xn )

la suma de los n! t erminos obtenidos de evaluar F (x1 , ..., xn ) en todas las posibles permutaciones de (x1 , ..., xn ). Nosotros consideramos s olo el caso particular
an 1 a2 F (x1 , ..., xn ) = xa 1 x2 xn , con xi > 0, ai 0.

Escribimos [a] = [a1 , a2 , ..., an ] = variables x, y , z > 0 se tiene

1 n!

an 1 a2 xa 1 x2 xn . Por ejemplo, para las

[1, 1] = xy , [1, 1, 1] = xyz , [2, 1, 0] =

1 2 [x (y + z ) + y 2 (x + z ) + z 2 (x + y )]. 3!

Es claro que [a] es invariante bajo cualquier permutaci on de (a1 , a2 , ..., an ) y entonces los dos conjuntos de a son del mismo tipo si ellos s olo dieren por reacomodo. Diremos que un valor de media del tipo [a] es una media sim etrica, (n1)! n 1 por ejemplo, [1, 0, ..., 0] = n! (x1 + x2 + + xn ) = n i=1 xi es la media

52

Desigualdades Num ericas

1 1 1 1 n! 1 1 n n n n x x x , n , ..., n ] = n aritm etica y [ n 1 2 n es la media ! (x1 x2 xn ) = geom etrica. Cuando a1 + a2 + + an = 1, [a] es una generalizaci on com un de la media aritm etica y de la media geom etrica. Si a1 a2 an , b1 b2 bn , en general [b] no es comparable con [a], en el sentido que exista una desigualdad entre ellos, v alida para todos los n umeros reales no negativos x1 , x2 , . . ., xn .

Muirhead deseaba comparar los valores de los polinomios sim etricos [a] y [b] para cualquier valor no negativo de las variables en ambos polinomios. Denamos de ahora en adelante (a) = (a1 , a2 , ..., an ). Denici on 1.8.1 Decimos que (a) mayoriza a (b) y escribimos (b) (a), cuando (a) y (b) se pueden arreglar de tal forma que satisfacen las siguientes dos condiciones:
n n

(1)
i=1

bi =
i=1

ai .

(2)
i=1

bi

i=1

ai , para toda 1 < n.

Es claro que (a) (a) y que si (c) (b) y (b) (a) garantizan (c) (a). Teorema 1.8.2 (Teorema de Muirhead) [b] [a] para cualesquiera valores no negativos de las variables (x1 , x2 , ..., xn ) si y s olo si (b) (a). La igualdad s olo se tiene cuando (b) y (a) son id enticos o cuando todos los xi son iguales. Antes de pasar a la demostraci on del teorema, la cual es un poco complicada, veamos algunos ejemplos. Primero, es claro que [2, 0, 0] no puede ser comparado con [1, 1, 1] ya que la primera condici on en la denici on 1.8.1 no se satisface, pero podemos ver que [2, 0, 0] [1, 1, 0], lo cual es equivalente a x2 + y 2 + z 2 xy + yz + zx. De la misma forma, podemos ver que: 1. x2 + y 2 2xy [2, 0] [1, 1], 2. x3 + y 3 + z 3 3xyz [3, 0, 0] [1, 1, 1], 3. x5 + y 5 x3 y 2 + x2 y 3 [5, 0] [3, 2], 4. x2 y 2 + y 2 z 2 + z 2 x2 x2 yz + y 2 xz + z 2 xy [2, 2, 0] [2, 1, 1],

1.8 Teorema de Muirhead

53

donde todas las desigualdades son verdaderas si aceptamos el teorema de Muirhead. Demostraci on. (Teorema de Muirhead) Supongamos que [b] [a] para todos los n umeros positivos x1 , x2 , . . ., xn . Tomando xi = x, para todo i, obtenemos x
P bi

= [b] [a] = x

ai

Esto s olo puede ser verdadero para todo x si bi = ai . Ahora, sean x1 = x2 = = x = x, x +1 = = xn = 1 para x grande. Como (b) y (a) est an en orden decreciente, los ndices de las potencias m as grandes de x en [b] y [a] son b1 + b2 + + b , a1 + a2 + + a , respectivamente. Luego, es claro que la primera no puede ser mayor que la segunda y esto prueba (2) en la denici on 1.8.1. La demostraci on en la otra direcci on es m as complicada, y para esto necesitaremos una nueva denici on y dos lemas. Deniremos un tipo especial de transformaci on lineal T de a, como sigue. Suponga que ak > al , y escribimos ak = + , al = (0 < ).

Si ahora 0 < , entonces la transformaci on T se dene como T (ak ) = bk = + = T (al ) = bl = = T (a ) = a + ak + al 2 2 + ak + al 2 2

( = k, = l).

Si (b) es la imagen de (a) bajo la transformaci on T , escribimos b = T a. La denici on no necesariamente implica que (a) o (b) est an en orden decreciente. La suciencia de nuestra condici on de comparabilidad ser a establecida, si podemos demostrar los siguientes dos lemas.

Lema 1.8.3 Si b = T a entonces [b] [a], con igualdad s olo cuando todos los xi son iguales.

54

Desigualdades Num ericas

Demostraci on. Podemos reacomodar (a) y (b) de tal forma que k = 1, l = 2. Entonces [a] [b] = [ + , , x3 , ...] [ + , , x3 , ...] 1 + + n xa3 xa x2 + x = n (x1 1 x2 ) ! 3 2n! 1 + + n x2 + x x2 ) xa3 xa n (x1 1 ! 3 2n! 1 + an + 3 = (x1 x2 ) xa x )(x x2 )0 2 1 3 xn (x1 ! 2n! con igualdad s olo cuando todos los xi son iguales. Lema 1.8.4 Si (b) (a), con (b) no id entica a (a), entonces (b) puede ser obtenida de (a) usando sucesivas aplicaciones de un n umero nito de transformaciones T . Demostraci on. Al n umero de diferencias a b distintas de cero, le llamaremos la discrepancia entre (a) y (b). Si la discrepancia es cero, los conjuntos son iguales. Demostremos el lema por inducci on, suponiendo que es verdadero cuando la discrepancia es menor que r y demostrando que es verdadero cuando la discrepancia es r . Supongamos entonces, que (b) (a) y que la discrepancia es r > 0. Como n n (a b ) = 0, y no todas las diferencias son cero, i=1 ai = i=1 bi , y deben existir diferencias positivas y negativas, y la primera que nos es cero debe ser positiva por la segunda condici on de (b) (a). Entonces, podemos encontrar k y l tal que bk < ak , bk+1 = ak+1 , ... , bl1 = al1 , bl > al , (1.12)

esto es, al bl es la primera diferencia negativa y ak bk es la u ltima diferencia positiva que la precede. Sea ak = + , al = , y dena por = m ax {|bk | , |bl |}. Entonces 0 < , ya que ak > al . Tambi en, uno o el otro de bl = y bk = , es verdadero, ya que bk bl , y < , adem as bk < ak y bl > al . Luego, 0 < . Ahora escribimos a k = + , al = , a = x ( = k , = l). Si bk = , a k = bk y si bl = entonces al = bl . Como cada una de las parejas ak ,

1.8 Teorema de Muirhead

55

bk y al , bl contribuye con una unidad a la discrepancia r entre (b) y (a), la discrepancia entre (b) y (a ) es menor, la cual es r 1 o r 2. Enseguida, comparemos la denici on de (a ) con la denici on de T y, observando que 0 < , podemos deducir que (a ) proviene de (a) por medio de una transformaci on T . Finalmente, (b) (a ). Para demostrar esto, debemos vericar que las dos condiciones de son satisfechas y que el orden de (a ) es no creciente. Para la primera condici on, tenemos
n n n

a k

a l

= 2 = ak + al ,
i=1

bi =
i=1

ai =
i=1

a i.

Para la segunda condici on, necesitamos demostrar que


b1 + b2 + + b a 1 + a2 + + a .

Ahora, esto es verdadero si < k o l, como se puede ver de la denici on de (a ) y usando la segunda condici on de (b) (a). Esto es cierto para = k, ya que es cierto para = k 1 y bk a k , y es verdadero para k < < l, ya que es v alido para = k y los elementos que aparecen en b y a son id enticos. Finalmente, podemos observar que bk + |bk | + = a k, bl |bl | = a l y entonces, usando (1.12),
a k 1 = ak 1 ak = + > + = ak bk bk +1 = ak +1 = ak +1 , a l1 = al1 = bl1 bl al = > = al al+1 = al+1 .

As las desigualdades que afectan a a son las buscadas. Por lo tanto, hemos demostrado que (b) (a ), un conjunto que proviene de (a) usando una transformaci on T que tiene una discrepancia con (b) menor a r . Esto completa la demostraci on del lema y del teorema de Muirhead. La demostraci on del teorema de Muirhead nos muestra, por medio de un uso repetido de la transformaci on T , como las diferencias entre dos medias comparables puede ser descompuesta como una suma de t erminos que son obviamente positivos. Podemos deducir, de este u ltimo resultado, una nueva demostraci on de la desigualdad M G M A.

56 Ejemplo 1.8.5 (Desigualdad M G M A)

Desigualdades Num ericas

y1 + y2 + + yn n . y1 y2 yn n Notemos que la desigualdad M G M A es equivalente a x1 x2 xn donde xi = n yi . Ahora, observemos que 1 n


n i=1 n n xn 1 + x2 + + x n n

xn i = [n, 0, 0, ..., 0] y x1 x2 xn = [1, 1, ..., 1].

Por el teorema de Muirhead, podemos deducir que [1, 1, ..., 1] [n, 0, 0, ..., 0]. Enseguida, daremos otra demostraci on de la desigualdad M G M A, siguiendo las ideas de la demostraci on del teorema de Muirhead, para ilustrar como trabaja. 1 n
n i=1

xn i (x1 x2 xn ) = [n, 0, 0, ..., 0] [1, 1, ..., 1] = ([n, 0, 0, ..., 0] [n 1, 1, 0, ..., 0]) + +([n 1, 1, 0, ..., 0] [n 2, 1, 1, 0, ..., 0]) +

+... + ([2, 1, 1, ..., 1] [1, 1, ..,1]) 1 n1 = (xn1 x2 )(x1 x2 )+ ! 1 2n! n2 n2 + (x1 x2 )(x1 x2 )x3 +
!

+([n 2, 1, 1, 0, ..., 0] [n 3, 1, 1, 1, 0, ..., 0]) +

n3 n3 (x1 x2 )(x1 x2 )x3 x4 + ... .

Como (x r xs )(xr xs ) > 0, a menos que xr = xs , la desigualdad se sigue.

Ejemplo 1.8.6 Si a, b son n umeros reales positivos, entonces a2 + b b2 a + b. a

1.8 Teorema de Muirhead Sea x = a, y = b, simplicando vemos que se tiene que demostrar x3 + y 3 xy (x + y ). Por el teorema de Muirhead, tenemos que 1 1 [3, 0] = (x3 + y 3 ) xy (x + y ) = [2, 1], 2 2 luego, el resultado se sigue. Ejemplo 1.8.7 Si a, b, c son n umeros reales no negativos, muestre que a3 + b3 + c3 + abc No es dif cil ver que (a + b + c)3 = 3[3, 0, 0] + 18[2, 1, 0] + 36[1, 1, 1]. Luego, demostraremos que 1 3[3, 0, 0] + 6[1, 1, 1] (3[3, 0, 0] + 18[2, 1, 0] + 36[1, 1, 1]), 7 esto es, 18 36 [3, 0, 0] + 6 7 7 [1, 1, 1] 18 [2, 1, 0] 7 [1, 1, 1] 0. 1 (a + b + c)3 . 7

57

18 36 ([3, 0, 0] [2, 1, 0]) + 6 7 7

Esto se sigue usando las desigualdades [3, 0, 0] [2, 1, 0] y [1, 1, 1] 0. Ejemplo 1.8.8 Si a, b, c son n umeros reales no negativos, muestre que a+b+c a3 b3 c3 a2 + b2 b2 + c2 c2 + a2 + + + + . 2c 2a 2b bc ca ab

Las desigualdades son equivalentes a 2(a2 bc + ab2 c + abc2 ) ab(a2 + b2 )+ bc(b2 + c2 )+ ca(c2 + a2 ) 2(a4 + b4 + c4 ), que a su vez es equivalente a [2, 1, 1] [3, 1, 0] [4, 0, 0]. Usando el teorema de Muirhead obtenemos el resultado.

58

Desigualdades Num ericas

Ejercicio 1.115 Cualesquiera tres n umeros reales a, b y c, satisfacen que a5 + b5 + c5 a3 bc + b3 ca + c3 ab.

Ejercicio 1.116 (IMO, 1961) Sean a, b, c las longitudes de los lados de un tri angulo y sea (ABC ) su area. Muestre que 4 3(ABC ) a2 + b2 + c2 .

Ejercicio 1.117 Sean a, b, c n umeros reales positivos, muestre que a b c 9 + + . (a + b)(a + c) (b + c)(b + a) (c + a)(c + b) 4(a + b + c)

Ejercicio 1.118 (IMO, 1964) Sean a, b, c n umeros reales positivos. Muestre que a3 + b3 + c3 + 3abc ab(a + b) + bc(b + c) + ca(c + a). Ejercicio 1.119 (Lista corta Iberoamericana, 2003) Sean a, b, c n umeros reales positivos. Muestre que b3 c3 a3 + + a + b + c. b2 bc + c2 c2 ca + a2 a2 ab + b2 Ejercicio 1.120 (Lista corta IMO, 1998) Sea a, b, c n umeros reales positivos tales que abc = 1. Muestre que b3 c3 3 a3 + + . (1 + b)(1 + c) (1 + c)(1 + a) (1 + a)(1 + b) 4

Cap tulo 2

Desigualdades Geom etricas

2.1.

Dos desigualdades b asicas

Las dos desigualdades b asicas en geometr a se reeren a los tri angulos. Una de ellas es la desigualdad del tri angulo y es la que m as adelante vamos a referir como D1, la segunda no es propiamente una desigualdad, es una observaci on importante de la geometr a de los tri angulos, donde se se nala que el lado mayor del tri angulo es el opuesto al angulo mayor y la que denotamos por D2. D1. Si A, B y C son puntos del plano, entonces AB + BC AC. Adem as, la igualdad se alcanza si y s olo si B est a en el segmento AC . D2. En un tri angulo el lado mayor es el que se opone al angulo mayor y rec procamente. As , si en el tri angulo ABC se tiene que A > B , entonces BC > CA. Ejercicio 2.1 (i) Si a, b, c son n umeros positivos con a < b + c, b < c + a y c < a + b, entonces se puede encontrar un tri angulo con lados de longitud a, b y c. (ii) Para poder formar un tri angulo con lados de longitud a b c, basta que c < a + b. (iii) Se puede construir un tri angulo con segmentos de longitud a, b, c si y s olo si existen n umeros positivos x, y , z con a = x + y , b = y + z y c = z + x.

60

Desigualdades Geom etricas

Ejercicio 2.2 (i) Si con segmentos de longitud a < b < c se puede construir un tri angulo, entonces con segmentos de longitud a < b < c se puede armar un tri angulo. (ii) El rec proco de (i) es falso. (iii) Si con segmentos de longitud a < b < c se puede armar un tri angulo, 1 1 1 entonces con segmentos de longitud a+b , b+c y c+a se puede formar un tri angulo. Ejercicio 2.3 Considere los segmentos de longitud a, b, c, d y e, de manera que con cualesquiera tres de ellos se puede formar un tri angulo, muestre que hay tres de ellos que forman un tri angulo acut angulo. Algunas veces la clave para resolver un problema est a en identicar ciertas cantidades con magnitudes geom etricas, como sucede en el siguiente ejemplo. Ejemplo 2.1.1 Si a, b, c son n umeros positivos con a2 + b2 ab = c2 , muestre que (a b)(b c) 0. Como c2 = a2 + b2 ab = a2 + b2 2ab cos 60 , podemos pensar que a, b, c son las longitudes de los lados de un tri angulo tal que la medida del angulo opuesto al lado de longitud c, es igual a 60 . Los angulos del tri angulo ABC cumplen, A 60 y B 60 , o bien A 60 y B 60 . Por lo tanto, por la propiedad D2 tenemos que a c b o bien a c b. En cualquier caso sucede que (a b)(b c) 0. Observaci on 2.1.2 Tambi en se puede resolver el ejemplo anterior sin tener que identicar a, b y c con las longitudes de los lados de un tri angulo. Supongamos primero que a b, el que a2 + b2 ab = c2 implica que a(a b) = c2 b2 = (c b)(c + b), por lo que c b 0 y entonces (a b)(b c) 0. An alogamente, a b implica c b 0, lo que nos lleva a que (a b)(b c) 0. Otro ejemplo donde no es inmediato que se trate de una desigualdad geom etrica, o que el uso de la geometr a ayude, se muestra a continuaci on. Ejemplo 2.1.3 Si a, b, c son n umeros positivos, entonces a2 + ac + c2 a2 ab + b2 + b2 bc + c2 .

2.1 Dos desigualdades b asicas

61

Los radicales sugieren usar la ley de los cosenos con angulos de 120 y de 60 2 2 2 2 como sigue: a + ac + c = a + c 2ac cos 120 , a2 ab + b2 = a2 + b2 2ab cos 60 y b2 bc + c2 = b2 + c2 2bc cos 60 . A a
60 60

D c C

b B

Entonces, si consideramos un cuadril atero ABCD, con ADB = BDC = , de manera que AD = a, BD = b y CD = c, tene60 y ADC = 120 mos que AB = a2 ab + b2 , BC = b2 bc + c2 y CA = a2 + ac + c2 . La desigualdad que debemos mostrar es la desigualdad del tri angulo para el tri angulo ABC . Ejercicio 2.4 Sea ABC un tri angulo con A > B , muestre que BC > 1 2 AB . Ejercicio 2.5 Sea ABCD un cuadril atero convexo, muestre que: (i) Si AB + BD < AC + CD entonces AB < AC .
1 AD. (ii) Si A > C y D > B entonces BC > 2

Ejercicio 2.6 Si a1 , a2 , a3 , a4 , a5 son las longitudes de los lados de un pent agono convexo y si d1 , d2 , d3 , d4 , d5 son las longitudes de sus diagonales, muestre que a1 + a2 + a3 + a4 + a5 1 < < 1. 2 d1 + d2 + d3 + d4 + d5 angulo ABC cumple Ejercicio 2.7 La longitud ma de la mediana AA de un tri a que ma > b+c . 2 Ejercicio 2.8 Si la longitud de la mediana AA de un tri angulo ABC cumple o. a , muestre que BAC < 90 ma > 1 2 Ejercicio 2.9 Si AA es la mediana del tri angulo ABC y si AB < AC , entonces BAA > A AC .

62

Desigualdades Geom etricas

Ejercicio 2.10 Si ma , mb y mc son las longitudes de las medianas de un tri angulo con lados de longitudes a, b y c, respectivamente. Muestre que se puede armar un tri angulo de lados con longitudes ma , mb y mc , y que 3 (a + b + c) < ma + mb + mc < a + b + c. 4 Ejercicio 2.11 (Desigualdad de Ptolomeo) Para un cuadril atero convexo ABCD se cumple que AC BD AB CD + BC DA. La igualdad se da si y s olo si ABCD es un cuadril atero c clico. Ejercicio 2.12 Sea ABCD un cuadril atero c clico. Muestre que AC > BD si y s olo si (AD BC )(AB DC ) > 0. Ejercicio 2.13 (Problema de Pompeiu) Sea ABC un tri angulo equil atero y P un punto del plano que no pertenece al circunc rculo de ABC. Muestre que P A, P B y P C son las longitudes de los lados de un tri angulo. Ejercicio 2.14 Si ABCD es un paralelogramo, muestre que AB 2 BC 2 < AC BD. Ejercicio 2.15 Si a, b y c son las longitudes de los lados de un tri angulo, ma , mb y mc representan las longitudes de las medianas y R el circunradio. Muestre que: (i) a2 + b2 b2 + c2 c2 + a2 12R. + + mc ma mb

(ii) ma (bc a2 ) + mb (ca b2 ) + mc (ab c2 ) 0. Ejercicio 2.16 Sea ABC un tri angulo cuyos lados tienen longitudes a, b y c. Supongamos que c > b, muestre que 3 1 (c b) < mb mc < (c b), 2 2 donde mb y mc son las longitudes de las medianas. Ejercicio 2.17 (Ir an, 2005) Sea ABC un tri angulo con A = 90 . Sea D la intersecci on de la bisectriz interna del A con el lado BC y sea Ia el centro de la circunferencia excrita al tri angulo ABC opuesta al v ertice A. Muestre que AD 2 1. DIa

2.2 Desigualdades entre los lados de un tri angulo

63

2.2.

Desigualdades entre los lados de un tri angulo

Las desigualdades entre las longitudes de los lados de un tri angulo aparecen frecuentemente como problemas en los concursos. Un tipo de problemas son los que piden demostrar una desigualdad que satisfacen las longitudes de los lados del tri angulo y no hay m as elementos geom etricos involucrados, como es el siguiente. Ejemplo 2.2.1 Las longitudes de los lados a, b y c de un tri angulo cumplen que a (b + c a) < 2bc. Como la desigualdad es sim etrica en b y c podemos suponer, sin perder generalidad, que c b. Estudiaremos la desigualdad en casos: Caso 1. a b. Por ser la longitud de los lados de un tri angulo tenemos que b < a + c; luego b + c a = b a + c < c + c = 2c 2bc . a

Caso 2. a b. Aqu resulta que b a 0, y como a < b + c 2b, tenemos que b+ca=c+bac < 2bc . a

Otro tipo de problemas para las longitudes de los lados de un tri angulo, son aquellos donde se pide demostrar que cierta relaci on entre los n umeros a, b y c es suciente para construir un tri angulo con lados de longitud a, b y c. Ejemplo 2.2.2 (i) Si a, b, c son n umeros positivos y satisfacen, a2 + b2 + c2 > 4 4 4 2 a + b + c , entonces a, b y c son las longitudes de los lados de un tri angulo. (ii) Si a, b, c, d son n umeros positivos y satisfacen a2 + b2 + c2 + d2
2 2

> 3 a4 + b4 + c4 + d4 ,

entonces con cualesquiera tres de ellos se puede formar un tri angulo. Para la parte (i) basta observar que, a2 + b2 + c2 2 a4 + b4 + c4 = (a+b+c)(a+bc)(ab+c)(a+b+c) > 0 y notar que de estos factores ninguno es negativo, compare con el ejemplo 1.2.5.
2

64 Para la parte (ii), tenemos que 3 a4 + b4 + c4 + d4 < a2 + b2 + c2 + d2 =


2

Desigualdades Geom etricas

a2 + b2 + c2 a2 + b2 + c2 + + d2 2 2 a2 + b2 + c2 2
2

a2 + b2 + c2 2

+ d4

La segunda desigualdad se sigue de la desigualdad de Cauchy-Schwarz; luego (a2 +b2 +c2 ) 2 a4 + b4 + c4 < 2 . Por la primera parte tenemos que con a, b y c se 4 puede formar un tri angulo. Como el argumento utilizado es sim etrico en a, b, c y d, tenemos el resultado. Existe una t ecnica que ayuda a transformar una desigualdad entre las longitudes de los lados de un tri angulo en una desigualdad entre n umeros positivos (desde luego relacionados con los lados), llamada la transformaci on de Ravi. Si el inc rculo (I, r ) del tri angulo ABC es tangente a los lados BC , CA y AB en los puntos X , Y y Z , respectivamente, tenemos que, x = AZ = Y A, y = ZB = BX , z = XC = CY . A x Z y B y I z x Y

C z X Es claro que a = y + z , b = z + x, c = x + y , x = s a, y = s b y z = s c, b+c donde s = a+2 . Veamos como usar la transformaci on de Ravi en el siguiente ejemplo. Ejemplo 2.2.3 Las longitudes de los lados a, b y c de un tri angulo cumplen que (b + c a)(c + a b)(a + b c) abc. Primero tenemos que (b + c a)(c + a b)(a + b c) = 8(s a)(s b)(s c) = 8xyz,

2.2 Desigualdades entre los lados de un tri angulo por otro lado abc = (x + y )(y + z )(z + x). Luego, la desigualdad es equivalente a 8xyz (x + y )(y + z )(z + x), que sabemos que es v alida por el ejercicio 1.26.

65

(2.1)

Ejemplo 2.2.4 (APMO, de los lados de un a, b, c las longitudes 1996) Sean tri angulo, muestre que a + b c + b + c a + c + a b a + b + c. Como a = y + z , b = z + x, c = x + y , podemos deducir a + b c = 2z , b + c a = 2x, c + a b = 2y . Luego, la desigualdad es equivalente a 2x + 2y + 2z x + y + y + z + z + x. Ahora usando la desigualdad entre la media aritm etica y la media cuadr atica (vea ejercicio 1.68), obtenemos 2y + 2z 2x + 2y 2z + 2x 2x + 2y + 2z = + + 2 2 2 2x + 2y 2y + 2z 2z + 2x + + 2 2 2 = x + y + y + z + z + x. Adem as, la igualdad se tiene si y s olo si x = y = z , es decir, si y s olo si a = b = c. Tambi en, es posible expresar el area de un tri angulo ABC , su inradio, su circunradio y su semiper metro en t erminos de x, y , z . Como a = x + y , b = y + z b+c = x + y + z . Usando la f ormula y c = z + x, se obtiene primero que s = a+2 de Her on para el area de un tri angulo se obtiene (ABC ) = s(s a)(s b)(s c) = (x + y + z )xyz = x+y+z se tiene (x + y )(y + z )(z + x) 4 (x + y + z )xyz . (x + y + z )xyz. (2.2)

Usando la f ormula (ABC ) = sr , vemos que r= (ABC ) = s


abc 4R

xyz . x+y+z

Finalmente, de (ABC ) =

R=

66

Desigualdades Geom etricas

Ejemplo 2.2.5 (India, 2003) Sean a, b, c las longitudes de los lados de un b , tri angulo ABC . Si construimos un tri angulo A B C con lados de longitud a + 2 a 9 c b + 2 , c + 2 , muestre que (A B C ) 4 (ABC ). Como a = y + z , b = z + x y c = x + y , se tiene que las longitudes de los lados y +3z y +2z y +z del tri angulo A B C son a = x+22 , b = 3x+2 , c = 2x+3 . Usando 2 la f ormula de Her on para el area de un tri angulo se obtiene (A B C ) = 3(x + y + z )(2x + y )(2y + z )(2z + x) . 16

Aplicando la desigualdad entre la M G M A podemos establecer que 2x + y 3 a a tener la 3 3 x2 y , 2y + z 3 3 y 2 z y 2z + x 3 z 2 x, lo que nos ayudar siguiente desigualdad (A B C ) 9 3(x + y + z )27(xyz ) = (ABC ), 16 4

donde la u ltima igualdad es consecuencia directa de la ecuaci on (2.2). Ejercicio 2.18 Sean a, b y c las longitudes de los lados de un tri angulo, muestre que 3(ab + bc + ca) (a + b + c)2 4(ab + bc + ca). Ejercicio 2.19 Sean a, b y c las longitudes de los lados de un tri angulo, muestre que ab + bc + ca a2 + b2 + c2 2(ab + bc + ca). Ejercicio 2.20 Sean a, b y c las longitudes de los lados de un tri angulo, muestre que 2 a2 + b2 + c2 (a + b + c)2 . Ejercicio 2.21 Sean a, b y c las longitudes de los lados de un tri angulo, muestre que a b c 3 + + < 2. 2 b+c c+a a+b Ejercicio 2.22 (IMO, 1964) Sean a, b y c las longitudes de los lados de un tri angulo, muestre que a2 (b + c a) + b2 (c + a b) + c2 (a + b c) 3abc.

2.2 Desigualdades entre los lados de un tri angulo

67

Ejercicio 2.23 Sean a, b y c las longitudes de los lados de un tri angulo, muestre que a b2 + c2 a2 + b c2 + a2 b2 + c a2 + b2 c2 3abc. Ejercicio 2.24 (IMO, 1983) Sean a, b y c las longitudes de los lados de un tri angulo, muestre que a2 b(a b) + b2 c(b c) + c2 a(c a) 0. Ejercicio 2.25 Sean a, b y c las longitudes de los lados de un tri angulo, muestre que 1 ab bc ca + + < . a+b b+c c+a 8 Ejercicio 2.26 Las longitudes de los lados de un tri angulo a, b y c, satisfacen ab + bc + ca = 3, muestre que 3 a + b + c 2 3. Ejercicio 2.27 Sean a, b, c las longitudes de los lados de un tri angulo y r el inradio del tri angulo, muestre que 3 1 1 1 + + . a b c 2r Ejercicio 2.28 Sean a, b, c las longitudes de los lados de un tri angulo y s el semiper metro, muestre que: (i) (s a)(s b) < ab, ab + bc + ca . 4

(ii) (s a)(s b) + (s b)(s c) + (s c)(s a)

Ejercicio 2.29 Si a, b, c son las longitudes de los lados de un tri angulo acut angulo, muestre que a2 + b2 c2 a2 b2 + c2 a2 + b2 + c2 ,

c clica

68

Desigualdades Geom etricas

donde c clicas, (a, b, c), (b, c, a) clica signica la suma sobre las permutaciones c y (c, a, b), de (a, b, c). Ejercicio 2.30 Si a, b, c son las longitudes de los lados de un tri angulo acut angulo, muestre que a2 + b2 c2 a2 b2 + c2 ab + bc + ca,

c clica

donde c clicas, (a, b, c), (b, c, a) clica signica la suma sobre las permutaciones c y (c, a, b), de (a, b, c).

2.3.

Uso de desigualdades en la geometr a del tri angulo

Un problema que ayuda a ejemplicar el uso de las desigualdades en la geometr a es el problema 2 que se plante o en la Olimpiada Internacional de Matem aticas en 1961. De este existen varias pruebas y tambi en sus aplicaciones son diversas como veremos m as adelante. Por ahora lo presentamos como un ejemplo. Ejemplo 2.3.1 Si a, b y de los lados de un tri angulo de c son las longitudes 2 2 2 area (ABC ), entonces 4 3(ABC ) a + b + c . Como un tri angulo equil atero con lado de longitud a tiene area igual a 43 a2 , la igualdad se alcanza en tal caso; trataremos entonces de comparar lo que sucede en un tri angulo en general con lo que pasa, al respecto, en un tri angulo equil atero con lado de longitud a. A c b

h d D e

Sea BC = a. Si AD es la altura de un tri angulo desde A, su longitud h la 3 podemos escribir como h = 2 a + y , donde y mide su defecto con respecto a la longitud de la altura del tri angulo equil atero. Tambi en escribimos d = a 2 x a y e = 2 + x, donde x se puede interpretar como el defecto o la diferencia, que

2.3 Uso de desigualdades en la geometr a del tri angulo

69

la proyecci on de A en BC tiene, con respecto a la proyecci on de A en BC en el tri angulo equil atero, que en tal caso es el punto medio del lado BC . Tenemos que

a +x a2 + b2 + c2 4 3(ABC ) = a2 + h2 + 2

+ h2 +

a x 2

ah 4 3 2

3 2 3 2 2 = a + 2h + 2x 2 3 a a+y 2 2 2 3 3 2 = a +2 a + y + 2x2 3a2 2 3ay 2 2 3 3 = a2 + a2 + 2 3ay + 2y 2 + 2x2 3a2 2 3ay 2 2 = 2(x2 + y 2 ) 0.

Adem as, la igualdad se da si y s olo si x = y = 0, es decir, cuando el tri angulo es equil atero. Veamos otra demostraci on del ejemplo anterior. Sea ABC el tri angulo con lados de longitud a b c, y sea A un punto de manera que A BC sea un tri angulo equil atero con lados de longitud a. Llamemos d = AA , entonces d mide de alguna manera el defecto que tiene ABC con respecto al tri angulo equil atero. d A

A c B

70 Por la ley de los cosenos, tenemos que d2 = a2 + c2 2ac cos(B 60 )

Desigualdades Geom etricas

= a2 + c2 2ac(cos B cos 60 + sen B sen 60 ) ac sen B = a2 + c2 ac cos B 2 3 2 2 + c2 b2 a = a2 + c2 ac 2 3(ABC ) 2ac 2 2 2 a +b +c = 2 3(ABC ). 2 Pero d2 0, luego tenemos que 4 3(ABC ) a2 + b2 + c2 , como dese abamos. Adem as, la igualdad se da si d = 0, esto es si A = A o, equivalentemente, si ABC es equil atero. Es com un encontrar entre los problemas de olimpiadas desigualdades que involucran elementos de un tri angulo. Algunas de ellas son consecuencia de la siguiente desigualdad, v alida para n umeros positivos a, b, c, (ver ejercicio 1.36 de la secci on 1.3). 1 1 1 + + 9. (2.3) (a + b + c) a b c Adem as, recordemos que se da la igualdad si y s olo si a = b = c. Otra desigualdad que ha resultado muy u til para problemas de geometr a, es la desigualdad de Nesbitt, (ver ejemplo 1.4.8 de la secci on 1.4). Esta dice que para n umeros positivos a, b, c, siempre se tiene b c 3 a + + . b+c c+a a+b 2 (2.4)

La desigualdad anterior se puede demostrar, usando la desigualdad (2.3) de la siguiente forma a b c a+b+c a+b+c a+b+c + + = + + 3 b+c c+a a+b b+c c+a a+b 1 1 1 + + 3 = (a + b + c) b+c c+a a+b 1 1 1 1 = [(a + b) + (b + c) + (c + a)] + + 3 2 b+c c+a a+b 9 3 3= . 2 2

2.3 Uso de desigualdades en la geometr a del tri angulo

71

La igualdad se da si y s olo si a + b = b + c = c + a, o equivalentemente, si a = b = c. Veamos ahora una serie de ejemplos de desigualdades geom etricas donde tales relaciones se utilizan. Ejemplo 2.3.2 Sea ABC un tri angulo equil atero con lados de longitud a; sea M un punto dentro de ABC y D, E , F las proyecciones de M sobre los lados BC , CA y AB , respectivamente. Muestre que: 1 1 6 3 1 + + , (i) MD ME MF a 1 1 1 3 3 (ii) + + . MD + ME ME + MF MF + MD a A F M E

Sean x = M D , y = M E y z = M F . Si denotamos por (ABC ) al area del tri angulo ABC , tenemos que (ABC ) = (BCM ) + (CAM ) + (ABM ), por lo que ah = ax + ay + az , donde h = 23 a es la longitud de la altura de ABC . Por lo tanto, h = x + y + z (este resultado es conocido como el lema de Viviani, ver la secci on 2.8). Por la desigualdad (2.3) tenemos que h 1 1 1 + + x y z 9, despejando, 9 6 3 1 1 1 + + = . x y z h a

Para demostrar la segunda parte, observemos que de la desigualdad (2.3), tenemos que (x + y + y + z + z + x) Por lo tanto,
1 x +y

1 1 1 + + x+y y+z z+x =


3 3 a .

9.

1 y +z

1 z +x

9 2h

72

Desigualdades Geom etricas

Ejemplo 2.3.3 Si ha , hb y hc son las longitudes de las alturas de un tri angulo ABC que tiene inc rculo de centro I y radio r , tenemos: (i) r r r + + = 1, ha hb hc

(ii) ha + hb + hc 9r. Para demostrar la primera ecuaci on, observemos que manera an aloga,
r hb r ha

(ICA) r (ABC ) , hc

(IAB ) (ABC ) ,

sumando

r a ha a

(IBC ) (ABC ) .

De

r r r + + ha hb hc

= =

(ICA) (IAB ) (IBC ) + + (ABC ) (ABC ) (ABC ) (IBC ) + (ICA) + (IAB ) = 1. (ABC ) A

ha

I r

La desigualdad es consecuencia directa de la desigualdad (2.3), ya que (ha + hb + hc ) 1 1 1 + + ha hb hc r 9r.

Ejemplo 2.3.4 Sea ABC un tri angulo con alturas AD, BE , CF y sea H el ortocentro. Muestre que: (i) (ii) BE CF AD + + 9, HD HE HF HF 3 HD HE + + . HA HB HC 2

2.3 Uso de desigualdades en la geometr a del tri angulo A F H E

73

Para demostrar el inciso (i), consideremos S = (ABC ), S1 = (HBC ), S2 = (HCA), S3 = (HAB ). Como los tri angulos ABC y HBC tienen la misma HD 1 base, la raz on de sus areas es igual a la raz on de sus alturas, es decir, S S = AD . S3 HE HF HD HE HF 2 An alogamente, S S = BE y S = CF . Luego, AD + BE + CF = 1. Usando la desigualdad (2.3), tenemos BE CF AD + + HD HE HF HF HD HE + + AD BE CF

9.

Al sustituir aqu la igualdad que calculamos obtenemos (i).


HE HF 1 M as a un, la igualdad se da si y s olo si HD AD = BE = CF = 3 , o lo que es lo 1 mismo, S1 = S2 = S3 = 3 S . Para demostrar la segunda parte observemos que, S1 S1 S2 S3 HD HF HD alogamente, HE HA = AD HD = S S1 = S2 +S3 y an HB = S3 +S1 , HC = S1 +S2 , HE HF 3 entonces por la desigualdad de Nesbitt tenemos que HD HA + HB + HC 2 .

Ejemplo 2.3.5 (Corea, 1995) Sean ABC un tri angulo y L, M , N puntos sobre BC , CA y AB , respectivamente. Sean P , Q y R los puntos de intersecci on de las l neas AL, BM y CN con el circunc rculo de ABC , respectivamente. Muestre que CN AL BM + + 9. LP MQ NR

Sean A el punto medio de BC , P el punto medio del arco BC , D y D las proyecciones de A y P sobre BC , respectivamente.

74

Desigualdades Geom etricas

A D P L

D P

AL AD AL AD CN Es claro que, LP nimo de LP =P + BM D P A . Luego, el valor m M Q + N R se obtiene cuando P , Q y R son los puntos medios de los arcos BC , CA y AB , respectivamente. Esto suceder a cuando AL, BM y CN sean las bisectrices internas del tri angulo ABC . Por lo que, sin perder generalidad, supondremos que AL, BM y CN son las bisectrices internas de ABC. Como AL es bisectriz interna, se tiene que1

BL = Adem as,

ba ca , LC = b+c b+c

AL2 = bc 1

a b+c

AL = = = LP AL LP BL LC BM (c + a)2 b2 = MQ b2 Por lo tanto, CN AL BM + + LP MQ NR =


1

AL2

AL2

(bc) 1

a b+c

a2 bc (b+c)2

(b + c)2 a2 . a2

An alogamente, para las bisectrices BM y CN , se tiene y (a + b)2 c2 CN = . NR c2


2

b+c a

c+a b

a+b c
2

1 b+c c+a a+b + + 3 a b c 1 (6)2 3 = 9. 3

Ver teorema de la bisectriz y ejercicio 2.8.17 del cap tulo 2 en [6], p ag.74 y 105 o [9], p ag.10 y 11.

2.3 Uso de desigualdades en la geometr a del tri angulo

75

La primera desigualdad se sigue de la convexidad de la funci on f (x) = x2 , y la c b segunda desigualdad de relaciones de la forma c + b 2. Observemos que la igualdad se da si y s olo si a = b = c. Otra manera de terminar el problema es como sigue, b+c a = a2 b2 + b2 a2 +
2

c+a b +

a+b c +2

3= ab bc ca + 2+ 2 c2 a b 3

b2 c2 + c2 b2

c2 a2 + a2 c2

2 3 + 2 3 3 = 9. Aqu usamos el hecho que


a2 b2 b +a 2 2 y que
2

ab c2

3 bc ca +a 2 + b2 3

(ab)(bc)(ca) a2 b2 c2

= 3.

Ejemplo 2.3.6 (Lista corta IMO, 1997) Las longitudes de los lados de un hex agono ABCDEF satisfacen AB = BC , CD = DE y EF = F A. Muestre que DE FA 3 BC + + . BE DA F C 2 B

A c F a b C

D E Sean a = AC , b = CE y c = EA. La desigualdad de Ptolomeo (ver ejercicio 2.11), aplicada en el cuadril atero ACEF , nos garantiza que AE F C F A CE + AC EF . Como EF = F A, tenemos entonces c F C F A b + F A a. Por lo tanto, FA c . FC a+b An alogamente, se puede ver que a BC BE b+c y DE b . DA c+a

DE FA a b c 3 Por tanto, BC ltima desigualdad es BE + DA + F C b+c + c+a + a+b 2 . La u la desigualdad de Nesbitt, que sabemos es cierta.

76

Desigualdades Geom etricas

Ejercicio 2.31 Sean a, b, c las longitudes de los lados de un tri angulo, muestre que: (i) (ii) b c a + + 3, b+ca c+ab a+bc

b+ca c+ab a+bc + + 3. a b c

Ejercicio 2.32 Sean AD, BE , CF las alturas de un tri angulo ABC y P Q, P R, P S las distancias de un punto P a los lados BC , CA, AB , respectivamente. Muestre que AD BE CF + + 9. PQ PR PS Ejercicio 2.33 Tres l neas se trazan por un punto O que se localiza dentro de un tri angulo de area S de manera que cada lado es cortado por dos de estas. Las l neas dividen al tri angulo en tres tri angulos con v ertice com un O y areas S1 , S2 y S3 , adem as de tres cuadril ateros. Muestre que: (i) (ii) 1 1 9 1 + + , S1 S2 S3 S 1 1 1 18 + + . S1 S2 S3 S

Ejercicio 2.34 Tres cevianas AL, BM , CN de un tri angulo ABC son concuBP CP + + olo si P es el rrentes en un punto P . Muestre que, AP PL PM P N = 6 si y s centroide. Ejercicio 2.35 Las alturas AD, BE , CF intersectan al circunc rculo del tri angulo ABC en D , E y F , respectivamente. Muestre que: (i) (ii) BE CF AD + + 9, DD EE FF BE CE 9 AD + + . AD BE CF 4

Ejercicio 2.36 Sea ABC un tri angulo y sean la , lb , lc las longitudes de las bisectrices internas, s el semiper metro y r el inradio de tal tri angulo. Muestre que:

2.4 La desigualdad de Euler y algunas aplicaciones (i) la lb lc rs2 .

77

2 + l 2 + l 2 s2 . (iii) la c b

(ii) la lb + lb lc + lc la s2 .

Ejercicio 2.37 Sea ABC un tri angulo y sean M , N , P puntos arbitrarios en los segmentos BC , CA, BA, respectivamente. Denote las longitudes de los lados del tri angulo por a, b, c y el circunradio por R. Muestre que ca ab bc + + 6R. AM BN CP

Ejercicio 2.38 Sea ABC un tri angulo con lados de longitud a, b y c. Sean ma , mb y mc las longitudes de las medianas desde A, B y C , respectivamente. Muestre que m ax {a ma , b mb , c mc } sR, donde R es el radio del circunc rculo y s el semiper metro.

2.4.

La desigualdad de Euler y algunas aplicaciones

Teorema 2.4.1 (Teorema de Euler) Si en un tri angulo ABC , O es su circuncentro, I su incentro, R su circunradio y r su inradio, entonces OI 2 = R2 2Rr. Demostraci on. Veamos una demostraci on2 que solamente depende del teorema de Pit agoras y del hecho de que el circunc rculo del tri angulo BCI tiene centro D, donde D es el punto medio del arco BC 3 . En la demostraci on usaremos segmentos dirigidos.
2 3

Otra demostraci on puede verse en [6], p ag. 122 o [9], p ag. 29. La demostraci on puede verse en [6], observaci on 3.2.7, p ag. 123 o [1], p ag. 76.

78

Desigualdades Geom etricas

Q O

I C

Sean M el punto medio de BC y Q la proyecci on ortogonal de I sobre el radio OD. Entonces OB 2 OI 2 = OB 2 DB 2 + DI 2 OI 2 = OM 2 M D 2 + DQ2 QO2

= DO(M O + M D + DQ + OQ) = R(2M Q) = 2Rr. Por lo tanto, OI 2 = R2 2Rr .

= (M O + DM ) (M O DM ) + (DQ + QO)(DQ QO)

Como una consecuencia del teorema anterior tenemos la siguiente desigualdad, Teorema 2.4.2 (Desigualdad de Euler) R 2r . Adem as, R = 2r si y s olo si el tri angulo es equil atero4 . Teorema 2.4.3 En un tri angulo ABC , con circunradio R, inradio r y semiper metro s, se tiene R s r . 2 3 3
Existen demostraciones directas de la desigualdad (esto es, sin utilizar la f ormula de Euler). Una de ellas es la siguiente: el c rculo de los nueve puntos de un tri angulo es el circunc rculo del tri angulo medial A B C , como este tri angulo es semejante a ABC en raz on 2:1, se tiene que el radio del c rculo de los nueve puntos es R . Claramente una circunferencia que corta a 2 los tres lados del tri angulo debe tener radio mayor que el radio del inc rculo, luego R r. 2
4

2.4 La desigualdad de Euler y algunas aplicaciones

79

la desigualdad M G Demostraci on. Usaremos que5 , (ABC ) = abc 4R = sr . Por M A, tenemos que 2s = a + b + c 3 3 abc = 3 3 4 Rrs. Luego, 8s3 27(4Rrs) 27(8r 2 s), ya que R 2r . Por lo tanto, s 3 3r . R s 3R. Pero , es equivalente a a + b + c 3 La segunda desigualdad, 3 2 3 esta, utilizando la ley de los senos, es equivalente a sen A + sen B + sen C 3 3 ltima desigualdad es verdadera, ya que la funci on 2 . Observemos que esta u A+B +C B +sen C sen = f (x) = sen x es c oncava en [0, ], entonces sen A+sen 3 3 sen 60 =
3 2 .

Ejercicio 2.39 Sean a, b y c las longitudes de los lados de un tri angulo, muestre que (a + b c)(b + c a)(c + a b) abc. Ejercicio 2.40 Sean a, b y c las longitudes de los lados de un tri angulo, muestre que 1 1 1 1 + + 2, ab bc ca R donde R denota el circunradio. Ejercicio 2.41 Sean A, B y C las medidas de los angulos en cada uno de los v ertices del tri angulo ABC , muestre que 1 1 1 + + 4. sen A sen B sen B sen C sen C sen A Ejercicio 2.42 Sean A, B y C las medidas de los angulos en cada uno de los v ertices del tri angulo ABC , muestre que sen A 2 sen B 2 sen C 2 1 . 8

Ejercicio 2.43 Sea ABC un tri angulo. Llamamos a los angulos en los v ertices en A, B , C , de la misma forma, es decir, A, B , C , respectivamente. Sean a, b y c las longitudes de los lados del tri angulo y sea R el radio del circunc rculo. Muestre que 2A
1 a

2B

1 b

2C

1 c

2 3

3 R

Consultar [6], p ag. 97 o [9], p ag. 13.

80

Desigualdades Geom etricas

Teorema 2.4.4 (Teorema de Leibniz) En un tri angulo ABC con lados de longitud a, b y c, y con circuncentro O, centroide G y circunradio R, se cumple que OG2 = R2 1 2 a + b2 + c2 . 9

Demostraci on. Usaremos el teorema de Stewart que dice6 que, si L es un punto sobre el lado BC de un tri angulo ABC y si AL = l, BL = m, LC = n, entonces a l2 + mn = b2 m + c2 n. A

G B A

O R C

Aplicando el teorema de Stewart al tri angulo OAA para encontrar la longitud de OG, donde A es el punto medio de BC , obtenemos AA OG2 + AG GA = A O2 AG + AO2 GA . Como 2 AO = R, AG = AA 3 sustituyendo tenemos 2 1 2 OG2 + (A A)2 = A O2 + R2 . 9 3 3
6

1 GA = AA , 3

Para la demostraci on v ease [6], p ag. 96 o [9], p ag. 6.

2.4 La desigualdad de Euler y algunas aplicaciones Por otro lado7 , (A A)2 = OG2 =
2(b2 +c2 )a2 4

81 tenemos que

y A O2 = R2

a2 4 ,

R2

a2 4

2 1 2 2 + R 3 3 9

2 b2 + c2 a2 4

a2 2 b2 + c2 a2 6 18 2 2 2 a +b +c = R2 . 9 = R2 Una consecuencia del teorema anterior es la desigualdad siguiente. Teorema 2.4.5 (Desigualdad de Leibniz) En un tri angulo ABC , con lados de longitud a, b y c, con circunradio R, se cumple que 9R2 a2 + b2 + c2 . Adem as, la igualdad se da si y s olo si O = G, es decir, cuando el tri angulo es equil atero. Ejemplo 2.4.6 En un tri angulo ABC con lados de longitud a, b y c, y de area (ABC ), se tiene que 9abc . 4 3(ABC ) a+b+c Usando que 4R(ABC ) = abc, tenemos las siguientes equivalencias 9R2 a2 +b2 +c2 a2 b2 c2 3abc a2 + b2 + c2 4(ABC ) . 2 2 16(ABC ) 9 a + b2 + c2 La desigualdad de Cauchy-Schwarz nos garantiza que a+b+c 3 a2 + b2 + c2 , por lo que 9abc 4 3(ABC ) . a+b+c

Ejercicio 2.44 Sean A, B y C las medidas de los angulos en cada uno de los v ertices del tri angulo ABC , muestre que sen2 A + sen2 B + sen2 C
7

9 . 4

Consultar [6], p ag. 83 o [9], p ag. 10.

82

Desigualdades Geom etricas

Ejercicio 2.45 Sean a, b y c las longitudes de los lados de un tri angulo, muestre que 3 4 3(ABC ) 3 a2 b2 c2 .

Ejercicio 2.46 Suponga que el inc rculo de ABC es tangente a los lados BC , CA, AB , en D, E , F , respectivamente. Muestre que EF 2 + F D 2 + DE 2 donde s es el semiper metro de ABC . Ejercicio 2.47 Sean a, b, c las longitudes de los lados de un tri angulo ABC y sean ha , hb , hc las longitudes de las alturas sobre A, B , C , respectivamente. Muestre que b2 c2 a2 + + 4. hb hc hc ha ha hb s2 , 3

2.5.

Funciones sim etricas de a, b y c

Las longitudes de los lados a, b y c de un tri angulo tienen una relaci on muy estrecha con s, r y R el semiper metro, el inradio y el circunradio del tri angulo, respectivamente. Las relaciones m as usadas son: a + b + c = 2s ab + bc + ca = s + r + 4rR abc = 4Rrs.
2 2

(2.5) (2.6) (2.7)

La primera es la denici on de s y la tercera se sigue de que el area de un abc ormula de Her on para el area de un tri angulo tri angulo es 4R = rs. Usando la f tenemos la relaci on s(s a)(s b)(s c) = r 2 s2 , por lo que s3 (a + b + c)s2 + (ab + bc + ca)s abc = r 2 s. Sustituyendo las ecuaciones (2.5) y (2.7) en esta igualdad, y despejando obtenemos que ab + bc + ca = s2 + r 2 + 4Rr.

2.5 Funciones sim etricas de a, b y c

83

Ahora bien, como cualquier polinomio sim etrico en a, b y c se puede expresar como un polinomio en t erminos de (a + b + c), (ab + bc + ca) y (abc), tambi en lo podemos expresar como un polinomio en s, r y R. Por ejemplo, a2 + b2 + c2 = (a + b + c)2 2(ab + bc + ca) = 2 s2 r 2 4Rr , = 2 s3 3r 2 s 6Rrs .

a3 + b3 + c3 = (a + b + c)3 3(a + b + c)(ab + bc + ca) + 3abc

Estas transformaciones ayudan a resolver diversos problemas como veremos m as adelante. Lema 2.5.1 Si A, B y C son las medidas de los angulos en cada uno de los r + 1. v ertices del tri angulo ABC , se tiene que cos A + cos B + cos C = R Demostraci on. cos A + cos B + cos C = = = = = = b2 + c2 a2 c2 + a2 b2 a2 + b2 c2 + + 2bc 2ca 2ab 2 2 2 2 2 a b + c + b c + a + c a + b2 a3 + b3 + c3 2abc 2 2 2 (a + b + c) (a + b + c ) 2(a3 + b3 + c3 ) 2abc 2 2 4s s r 4Rr 4 s3 3r 2 s 6Rrs 8Rrs 2 2 s r 4Rr (s2 3r 2 6Rr ) 2Rr 2 2r + 2Rr r = + 1. 2Rr R

Ejemplo 2.5.2 Sean A, B y C los angulos en cada uno de los v ertices del . tri angulo ABC , muestre que cos A + cos B + cos C 3 2 El lema 2.5.1 nos garantiza que cos A + cos B + cos C = desigualdad de Euler, R 2r , tenemos el resultado. Podemos dar otra demostraci on directa. Observemos que,
r R

+ 1, y usando la

a(b2 +c2 a2 )+b(c2 +a2 b2 )+c(a2 +b2 c2 ) = (b+ca)(c+ab)(a+bc)+2abc.

84 Luego, cos A + cos B + cos C = =

Desigualdades Geom etricas

b2 + c2 a2 c2 + a2 b2 a2 + b2 c2 + + 2bc 2ca 2ab (b + c a)(c + a b)(a + b c) + 1, 2abc

y como (b + c a)(c + a b)(a + b c) abc, se tiene el resultado. Ejemplo 2.5.3 (IMO, 1991) Sean ABC un tri angulo, I su incentro y L, M , N las intersecciones de las bisectrices en A, B , C con BC , CA, AB , respectiAI BI CI 8 vamente. Muestre que 1 4 < AL BM CN 27 . A M I C L AB c El teorema de la bisectriz nos asegura que BL LC = CA = b y, como BL + LC = a, ac ab se tiene que BL = b+c y LC = b+c . El mismo teorema de la bisectriz aplicado a IL BL ac a la bisectriz BI del angulo ABL, garantiza que AI = AB = (b+ c)c = b+c . Por lo que, AI + IL IL a a+b+c AL = =1+ =1+ = . AI AI AI b+c b+c
AI b+c 8 BI c +a CI Luego, AL = a+ alogamente, BM = a+ b+c . An b+c y CN = desigualdad que se pide demostrar en t erminos de a, b y c, es a+b a+b+c .

As , la

(b + c)(c + a)(a + b) 8 1 < . 3 4 (a + b + c) 27 La desigualdad M G AM nos garantiza que (b + c)(c + a)(a + b) (b + c) + (c + a) + (a + b) 3


3

8 (a + b + c)3 , 27

por lo que la desigualdad de la derecha que debemos demostrar es ahora evidente.


Otra forma de ver la identidad es como sigue: Considere = (ABI ), = (BCI ) y r (c+b) + AI c+ b = (CAI ), es claro que, AL = + = r( = a+ . + a+c+b) c+ b
8

2.5 Funciones sim etricas de a, b y c Para demostrar la desigualdad de la izquierda, notemos primero que (a + b + c)(ab + bc + ca) abc (b + c)(c + a)(a + b) = . 3 (a + b + c) (a + b + c)3 Sustituyendo las ecuaciones (2.5), (2.6) y (2.7) tenemos (b + c)(c + a)(a + b) (a + b + c)3 = = 2s(s2 + r 2 + 4Rr ) 4Rrs 8s3 1 2r 2 + 4Rr 1 2s3 + 2sr 2 + 4Rrs + = > . 3 2 8s 4 8s 4

85

Podemos usar tambi en la transformaci on de Ravi con a = y + z , b = z + x, c = x + y , para concluir de la siguiente manera (b + c)(c + a)(a + b) (x + y + z + x)(x + y + z + y )(x + y + z + z ) = 3 (a + b + c) 8(x + y + z )3 = = 1 8 1 8 1+ 1+ x x+y+z 1+ y x+y+z 1+ z x+y+z > 1 . 4

xyz x + y + z xy + yz + zx + + x+y+z x+y+z x+y+z

Ejercicio 2.48 Sean A, B y C las medidas de los angulos en cada uno de los v ertices del tri angulo ABC , muestre que sen2 B C 3 A + sen2 + sen2 . 2 2 2 4

Ejercicio 2.49 Sean a, b y c las longitudes de los lados de un tri angulo y (ABC ) el area del tri angulo. Con la herramienta estudiada en esta secci on muestre que 4 3(ABC ) 9abc . a+b+c

Ejercicio 2.50 Sean a, b y c las longitudes de los lados de un tri angulo y (ABC ) el area del tri angulo, con la herramienta estudiada en esta secci on, muestre que 3 4 3(ABC ) 3 a2 b2 c2 .

86

Desigualdades Geom etricas

Ejercicio 2.51 (IMO, 1961) Sean a, b y c las longitudes de los lados de un tri angulo y (ABC ) el area del tri angulo, muestre que 4 3(ABC ) a2 + b2 + c2 . Ejercicio 2.52 Sean a, b y c las longitudes de los lados de un tri angulo y (ABC ) el area del tri angulo, muestre que 4 3(ABC ) a2 + b2 + c2 (a b)2 (b c)2 (c a)2 . Ejercicio 2.53 Sean a, b y c las longitudes de los lados de un tri angulo y (ABC ) el area del tri angulo, muestre que 4 3(ABC ) ab + bc + ca. Ejercicio 2.54 Sean a, b y c las longitudes de los lados de un tri angulo y (ABC ) el area del tri angulo, muestre que 3(a + b + c)abc 4 3(ABC ) . ab + bc + ca Ejercicio 2.55 Sean a, b y c las longitudes de los lados de un tri angulo. Si a + b + c = 1, muestre que 1 a2 + b2 + c2 + 4abc < . 2 Ejercicio 2.56 Sean a, b y c las longitudes de los lados de un tri angulo, R el circunradio y r el inradio, muestre que 2r (b + c a)(c + a b)(a + b c) = . abc R Ejercicio 2.57 Sean a, b y c las longitudes de los lados de un tri angulo y R el circunradio, muestre que 3 3R a2 b2 c2 + + . b+ca c+ab a+bc

Ejercicio 2.58 Sean a, b y c las longitudes de los lados de un tri angulo y b b+ca (ABC ) el area del tri angulo. Tomemos como siempre x = 2 , y = c+a 2 a+bc y z = 2 . Si 1 = x + y + z , 2 = xy + yz + zx y 3 = xyz , verique las siguientes relaciones:

2.6 Desigualdades con areas y per metros

87

2 3 ). (1) (a b)2 + (b c)2 + (c a)2 = (x y )2 + (y z )2 + (z x)2 = 2(1 2

(2) a + b + c = 21 .
2 2 . (3) a2 + b2 + c2 = 21 2 2+ . (4) ab + bc + ca = 1 2

(5) abc = 1 2 3 . (6) 16(ABC )2 = 2(a2 b2 + b2 c2 + c2 a2 ) (a4 + b4 + c4 ) = 16r 2 s2 = 161 3 . (7) R = 1 2 3 . 4 1 3 3 . 1

(8) r =

(9) 1 = s, 2 = r (4R + r ), 3 = r 2 s.

2.6.

Desigualdades con areas y per metros

Empezamos esta secci on con los siguientes ejemplos. Ejemplo 2.6.1 (Austria-Polonia, 1985) Si ABCD es un cuadril atero convexo de area 1, entonces AB + BC + CD + DA + AC + BD 4 + 8.

Finalmente, como (e + f )2 = 4ef + (e f )2 4ef 8 y (a + b + c + d)2 = 4(a + c)(b + d)+((a + c) (b + d))2 4(a + c )(b + d) = 4(ab + bc + cd + da) 16, tenemos que, a + b + c + d + e + f 4 + 8.

B cd sen D Como (ABC ) = ab sen ab 2 2 y (CDA) = 2 ab+cd 1 = (ABCD) 2 . An alogamente, 1 = (ABCD) desigualdades nos garantizan que ab + bc + cd + da 4.

Sean a = AB , b = BC , c = CD, d = DA, e = AC y f = BD. El area ef sen ngulo entre las del cuadril atero ABCD es (ABCD) = 2 , donde es el a ef ef sen diagonales, luego es claro que, 1 = 2 2 .
cd 2 , tenemos bc+da 2 . Estas

que dos

88

Desigualdades Geom etricas

Ejemplo 2.6.2 (Iberoamericana, 1992) A partir de un tri angulo ABC , se construye un hex agono H de v ertices A1 , A2 , B1 , B2 , C1 , C2 como se muestra en la gura. Demuestre que el area del hex agono H es mayor o igual que trece veces el area del tri angulo ABC . A2 a c B1 b B b B2 Es claro, usando la f ormula de area (ABC ) =
ab sen C , 2

A1 a A b a C c c C1 C2

que

(A1 A2 B1 B2 C1 C2 ) =(A1 BC2 ) + (A2 CB1 ) + (B2 AC1 ) + (AA1 A2 )+ + (BB1 B2 ) + (CC1 C2 ) 2(ABC ) = (b + c)2 sen A (c + a)2 sen B (a + b)2 sen C + + + 2 2 2 +

a2 sen A b2 sen B c2 sen C + + 2(ABC ) 2 2 2 (a2 + b2 + c2 )(sen A + sen B + sen C ) + ca sen B + = 2 + ab sen C + b c sen A 2(ABC ) = (a2 + b2 + c2 )(sen A + sen B + sen C ) + 4(ABC ). 2

Por lo tanto, (A1 A2 B1 B2 C1 C2 ) 13(ABC ) si y s olo si 9abc (a2 + b2 + c2 )(sen A + sen B + sen C ) 9(ABC ) = . 2 4R Usando la ley de los senos, y s olo si
(a2 +b2 +c2 )(a+b+c) 4R 1 sen A a = 2R , tenemos abc 94 R , o lo que es lo

que la desigualdad es cierta si mismo

(a2 + b2 + c2 )(a + b + c) 9abc.

2.6 Desigualdades con areas y per metros

89

La u ltima desigualdad se deduce ya sea de la desigualdad M G M A, de la desigualdad del reacomodo o bien de la desigualdad de Tchebychev. Adem as, se da la igualdad solamente en el caso en que a = b = c. Ejemplo 2.6.3 (China, 1988 y 1993) Se tienen dos circunferencias conc entricas de radios R y R1 (R1 > R) y un cuadril atero convexo ABCD inscrito en la circunferencia peque na. Las extensiones de AB , BC , CD y DA intersectan a la circunferencia grande en C1 , D1 , A1 y B1 , respectivamente. Muestre que: (i) R1 per metro de A1 B1 C1 D1 . per metro de ABCD R (A1 B1 C1 D1 ) (ABCD) R1 R
2

(ii)

, donde (ABCD) denota el area del cuadril atero.


A1
A D O B C

B1

C1

D1

Para demostrar (i), utilizamos la desigualdad de Ptolomeo (ver ejercicio 2.11) aplicada a los cuadril ateros OAB1 C1 , OBC1 D1 , OCD1 A1 y ODA1 B1 , esta nos asegura que AC1 R1 B1 C1 R + AB1 R1

BD1 R1 C1 D1 R + BC1 R1

(2.8)

DB1 R1 A1 B1 R + DA1 R1 . Luego, al sumar estas desigualdades y descomponer AC1 , BD1 , CA1 y DB1 como AB + BC1 , BC + CD1 , CD + DA1 y DA + AB1 , respectivamente, tenemos R1 per metro (ABCD) + R1 (BC1 + CD1 + DA1 + AB1 )

CA1 R1 D1 A1 R + CD1 R1

R per metro (A1 B1 C1 D1 ) + R1 (AB1 + BC1 + CD1 + DA1 ).

90 Por lo tanto,

Desigualdades Geom etricas

R1 per metro (A1 B1 C1 D1 ) . per metro (ABCD) R


bc sen A = Para demostrar (ii), usaremos el hecho que (ABCD) = ad sen A+ 2 sen A ab sen B +cd sen B sen B en que (ABCD) = = 2 (ad + bc) y tambi 2 2 (ab + cd), donde A = DAB y B = ABC .

B1

A1
x A d a O B y b z C c D w

C1
(180 A) Como (AB1 C1 ) = x(a+y)sen = 2 (AB1 C1 ) x(a+y ) alogamente, dad (ABCD) = ad+bc . An (DA1 B1 ) (ABCD )

D1

x(a+y )sen A , podemos deducir la igual2 (BC1 D1 ) y (b+z ) (CD1 A1 ) z (c+w ) (ABCD ) = ab+cd , (ABCD ) = ad+bc ,

w (d+x) ab+cd .

Luego,

x(a + y ) + z (w + c) y (b + z ) + w(d + x) (A1 B1 C1 D1 ) =1+ + . (ABCD) ad + bc ab + cd La potencia de un punto de la circunferencia grande con respecto a la circunfe2 R2 . En particular, las potencias de A , B , C rencia peque na es igual a R1 1 1 1 y D1 son la misma. Por otro lado, tenemos que estas potencias son w(w + c), x(x + d), y (y + a) y z (z + b), respectivamente. Sustituyendo en la ecuaci on anterior, tenemos que la raz on de las areas es (A1 B1 C1 D1 ) = (ABCD)
2 = 1 + (R1 R2 )

x z y w + + + , y (ad + bc) w(ad + bc) z (ab + cd) x(ab + cd)

La desigualdad M G M A, nos permite deducir que (A1 B1 C1 D1 ) 1+ (ABCD) (ad + bc)(ab + cd)
2 R2 ) 4(R1

2.6 Desigualdades con areas y per metros

91

2R) + b + c + d)2 (4 4 = 8R2 , las dos primeras desigualdades se dan por la desigualdad M G M A, y la u ltima se debe a que el cuadrado es de los cuadril ateros inscritos en una circunferencia, el de mayor per metro. As 1 4 (a 2 R2 ) 4(R1 (A1 B1 C1 D1 ) 1+ = (ABCD) 4R2

Ahora bien, como 2 (ad + bc)(ab + cd) ad + bc + ab + cd = (a + c)(b + d)


2

R1 R

Desde luego, las igualdades en i) y ii) ocurren cuando ABCD es un cuadrado y u nicamente en este caso. Ya que para reducir la desigualdad (2.8) a una igualdad, se debe tener que los cuatro cuadril ateros OAB1 C1 , OBC1 D1 , OCD1 A1 y ODA1 B1 sean c clicos. Luego, OA es bisectriz del angulo BAD, lo mismo sucede con OB , OC y OD. Existen problemas que a pesar de no ser planteados de forma geom etrica nos invitan a buscar relaciones geom etricas, como en el siguiente ejemplo. Ejemplo 2.6.4 Si a, b, c son umeros positivos con c < a y c < b, se tiene que n c(a c) + c(b c) ab. Consideremos los tri angulos is osceles ABC y ACD, los cuales comparten el lado AC de longitud 2 c; tomamos el primero con lados iguales, AB = BC , de longitud igual a a y el segundo que satisfaga que CD = DA y de longitud igual a b. D
b b c c

A
a

E a

B El area del papalote ABCD es por un lado (ABCD) = (ABC ) + (ACD) = y por otro, (ABCD) = 2(ABD) = c(a c) + b(b c)

2 ab sen BAD . 2

92

Desigualdades Geom etricas ab,

Esta u ltima forma de calcular el area muestra claramente que (ABCD) de donde se sigue el resultado buscado.

Podemos dar otra soluci on del ejemplo anterior. Como AC y BD son perpendi culares, por el teorema de Pit agoras, tenemos DE = b c y EB = a c. Por la desigualdad de Ptolomeo (ver ejercicio 2.11) ( b c + a c)(2 c) a b + a b, de donde el resultado se sigue. Ejercicio 2.59 En cada lado de un cuadrado con lados de longitud uno, se escoge un punto. Los cuatro puntos forman un cuadril atero con lados de longitud a, b, c y d, muestre que: (i) 2 a2 + b2 + c2 + d2 4, (ii) 2 2 a + b + c + d 4. Ejercicio 2.60 En cada lado de un hex agono regular de lados de longitud 1 se escoge un punto. Los seis puntos forman un hex agono de per metro h. Muestre que 3 3 h 6. Ejercicio 2.61 Considere las tangentes al inc rculo, de un tri angulo ABC , que son paralelas a los lados del tri angulo. Estas determinan junto con los lados del tri angulo un hex agono T . Muestre que per metro de T 2 per metro de (ABC ). 3

Ejercicio 2.62 Encontrar el radio del c rculo de area m axima que puede cubrirse con tres c rculos de radio 1. Ejercicio 2.63 Encontrar el radio del c rculo de area m axima que puede cubrirse con tres c rculos de radios r1 , r2 y r3 . Ejercicio 2.64 Dentro de un cuadrado de lado 1, coloque dos cuadrados ajenos. Si las longitudes de los lados de los dos cuadrados son a y b, respectivamente, muestre que a + b 1. Ejercicio 2.65 Un cuadril atero convexo est a inscrito en una circunferencia de radio 1, de manera que uno de sus lados es un di ametro y los otros son de longitudes a, b, c. Muestre que abc 1.

2.7 Teorema de Erd os-Mordell

93

Ejercicio 2.66 Sea ABCDE un pent agono convexo de area (ABCDE ) y tal que las areas de los tri angulos ABC , BCD, CDE , DEA y EAB son iguales, muestre que: (i) (ABCDE ) (ABCDE ) < (ABC ) < , 4 3 5+ 5 (ii) (ABCDE ) = (ABC ). 2

Ejercicio 2.67 Si AD, BE y CF son las alturas del tri angulo ABC , muestre que per metro (DEF ) s, donde s es el semiper metro de ABC . Ejercicio 2.68 Las longitudes de las bisectrices internas de un tri angulo son menores o iguales a 1, muestre que el area de dicho tri angulo es menor o igual a 33 . Ejercicio 2.69 Si a, b, c, d son las longitudes de los lados de un cuadril atero convexo de area (ABCD), muestre que: ab + cd (i) (ABCD) , 2 ac + bd , (ii) (ABCD) 2 a+c b+d (iii) (ABCD) . 2 2

2.7.

Teorema de Erd os-Mordell

Teorema 2.7.1 (Teorema de Pappus) Sean ABC un tri angulo, AA B B y CC A A dos paralelogramos construidos sobre AC y AB , de manera que ambos est en hacia adentro o ambos hacia afuera del tri angulo. Sea P la intersecci on de B A con C A . Construyamos otro paralelogramo BP P C sobre BC de manera que BP sea paralela a AP y de la misma longitud. Entonces se tiene la siguiente relaci on entre las areas, (BP P C ) = (AA B B ) + (CC A A).

94 Demostraci on. A

Desigualdades Geom etricas

P A A

B B P C P

Teorema 2.7.2 (Teorema de Erd os-Mordell) Sea P un punto arbitrario dentro o sobre la frontera de un tri angulo ABC . Si pa , pb , pc son las distancias de P a los lados a, b, c de ABC , respectivamente, entonces P A + P B + P C 2 ( pa + pb + pc ) . Adem as, se da la igualdad si y s olo si el tri angulo ABC es equil atero y P es el circuncentro. Demostraci on. (Kazarino) Al tri angulo ABC , lo reejamos con respecto a la bisectriz BL del angulo B . Sean A y C los reejados de A y C . Al punto P no lo reejamos. Ahora consideramos los paralelogramos determinados por B , P y A , y por B , P y C . C A P
L

C B A La suma de las areas de estos paralelogramos es cpa + apc y es igual al area del paralelogramo A P P C , donde A P es paralela a BP y de igual longitud. El area de A P P C es menor o igual que b P B . Adem as, las areas son iguales si BP es perpendicular a A C si y s olo si P est a sobre BO, donde O es el circuncentro de ABC 9 .
BP es perpendicular a A C si y s olo si P BA = 90 A , pero A = A y OBC = 90 A, luego P deber a de estar sobre BO.
9

2.7 Teorema de Erd os-Mordell P C C

95

a pc P pa B Luego, Por lo tanto, c A B cpa + apc bP B. c a P B pa + pc . b b

b P A

An alogamente, PA c b pc + pb a a y b a P C pa + pb . c c

Al sumar las desigualdades, tenemos P A+P B +P C b c + c b pa + a b c a pb + + + a c b a pc 2 ( pa + pb + pc ) ,

c ya que b as, tenemos la igualdad si y s olo si a = b = c y P se c + b 2. Adem encuentra en AO, BO y CO. Es decir, si el tri angulo es equil atero y P = O.

Ejemplo 2.7.3 Con la notaci on del teorema de Erd os-Mordell, muestre que aP A + bP B + cP C 4(ABC ).

Considere los dos paralelogramos que determinan B , C , P y B , A, P , como se muestra en la gura, y el paralelogramo que se construye siguiendo el teorema de Pappus. Es claro que bP B apa + cpc .

96

Desigualdades Geom etricas

A c pc P pa B De manera an aloga, se tienen aP A bpb + cpc , Por lo tanto, aP A + bP B + cP C 2(apa + bpb + cpc ) = 4(ABC ). Ejemplo 2.7.4 Con la notaci on del teorema de Erd os-Mordell, muestre que pa P A + pb P B + pc P C 2 ( pa pb + pb pc + pc pa ) . Como en el ejemplo anterior, tenemos que aP A bpb + cpc . Por lo que, pa P A c b pa pb + pc pa . a a a C b

cP C apa + bpb .

c a b a pa pb + pb pc , pc P C pc pa + pb pc . b b c c Al sumar las tres desigualdades, obtenemos An alogamente, tenemos que pb P B pa P A + pb P B + pc P C a b b c + + pa pb + b a c b 2 ( pa pb + pb pc + pc pa ) . pb pc + c a pc pa + a c

Ejemplo 2.7.5 Con la notaci on del teorema de Erd os-Mordell, muestre que 2 1 1 1 + + PA PB PC 1 1 1 + . + pa pb pc

2.7 Teorema de Erd os-Mordell A

97

C1

C1

A P C A 1

B1

A1

Hagamos la inversi on en la circunferencia de centro P y radio d = pb . Si A , B , C son los inversos de A, B , C , respectivamente, y A 1 , B1 , C1 , los inversos de A1 , B1 , C1 , respectivamente, tenemos que
2 P A1 P A 1 = P B1 P B 1 = P C1 P C 1 = d . C , C A , y A B , respectivaM as a un, A , B y C se encuentran sobre B1 1 1 1 1 1 C , C A y mente, y los segmentos P A , P B y P C son perpendiculares a B1 1 1 1 A 1 B1 , respectivamente. Aplicando el teorema de Erd os-Mordell al tri angulo A 1 B1 C1 , obtenemos que P A1 + P B1 + P C1 2 (P A + P B + P C ). Pero como,

P A P A = P B P B = P C P C = d2

P A 1 = P C = luego, d2 por lo tanto, 2

d2 d2 d2 , P B1 = , P C1 = , P A1 P B1 P C1 d2 , PC P B = d2 d2 , P A = , PB PA

1 1 1 + + P A1 P B1 P C1

2d2

1 1 1 + + PA PB PC

1 1 1 + + PA PB PC

1 1 1 + + pa pb pc

98

Desigualdades Geom etricas

Ejemplo 2.7.6 Utilizando la notaci on del teorema de Erd os-Mordell, muestre que R (pa + pb ) (pb + pc ) (pc + pa ) . PA PB PC 2r

A pc P pa B c C1 C b

Sea C1 sobre BC tal que BC1 = AB . Luego, AC1 = 2c sen B 2 , y el teorema de c p + c p . Por lo tanto, Pappus nos garantiza que, P B 2c sen B a c 2 PB An alogamente, PA pb + pc 2 sen A 2 y PC pa + pb . 2 sen C 2 pa + pc . 2 sen B 2

Luego, al multiplicar obtenemos PA PB PC 1 8 sen A 2 1 sen B 2 (pa + pb ) (pb + pc ) (pc + pa ) , sen B 2 sen C 2 =

sen C 2

La soluci on del ejercicio 2.42, nos ayuda a probar que sen A 2 r 4R , de donde el resultado se sigue inmediatamente.

Ejemplo 2.7.7 (IMO, 1991) Sea P un punto interior del tri angulo ABC . Muestre que alguno de los angulos P AB , P BC , P CA es menor o igual a 30 . Tracemos A1 , B1 y C1 las perpendiculares desde P a los lados BC , CA y AB , respectivamente. Por el teorema de Erd os Mordell, obtenemos que P A + P B + P C 2P A1 + 2P B1 + 2P C1 .

2.7 Teorema de Erd os-Mordell

99

C1

B1

A1

Luego, alguna de las desigualdades siguientes deber a ocurrir, P A 2P C1 , P B 2P A1 o P C 2P B1 .

P C1 Si por ejemplo, P A 2P C1 , se tiene que 1 2 P A = sen P AB , por lo que P AB 30 o P AB 150 . Pero si, P AB 150 , entonces deber a suceder que P BC < 30 y en cualquier caso terminamos.

Ejemplo 2.7.8 (IMO, 1996) Sea ABCDEF un hex agono convexo, donde AB es paralelo a DE , BC paralelo a EF y CD paralelo a F A. Sean RA , RC , RE los circunradios de los tri angulos F AB , BCD y DEF , respectivamente, y sea P el per metro del hex agono. Muestre que RA + R C + R E P . 2

Sean M , N y P puntos dentro del hex agono de manera que M DEF , N F AB y P BCD sean paralelogramos. Sea XY Z el tri angulo formado por las rectas que pasan a trav es de B , D, F perpendiculares a F A, BC , DE , respectivamente, donde B est a sobre Y Z , D sobre ZX y F sobre XY . Observemos que M N P y XY Z son tri angulos semejantes. X

E F A B
N P M

D C

100

Desigualdades Geom etricas

Como los tri angulos DEF y DM F son congruentes, estos tienen el mismo circunradio; adem as, como XM es un di ametro del circunc rculo del tri angulo DM F , luego XM = 2RE . An alogamente, Y N = 2RA y ZP = 2RC . As , la desigualdad que se desea mostrar se puede escribir como XM + Y N + ZP BN + BP + DP + DM + F M + F N. El caso M = N = P , es la desigualdad de Erd os-Mordell, en la que se basa lo que sigue de la prueba. X
F
M

Sean Y , Z las respectivas reexiones de Y y Z con respecto a la bisectriz interna del X . Sean G, H los pies de las perpendiculares de M , X sobre Y Z , respectivamente. Como (XY Z ) = (XM Z ) + (Z M Y ) + (Y M X ), tenemos XH Y Z = M F XZ + M G Y Z + M D Y X. Si hacemos x = Y Z , y = ZX , z = XY , la igualdad es xXH = xM G + zDM + yF M. Como XHG = 90o , entonces XH = XG sen XGH XG . Adem as, por la desigualdad del tri angulo, XG XM + M G, tenemos que z y XM XH M G = DM + F M. x x An alogamente, YN ZP z x F N + BN , y y x y BP + DP. z y

Al sumar estas tres desigualdades, obtenemos z y x z y x XM + Y N + ZP DM + F M + F N + BN + BP + DP. (2.9) x x y y z z

2.7 Teorema de Erd os-Mordell Observemos que z y BP + BN = z y y z + z y BP + BN 2 + y z z y BP BN 2

101

Como los tri angulos XY Z y M N P son semejantes, podemos denir r como r= BN BP DP DM FM FN = = . XY YZ ZX


y z

Al aplicar la desigualdad y z BP + BN z y

z y

2, obtenemos yx zx z y

BP + BN r 2 2 r yx zx BP + BN . 2 z y

y z + z y

Desigualdades an alogas se tienen para y x FN + FM y x x z DM + DP x z r xz yz , 2 y x r zy xy . DM + DP 2 x z FN + FM

Al sumar las desigualdades y sustituirlas en la ecuaci on (2.9), tenemos que XM + Y N + ZP BN + BP + DP + DM + F M + F N como se deseaba. Ejercicio 2.70 Utilizando la notaci on del teorema de Erd os-Mordell, muestre que 4R PA PB PC pa pb pc . r

Ejercicio 2.71 Utilizando la notaci on del teorema de Erd os-Mordell, muestre que: (i) (ii) P A2 P B 2 P C 2 + + 12, pb pc pc pa pa pb PB PC PA 3, + + pb + pc pc + pa pa + pb

102 PA PB PC (iii) 6, + + pb pc pc pa pa pb

Desigualdades Geom etricas

(iv ) P A P B + P B P C + P C P A 4(pa pb + pb pc + pc pa ). Ejercicio 2.72 Sea ABC un tri angulo, P un punto arbitrario del plano y sean pa , pb y pc las distancias de P a los lados del tri angulo con longitudes a, b y c, respectivamente. Si por ejemplo, P y A est an en distintos lados de la recta BC , entonces pa es negativo, y lo mismo pasa en los otros dos casos. Muestre que PA + PB + PC b c + c b pa + c a pb + + a c a b + b a pc .

2.8.

Problemas de optimizaci on

En esta secci on damos dos ejemplos cl asicos conocidos como el problema de Fermat-Steiner y el problema de Fagnano. El problema de Fermat-Steiner. Este problema plantea encontrar un punto en el interior del tri angulo o sobre los lados, de manera que la suma de las distancias del punto a los v ertices del tri angulo sea m nima. Presentaremos tres soluciones donde resaltamos los m etodos para resolver el problema. La soluci on de Torricelli. Esta se basa en los siguientes dos lemas.

Lema 2.8.1 (Lema de Viviani) La suma de las distancias de un punto interior a los lados de un tri angulo equil atero es igual a la altura del tri angulo. Demostraci on. Sea P un punto interior del tri angulo ABC . Tracemos el tri angulo A B C de lados paralelos a los lados de ABC , con P sobre C A y B C sobre la recta por B y C . A A

M P B B N P L P L C M

2.8 Problemas de optimizaci on

103

Si L, M y N son los pies de las perpendiculares sobre los lados, es claro que P M = N M , donde M es la intersecci on de P N con A B . Adem as, P M es altura del tri angulo equil atero A P P . Si A P es la altura del tri angulo A P P desde A , es claro que P M = A P . Sea L el pie de la altura desde el v ertice A del tri angulo A B C . Luego, P L + P M + P N = P L + N M + P N = P L + A P = A P + P L = A L .

Observaci on 2.8.2 (i) Hay una prueba del lema de Viviani que se basa en el manejo de las areas. Denotemos por (ABC ) el area del tri angulo ABC , entonces (ABC ) = (ABP ) + (BCP ) + (CAP ). Luego, si a es la longitud del lado del tri angulo y h la longitud de su altura, se tiene que ah = aP N + aP L + aP M , cancelando a obtenemos h = P N + P L + P M . (ii) Otra demostraci on del lema de Viviani se deduce de la siguiente gura A N P
M

Lema 2.8.3 Si ABC es un tri angulo con angulos menores o iguales a 120 , hay un u nico punto P tal que AP B = BP C = CP A = 120 . El punto P se conoce como el punto de Fermat.

Demostraci on. Primero veamos la existencia de P . Sobre los lados AB y CA construimos tri angulos equil ateros ABC y CAB . Sus circunc rculos se intersectan en A y en otro punto que denotamos por P .

104

Desigualdades Geom etricas B A C P B C

Como AP CB es c clico, tenemos que, CP A = 180o B = 120o . An alogamente, por ser AP BC c clico, AP B = 120o . Finalmente, BP C = 360o AP B CP A = 360o 120o 120o = 120o . Para la unicidad, supongamos que Q cumple con AQB = BQC = CQA = 120o . Como AQB = 120o , el punto Q deber a estar en el circunc rculo de ABC . An alogamente, en el circunc rculo de CAB , por lo que Q = P . Estudiaremos ahora la soluci on de Torricelli al problema de Fermat-Steiner. Dado el tri angulo ABC con angulos menores o iguales a 120o , construimos el punto de Fermat P , este satisface que AP B = BP C = CP A = 120o . Ahora por A, B y C tracemos perpendiculares a AP , BP y CP , respectivamente.
A A

F
P

B B

C C

D Estas perpendiculares determinan un tri angulo DEF . Veamos que este es equil atero; como el cuadril atero P BDC es c clico por tener en B y C angulos de 90 y como BP C = 120 , podemos deducir que BDC = 60 . Podemos repetir este argumento en cada angulo, luego DEF es equil atero. Ahora bien, sabemos que la distancia de P a los v ertices del tri angulo ABC es igual a la longitud de la altura del tri angulo equil atero DEF . Observemos que cualquier otro punto Q, dentro del tri angulo ABC , satisface que AQ A Q,

2.8 Problemas de optimizaci on

105

donde A Q es la distancia de Q al lado EF , an alogamente, BQ B Q y CQ C Q. Por lo tanto, AQ + BQ + CQ es mayor o igual que la longitud de la altura de DEF que es AP + BP + CP , que a su vez es igual a A Q + B Q + C Q, esto u ltimo se sigue del lema de Viviani. La soluci on de Hofmann-Gallai. Esta forma de resolver el problema usa la ingeniosa idea de rotar la gura para tratar de colocar los tres segmentos que necesitamos, uno seguido de otro, para formar una poligonal y luego sumarlos. Entonces, al unir con un segmento los dos puntos extremos y como el camino m as corto entre ellos es este segmento de recta, se buscan entonces las condiciones bajo las cuales la poligonal est a sobre tal recta. La demostraci on la da J. Hofmann en 1929, pero el m etodo para resolverlo era ya conocido y deber a ser atribuido al h ungaro Tibor Gallai. Estudiemos entonces esta soluci on. Consideremos un tri angulo ABC y un punto P dentro de el; tracemos AP , BP y CP . Despu es giremos la gura alrededor de B un angulo de 60 , en sentido positivo. A C P
60

P C

Tenemos varias cosas que se nalar. Si C es la imagen de A y P la imagen de P , los tri angulos BP P y BAC son equil ateros. Adem as, AP = P C y BP = P B = P P , por lo que, AP + BP + CP = P C + P P + CP . La trayectoria CP + P P + P C es m nima cuando C , P , P y C son colineales. Esta u ltima condici on exige que C P B = 120 y BP C = 120 ; pero como C P B = AP B , el punto P debe cumplir que AP B = BP C = 120 (y entonces tambi en CP A = 120 ). Una ventaja de esta soluci on es que proporciona otra caracterizaci on del punto de Fermat y otra forma de localizarlo. Si revisamos la demostraci on podemos resaltar que el punto P se encuentra en el segmento CC , donde C es el tercer v ertice del tri angulo equil atero de lado AB , pero si en lugar de girar con centro en B , se gira alrededor de C , se obtiene otro tri angulo equil atero AB C y se puede concluir que P se encuentra sobre BB . Luego, podemos encontrar P

106 como la intersecci on de BB y CC .

Desigualdades Geom etricas

La soluci on de Steiner. Cuando tratamos de resolver problemas de m aximos y m nimos nos enfrentamos principalmente a tres preguntas, (i) existe una soluci on?, (ii) es u nica la soluci on? (iii) qu e propiedades caracterizan a la o las soluciones? La soluci on de Torricelli muestra que, de entre todos los puntos del tri angulo, este punto particular P , desde el cual se ven los tres lados del tri angulo con un angulo de 120 , corresponde al menor valor de P A + P B + P C . De esta forma responde a las tres preguntas que se nalamos, haci endolo adem as de una manera elegante. Sin embargo, la soluci on no da indicios de por qu e Torricelli elige tal punto; Cu al fu e su primer impulso para tomar ese punto? Probablemente esta pregunta no se podr a responder. En lo que sigue veremos una sucesi on de ideas que nos llevan a descubrir por qu e el punto de Fermat es el optimo, estas son debidas al ge ometra suizo Jacob Steiner. Antes veamos los siguientes dos lemas. Lema 2.8.4 (Problema de Her on) Encontrar la trayectoria m as corta entre dos puntos A y B que est an del mismo lado de una recta d, pasando por la recta. B A d P La distancia m as corta entre A y B , pasando por la recta d, la podemos encontrar de la siguiente manera. Reejemos B sobre d para obtener un punto B ; el segmento AB corta a d en un punto P que hace que AP + P B sea el m nimo entre las cantidades AP + P B , con P en d. B A d P P

2.8 Problemas de optimizaci on Para convencernos basta observar que AP + P B = AP + P B = AB AP + P B = AP + P B.

107

Este punto cumple con el siguiente principio de reexi on: el angulo de incidencia es igual al angulo de reexi on. Desde luego, el punto con esta propiedad es el m nimo. Lema 2.8.5 (Problema de Her on con circunferencia) Encontrar la trayectoria m as corta entre dos puntos A y B que est an fuera de una circunferencia C , tocando a la circunferencia. A

B C

Veremos u nicamente un esbozo de la soluci on. Sea D un punto sobre C , entonces tenemos que el conjunto {P : P A + P B = DA + DB } es una elipse ED , con focos en los puntos A y B , y que D pertenece a ED . En general, Ed = {P : P A + P B = d}, donde d es un n umero positivo, es una elipse con focos en A y B (si d > AB ). M as a un, estas elipses tienen la olo si d < d . propiedad de que Ed es un subconjunto del interior de Ed si y s Luego, queremos encontrar el punto Q en C tal que QA + QB sea m nima. El punto optimo Q pertenece a una elipse, precisamente a EQ . Esta elipse EQ no intersecta a C en otro punto, de hecho, si C es otro punto en com un de EQ y C , entonces todo punto C , en el arco de circunferencia entre Q y C de C , ser a un punto interior de EQ , luego C A + C B < QA + QB y Q no ser a el optimo, lo que es una contradicci on. Luego, el punto Q que minimiza AQ + QB debe pertenecer a la elipse EQ que es tangente a C en Q. La recta tangente com un a EQ y C deber a ser perpendicular al radio CQ, donde C es el centro de C y, por el principio de reexi on de la elipse ( angulo de incidencia es igual al angulo de reexi on), deber a suceder que CQ es bisectriz interna del angulo AQB y entonces BQC = CQA.

108

Desigualdades Geom etricas

C B

Q A

Ahora volvamos a la soluci on de Steiner del problema de Fermat-Steiner. Un punto P que hace que la suma P A + P B + P C sea m nima puede ser uno de los v ertices A, B , C o un punto del tri angulo distinto a los v ertices. En el primer caso, si P es un v ertice, entonces un t ermino de la suma P A + P B + P C es cero y los otros dos son las longitudes de los lados del tri angulo ABC que tienen en com un el v ertice elegido. Luego, la suma ser a m nima cuando el v ertice elegido sea el opuesto al lado mayor del tri angulo. Para analizar el segundo de los casos, Steiner sigue una idea, muy u til en los problemas de optimizaci on y que podr a caer en la estrategia de divide y vencer as, que es mantener jas algunas de las variables y optimizar las restantes. Las condiciones resultantes en las variables no jas ir an restringuiendo el espacio soluci on hasta llegar al optimo global. Concretamente proced a as : Supongamos que P A tiene una longitud ja, es decir, P pertenece a la circunferencia de centro en A y radio P A. Ahora queremos encontrar el punto P que hace que la suma P B + P C sea m nima. Observemos que B tiene que estar fuera de dicho c rculo, de lo contrario P A AB y por la desigualdad del tri angulo P B + P C > BC , entonces P A + P B = P C > AB + BC . Por lo tanto, B ser a un mejor punto (en lugar de P ). An alogamente, C debe estar fuera del c rculo. Ahora bien, como B y C son puntos fuera de la circunferencia C = (A, P A), el punto optimo para minimizar P B + P C , con la condici on de que P sea un punto sobre la circunferencia C es, por el lema 2.8.5, un punto Q sobre C , de forma tal que esta circunferencia sea tangente a la elipse con focos B y C en Q. Este punto Q satisface la condici on de que los angulos AQB y CQA sean iguales. Como los papeles de A, B y C puede intercambiarse, si ahora jamos el punto B (y P B ), entonces el punto optimo Q satisface que los angulos AQB y BQC son iguales, y entonces AQB = BQC = CQA = 120 . Por lo tanto, Q tiene que ser el punto de Fermat. Todo lo anterior funciona en este segundo caso para asegurar que Q est a dentro del tri angulo ABC , si los angulos del tri angulo no son mayores que 120 . El problema de Fagnano Este plantea encontrar, dentro de un tri angulo

2.8 Problemas de optimizaci on

109

acut angulo, un tri angulo inscrito de per metro m nimo. Damos dos soluciones cl asicas, donde el reejar sobre rectas juega un papel central. Una debida a H. Schwarz y la otra a L. Fej er. La soluci on de Schwarz. El matem atico alem an Hermann Schwarz dio la siguiente soluci on del problema bas andose en dos observaciones las cuales aparecen en los siguientes dos lemas. Estos lemas nos hacen ver que el tri angulo inscrito de menor per metro es el tri angulo formado con los pies de las alturas del tri angulo, el cual se conoce como tri angulo ortico. Lema 2.8.6 Sea ABC un tri angulo. Sean D, E y F los pies de las alturas sobre BC , CA y AB , desde los v ertices A, B y C , respectivamente. Entonces los tri angulos ABC , AEF , DBF y DEC son semejantes. Demostraci on. Basta ver que los primeros dos tri angulos son semejantes, ya que las otras semejanzas se demuestran de manera an aloga. A

E F
H

Como estos dos tri angulos tienen en com un al angulo en A, bastar a ver que AEF = ABC . Pero es claro que AEF + F EC = 180 y como el cuadril atero BCEF es c clico se tiene que ABC + F EC = 180 , por lo tanto AEF = ABC .

Lema 2.8.7 Con la notaci on del lema anterior se tiene que, el reejado de D con respecto a AB es colineal a E y F , y el reejado de D con respecto a CA es colineal con E y F . Demostraci on. Es inmediata del lema anterior.

110 A

Desigualdades Geom etricas

D E F D B D C

Con estos elementos podemos ahora nalizar la soluci on, propuesta por H. Schwarz, al problema de Fagnano. Demostraremos ahora que el tri angulo de per metro m nimo es el tri angulo ortico. Denotemos a este por DEF y consideremos otro tri angulo LM N inscrito en ABC . C
E M L

A N
F

A N F

C Reejemos la gura completa sobre el lado BC , despu es el tri angulo resultante lo reejamos sobre CA, luego sobre AB , sobre BC y nalmente en CA. Tenemos en total seis tri angulos congruentes y dentro de cada uno de ellos tenemos su tri angulo ortico y el otro tri angulo inscrito LM N . El lado AB del u ltimo tri angulo es paralelo al lado AB del primero, ya que como resultado de la primera reexi on, el lado AB se rota en sentido negativo un angulo 2B , despu es en sentido negativo un angulo 2A, en la tercera reexi on queda invariante, en la cuarta reexi on gira un angulo 2B en sentido positivo y en la quinta tambi en gira en sentido positivo un angulo 2A. Luego, el angulo total de rotaci on del segmento AB es cero. El segmento F F es igual a dos veces el per metro del tri angulo ortico, ya que F F se compone de seis pedazos, donde cada lado del ortico est a tomado dos veces. Tambi en la l nea quebrada N N es el doble del per metro de LM N .

2.8 Problemas de optimizaci on

111

Adem as, N N es una recta paralela a F F y de la misma longitud, luego como la longitud de la quebrada N N es mayor que la longitud del segmento N N , tenemos que el per metro de DEF es menor que el per metro de LM N . La soluci on de Fej er. La soluci on que da el matem atico h ungaro L. Fej er se basa tambi en en reexiones. Sea LM N un tri angulo inscrito en ABC . Tomemos la reexi on L del punto L sobre el lado CA y L la reexi on de L sobre el lado AB , tracemos los segmentos M L y N L , es claro que LM = M L y que L N = N L. Por lo que el per metro de LM N satisface que, LM + M N + N L = L N + N M + M L L L . A

L B

De lo anterior podemos asegurar que, si el punto L est a jo, los puntos M y N que hacen m nimo el per metro LM N son las intersecciones de L L con CA y AB , respectivamente. Ahora veamos cu al es la mejor opci on para el punto L. Ya sabemos que el per metro de LM N es L L , as que el punto L deber a hacer que esta cantidad sea m nima. Es evidente que, AL = AL = AL y que AC y AB son bisectrices de los angulos LAL y L AL, respectivamente, por lo que L AL = 2BAC = 2, el cual es un angulo jo. A

L B C

112

Desigualdades Geom etricas

La ley de los cosenos aplicada al tri angulo AL L nos garantiza que (L L )2 = (AL )2 + (AL )2 2AL AL cos 2 = 2AL2 (1 cos 2). As , L L es m nima cuando AL lo sea, es decir, cuando AL sea la altura10 . Un an alisis semejante desde los puntos B y C nos lleva a que tambi en BM y CN deber an ser alturas. Luego, el tri angulo LM N de per metro m nimo es el tri angulo ortico. Ejercicio 2.73 Sea ABCD un cuadril atero c clico convexo. Si O es la intersecci on de las diagonales AC y BD, y P , Q, R, S son los pies de las perpendiculares desde O sobre los lados AB , BC , CD, DA, respectivamente, muestre que P QRS es el cuadril atero de per metro m nimo inscrito en ABCD. Ejercicio 2.74 Sea P un punto dentro del tri angulo ABC . Sean D, E y F los puntos de intersecci on de AP , BP y CP con los lados BC , CA y AB , respectivamente. Determine P tal que el area del tri angulo DEF sea m axima. Ejercicio 2.75 (IMO, 1981) Sea P un punto dentro del tri angulo ABC . Sean D, E y F los pies de las perpendiculares desde P a los lados BC , CA y AB , CA AB BC respectivamente. Encuentre el punto P que minimiza a P D + PE + PF . Ejercicio 2.76 Sean P , D, E y F , como en el ejercicio 2.75. Para qu e punto 2 2 2 P , la suma BD + CE + AF es m nima? Ejercicio 2.77 Sean P , D, E y F como en el ejercicio 2.75. Para qu e punto P , el producto P D P E P F es m aximo? Ejercicio 2.78 Sea P un punto dentro del tri angulo ABC . Para qu e punto P , la suma P A2 + P B 2 + P C 2 es m nima? Ejercicio 2.79 Para cada punto P del circunc rculo del tri angulo ABC , se trazan las perpendiculares P M y P N a los lados AB y CA, respectivamente. Determine en que punto P , la longitud M N es m axima y cu al es su longitud.
Hasta aqu bastar a para terminar la demostraci on de Fej er al problema de Fagnano, ya que si AL es altura y L L corta en E y F a los lados CA y AB , respectivamente, entonces BE y CF son alturas. Veamos porqu e. El tri angulo AL L es is osceles con L AL = 2A, luego, AL L = 90 A y por simetr a ELA = 90 A, por tanto CLE = A. Luego, AELB es un cuadril atero c clico, por lo tantoAEB = ALB = 90 , lo que implica que BE sea altura. An alogamente, CF es altura.
10

2.8 Problemas de optimizaci on

113

Ejercicio 2.80 (Turqu a, 2000) Sea ABC un tri angulo acut angulo con circunradio R, las longitudes de las alturas AD, BE y CF son ha , hb y hc , respectivamente. Sean ta , tb y tc las longitudes de las tangentes desde A, B y C , respectivamente, al circunc rculo de DEF . Muestre que t2 t2 3 t2 a + b + c R. ha hb hc 2 Ejercicio 2.81 Sean ha , hb , hc las longitudes de las alturas de un tri angulo ABC y pa , pb , pc las distancias desde un punto P a los lados BC , CA, AB , respectivamente, donde P es un punto dentro del tri angulo ABC . Muestre que: (i) ha hb hc + + 9. pa pb pc

(ii) ha hb hc 27pa pb pc . (iii) (ha pa )(hb pb )(hc pc ) 8pa pb pc . Ejercicio 2.82 Si h es la longitud de la altura m as grande de un tri angulo acut angulo entonces r + R h. Ejercicio 2.83 De los tri angulos que tienen una base com un y un mismo per metro, el is osceles es el que tiene mayor area. Ejercicio 2.84 De todos los tri angulos con un per metro dado, el tri angulo equil atero es el que tiene mayor area. Ejercicio 2.85 De todos los tri angulos inscritos en un c rculo dado, el tri angulo equil atero es el que tiene mayor per metro. Ejercicio 2.86 Si P es un punto dentro del tri angulo ABC , l = P A, m = P B y n = P C , muestre que (lm + mn + nl)(l + m + n) a2 l + b2 m + c2 n. Ejercicio 2.87 (IMO, 1961) Sean a, b y c las longitudes de los lados de un tri angulo ABC y sea (ABC ) su a rea, muestre que 4 3(ABC ) a2 + b2 + c2 .

114

Desigualdades Geom etricas

Ejercicio 2.88 Sea (ABC ) el area del tri angulo ABC y sea F el punto de Fermat del tri angulo. Muestre que 4 3(ABC ) (AF + BF + CF )2 . Ejercicio 2.89 Sea P un punto dentro del tri angulo ABC , muestre que P A + P B + P C 6r. Ejercicio 2.90 (El area del tri angulo pedal). Para un tri angulo ABC y un punto P del plano, se dene el tri angulo pedal de P con respecto a ABC como el tri angulo A1 B1 C1 , donde A1 , B1 , C1 son los pies de las perpendiculares desde P sobre BC , CA, AB , respectivamente. Muestre que el area del tri angulo A1 B1 C1 satisface (A1 B1 C1 ) = (R2 OP 2 )(ABC ) , 4R2

donde O es el circuncentro y (ABC ) es el area del tri angulo ABC . Concluya que el tri angulo pedal de area m axima es el tri angulo medial.

Cap tulo 3

Problemas Recientes de Desigualdades

Problema 3.1 (Bulgaria, 1995) Sean SA , SB y SC las areas de los hept agonos regulares A1 A2 A3 A4 A5 A6 A7 , B1 B2 B3 B4 B5 B6 B7 y C1 C2 C3 C4 C5 C6 C7 , respectivamente. Suponga que A1 A2 = B1 B3 = C1 C4 , muestre que SB + SC 1 < < 2 2. 2 SA Problema 3.2 (Rep ublicas Checa y Eslovaca, 1995) Sea ABCD un tetraedro con BAC + CAD + DAB = ABC + CBD + DBA = 180 . Muestre que CD AB . Problema 3.3 (Estonia, 1995) Sean a, b, c las longitudes de los lados de un tri angulo y , , los angulos opuestos a los lados, respectivamente. Muestre que si el inradio del tri angulo es r , entonces a sen + b sen + c sen 9r.

116

Problemas Recientes de Desigualdades

Problema 3.4 (Francia, 1995) Tres c rculos con el mismo radio tienen un punto com un. Si S es el area del conjunto de puntos que son interiores al menos a dos c rculos, c omo deber an colocarse los c rculos para que S sea m nima? Problema 3.5 (Alemania, 1995) Sea ABC un tri angulo con D y E puntos sobre BC y CA, respectivamente, de manera que DE pasa por el incentro de ABC . Si S = area(CDE ) y r es el inradio, muestre que S 2r 2 . Problema 3.6 (Irlanda, 1995) Sean A, X , D puntos sobre una recta con X entre A y D. Sea B un punto tal que ABX = 120 y sea C un punto entre B y X . Muestre que 2AD 3 (AB + BC + CD). Problema 3.7 (Corea, 1995) Un n umero nito de puntos del plano, tienen la propiedad de que cada tres de ellos forman un tri angulo de area menor o igual a 1. Muestre que todos los puntos est an dentro o sobre los lados de un tri angulo de area menor o igual a 4. Problema 3.8 (Polonia, 1995) Para un entero positivo n jo, encuentre el valor m nimo de la suma x1 + x3 xn x2 2 + 3 + + n, 2 3 n

donde x1 , x2 , . . . , xn son n umeros positivos que satisfacen que la suma de sus rec procos es n. Problema 3.9 (IMO, 1995) Sea ABCDEF un hex agono convexo con AB = BC = CD y DE = EF = F A, tal que BCD = EF A = 3 . Sean G y H puntos en el interior del hex agono tales que AGB = DHE = 23 . Muestre que AG + GB + GH + DH + HE CF. Problema 3.10 (Balc anica, 1996) Sean O el circuncentro y G el centroide de un tri angulo ABC . Sean R y r el circunradio y el inradio del tri angulo. Muestre que OG R(R 2r ). Problema 3.11 (China, 1996) Sup ongase que x0 = 0, xi > 0 para i = 1, 2, ..., n, y n x = 1 . Muestre que i=1 i
n

i=1

xi < . 1 + x0 + + xi1 xi + + xn 2

Problemas Recientes de Desigualdades Problema 3.12 (Polonia, 1996) Sean n 2 y a1 , a2 , ..., an R+ con 1. Muestre que para x1 , x2 , ..., xn R+ con n i=1 xi = 1, se tiene 2
i<j

117
n i=1 ai

xi xj

n2 + n1

n i=1

ai x2 i . 1 ai

Problema 3.13 (Rumania, 1996) Sean x1 , x2 , ..., xn , xn+1 n umeros reales positivos con x1 + x2 + + xn = xn+1 . Muestre que
n i=1 n

xi (xn+1 xi )

i=1

xn+1 (xn+1 xi ).

Problema 3.14 (San Petesburgo, 1996) Sean M la intersecci on de las diagonales de un cuadril atero c clico, N la intersecci on de los segmentos que unen los puntos medios de lados opuestos y O el circuncentro. Muestre que OM ON . Problema 3.15 (Austria-Polonia, 1996) Si w, x, y y z son n umeros reales que satisfacen w + x + y + z = 0 y w2 + x2 + y 2 + z 2 = 1. Muestre que 1 wx + xy + yz + zw 0.

Problema 3.16 (Taiwan, 1997) Sean a1 , ..., an n umeros positivos, tales que ai1 +ai+1 es entero para toda i = 1 , ..., n , a = a , a 0 n n+1 = a1 y n 3. Muestre ai que an1 + a1 an + a2 a1 + a3 a2 + a4 + + + + 3n. 2n a1 a2 a3 an Problema 3.17 (Taiwan, 1997) Sea ABC un tri angulo acut angulo, con circuncentro O y circunradio R. Muestre que si AO corta al circunc rculo de OBC en D, BO corta al circunc rculo de OCA en E y CO corta al circunc rculo de OAB en F , entonces OD OE OF 8R3 . Problema 3.18 (APMO, 1997) Sea ABC un tri angulo. La bisectriz interna del angulo A intersecta al segmento BC en X y al circunc rculo en Y . Sea AX alogamente, dena lb y lc . Muestre que la = AY . An la lb lc + + 3 2 2 sen A sen B sen2 C

118

Problemas Recientes de Desigualdades

con igualdad si y s olo si el tri angulo es equil atero. Problema 3.19 (IMO, 1997) Sean x1 , ..., xn n umeros reales con |x1 + + xn | n+1 on y1 , ..., = 1 y |xi | 2 , para i = 1, ..., n. Muestre que existe una permutaci yn de x1 , ..., xn tal que |y1 + 2y2 + + nyn | n+1 . 2

Problema 3.20 (Rep ublicas Checa y Eslovaca, 1998) Sean a, b, c n umeros reales positivos. Existe un tri angulo con lados de longitud a, b, c si y s olo si existen n umeros x, y , z , tales que a y z + = , z y x z x b + = , x z y x y c + = . y x z

Problema 3.21 (Hungr a, 1998) Sea ABCDEF un hex agono centralmente sim etrico y P , Q, R puntos sobre los lados AB , CD, EF , respectivamente. Muestre que el area del tri angulo P QR es a lo m as un medio del area del hex agono. Problema 3.22 (Ir an, 1998) Sean x1 , x2 , x3 y x4 n umeros reales positivos que cumplen x1 x2 x3 x4 = 1. Muestre que
3 3 3 ax x1 + x2 + x3 + x4 , x3 1 + x2 + x3 + x4 m

1 1 1 1 + + + x1 x2 x3 x4

Problema 3.23 (Ir an, 1998) Sean x, y , z n umeros mayores que 1 y tales que 1 1 1 + + = 2 . Muestre que x y z x+y+z x1+ y1+ z 1.

Problema 3.24 (Mediterr anea, 1998) Sea ABCD un cuadrado inscrito en una circunferencia. Si M es un punto en el arco AB , muestre que M C M D 3 3M A M B.

Problemas Recientes de Desigualdades

119

Problema 3.25 (N ordica, 1998) Sea P un punto dentro de un tri angulo equil atero ABC con lados de longitud a. Si las rectas AP , BP y CP intersectan los lados BC , CA y AB del tri angulo en L, M y N , respectivamente, muestre que P L + P M + P N < a.

Problema 3.26 (Espa na, 1998) Una recta que contiene al centroide G del tri angulo ABC intersecta el lado AB en P y el lado CA en Q. Muestre que 1 P B QC . P A QA 4

Problema 3.27 (Armenia, 1999) Sea O el centro del circunc rculo del tri angulo acut angulo ABC . Las rectas CO, AO y BO intersectan, por segunda vez, al circunc rculo de los tri angulos AOB , BOC y AOC en C1 , A1 y B1 , respectivamente. Muestre que BB1 CC1 9 AA1 + + . OA1 OB1 OC1 2

Problema 3.28 (Balc anica, 1999) Sea ABC un tri angulo acut angulo y sean L, M , N los pies de las perpendiculares desde G, el centroide de ABC , a los lados BC , CA, AB , respectivamente. Muestre que 4 (LM N ) 1 < . 27 (ABC ) 4 Problema 3.29 (Bielorusia, 1999) Sean a, b, c n umeros reales positivos tales que a2 + b2 + c2 = 3. Muestre que 1 1 3 1 + + . 1 + ab 1 + bc 1 + ca 2 Problema 3.30 (Rep ublicas Checa y Eslovaca, 1999) Para n umeros positivos a, b y c, muestre que b c a + + 1. b + 2c c + 2a a + 2b

120

Problemas Recientes de Desigualdades

Problema 3.31 (Irlanda, 1999) Sean a, b, c, d n umeros reales positivos con a + b + c + d = 1. Muestre que b2 c2 d2 1 a2 + + + . a+b b+c c+d d+a 2 Problema 3.32 (Italia, 1999) Sean D y E puntos sobre los lados AB y CA de un tri angulo ABC , respectivamente, de manera que DE es paralela a BC y DE es tangente al inc rculo de ABC . Muestre que DE AB + BC + CA . 8

Problema 3.33 (Polonia, 1999) Sea D un punto sobre el lado BC del tri angulo ABC tal que AD > BC . Considere un punto E sobre el lado CA de manera AE BD que EC = AD BC . Muestre que AD > BE . Problema 3.34 (Rumania, 1999) Sean a, b, c n umeros reales positivos tales que ab + bc + ca 3abc. Muestre que a + b + c a3 + b3 + c3 . Problema 3.35 (Rumania, 1999) Sean x1 , x2 , ..., xn n umeros reales positivos tales que x1 x2 xn = 1. Muestre que 1 1 1 + + + 1. n 1 + x1 n 1 + x2 n 1 + xn Problema 3.36 (Rumania, 1999) Sea n 2 un entero positivo y sean x1 , y1 , x2 , y2 , ..., xn , yn n umeros reales positivos tales que x1 + x2 + + xn x1 y1 + x2 y2 + + xn yn . Muestre que x1 + x2 + + x n x1 x2 xn + + + . y1 y2 yn

Problema 3.37 (Rusia, 1999) Sean a, b y c n umeros reales positivos con 1 1 1 1 1 abc = 1. Muestre que si a+b+c a + b + c entonces an +bn +cn a1 n + bn + cn , para todo n entero positivo. Problema 3.38 (Rusia, 1999) Si {x} = x [x] es la parte fraccionaria del n umero x. Muestre que, para cada entero positivo n,
n2

j
j =1

n2 1 . 2

Problemas Recientes de Desigualdades

121

Problema 3.39 (Rusia, 1999) Los n umeros reales positivos x, y satisfacen x2 + y 3 x3 + y 4 . Muestre que x3 + y 3 2.

Problema 3.40 (San Petesburgo, 1999) Sean x0 > x1 > > xn n umeros reales. Muestre que x0 + 1 1 1 + + + xn + 2n. x0 x1 x1 x2 xn1 xn

Problema 3.41 (Turqu a, 1999) Muestre que (a +3b)(b +4c)(c +2a) 60abc, para todos los n umeros reales que satisfacen 0 a b c. Problema 3.42 (Reino Unido, 1999) Tres n umeros reales no negativos a, b y c satisfacen a + b + c = 1. Muestre que 7(ab + bc + ca) 2 + 9abc.

Problema 3.43 (E.U., 1999) Sea ABCD un cuadril atero convexo. Muestre que |AB CD| + |AD BC | 2 |AC BD| .

Problema 3.44 (APMO, 1999) Sea {an } una sucesi on de n umeros reales que satisfacen, ai+j ai + aj , para toda i, j = 1, 2, . . .. Muestre que a1 + an a2 + + an , para toda n N. 2 n

Problema 3.45 (IMO, 1999) Sea n 2 un entero dado.

122

Problemas Recientes de Desigualdades

(a) Determine la menor constante C para la cual se verica la desigualdad 4


1i<j n

para todos los n umeros reales no negativos x1 , ..., xn .

2 xi xj (x2 i + xj ) C

1in

xi ,

(b) Para esta constante C determine cuando ocurre la igualdad. Problema 3.46 (Rep ublicas Checa y Eslovaca, 2000) Muestre que para todos los n umeros reales positivos a y b,
3

a + b

b a

2(a + b)

1 1 + . a b

Problema 3.47 (Corea, 2000) Los n umeros reales a, b, c, x, y , z satisfacen a b c > 0 y x y z > 0. Muestre que b2 y 2 c2 z 2 3 a2 x2 + + . (by + cz )(bz + cy ) (cz + ax)(cx + az ) (ax + by )(ay + bx) 4

Problema 3.48 (Mediterranea, 2000) Sean P , Q, R, S los puntos medios de los lados BC , CD, DA, AB , respectivamente, del cuadril atero convexo ABCD. Muestre que 4(AP 2 + BQ2 + CR2 + DS 2 ) 5(AB 2 + BC 2 + CD2 + DA2 ).

Problema 3.49 (Austria-Polonia, 2000) Sean x, y , z n umeros reales no negativos tales que x + y + z = 1. Muestre que 2 (1 x2 )2 + (1 y 2 )2 + (1 z 2 )2 (1 + x)(1 + y )(1 + z ).

Problema 3.50 (IMO, 2000) Sean a, b, c n umeros reales positivos con abc = 1. Muestre que a1+ 1 b b1+ 1 c c1+ 1 a 1.

Problemas Recientes de Desigualdades

123

Problema 3.51 (Balc anica, 2001) Sea a, b, c n umeros reales positivos tales que abc a + b + c su suma. Muestre que a2 + b2 + c2 3 abc.

Problema 3.52 (Brasil, 2001) Muestre que (a + b)(a + c) 2 para todos los n umeros reales positivos a, b, c. Problema 3.53 (Polonia, 2001) Muestre que la desigualdad
n i=1

abc(a + b + c)

ixi

n 2

+
i=1

xi i

se tiene para cada entero n 2 y para todos los n umeros reales no negativos x1 , x2 , ..., xn . Problema 3.54 (Austria-Polonia, 2001) Muestre que 2< a + b b + c c + a a3 + b3 + c3 + + 3, c a b abc

donde a, b, c son las longitudes de los lados de un tri angulo. Problema 3.55 (IMO, 2001) Muestre que para cualesquiera n umeros reales positivos a, b y c se cumple a2 a b c + + 1. 2 2 + 8bc b + 8ca c + 8ab

Problema 3.56 (Lista corta IMO, 2001) Muestre que para todos los n umeros reales x1 , x2 , . . . , xn , x2 xn x1 + + + < n. 2 2 2 2 2 1 + x 1 1 + x 1 + x2 1 + x1 + + x n Problema 3.57 (Austria, 2002) Sean a, b, c n umeros reales tales que existen , , {1, 1}, con a + b + c = 0. Determine el valor positivo m as peque no de
a3 +b3 +c3 abc 2

124

Problemas Recientes de Desigualdades

Problema 3.58 (Balc anica, 2002) Muestre que 2 2 27 2 + + , b(a + b) c(b + c) a(c + a) (a + b + c)2 para cualesquiera n umeros reales positivos a, b, c. Problema 3.59 (Canad a, 2002) Muestre que para cualesquiera n umeros reales positivos a, b, c, a3 b3 c3 + + a + b + c, bc ca ab y determine cu ando ocurre la igualdad. Problema 3.60 (Irlanda, 2002) Muestre que para cualesquiera n umeros reales positivos x, y , z menores que 1, se tiene que 3 3 xyz y z x + + . 1x 1y 1z 1 3 xyz Problema 3.61 (Rioplatense, 2002) Sean a, b, c n umeros reales positivos. Muestre que a 1 + b+c 2 1 b + c+a 2 1 c + a+b 2 1.

Problema 3.62 (Rioplatense, 2002) Sean a, b, c n umeros reales positivos. Muestre que 9 1 1 1 a+b b+c c+a + 2 + 2 + + + . 2 c a b a+b+c a b c Problema 3.63 (Rusia, 2002) Muestre que x + y + z xy + yz + zx, para n umeros reales positivos x, y , z tales que x + y + z = 3. Problema 3.64 (APMO, 2002) Los n umeros reales positivos a, b, c satisfacen 1 1 1 + + = 1 . Muestre que a b c a + bc + b + ca + c + ab abc + a + b + c.

Problemas Recientes de Desigualdades

125

Problema 3.65 (Irlanda, 2003) Las longitudes a, b, c de los lados de un tri angulo cumplen que a + b + c = 2. Muestre que 1 ab + bc + ca abc 1 + 1 . 27

Problema 3.66 (Rumania, 2003) Muestre que en cualquier tri angulo ABC la siguiente desigualdad es verdadera 1 1 1 3 + + , mb mc mc ma ma mb S donde S es el area del tri angulo y ma , mb , mc son las longitudes de las medianas. Problema 3.67 (Rumania, 2003) Sean a, b, c, d n umeros reales positivos con abcd = 1. Muestre que 1 + ab 1 + bc 1 + cd 1 + da + + + 4. 1+a 1+b 1+c 1+d

Problema 3.68 (Rumania, 2003) En un tri angulo ABC , sean la , lb , lc las longitudes de las bisectrices, y sea s el semiper metro. Muestre que la + lb + lc 3s.

Problema 3.69 (Rusia, 2003) Sean a, b, c n umeros reales positivos con a + b + c = 1. Muestre que 1 1 1 2 2 2 + + + + . 1a 1b 1c 1+a 1+b 1+c Problema 3.70 (APMO, 2003) Muestre que (an + bn ) n + (bn + cn ) n + (cn + an ) n < 1 +
1 1 1

2n , 2

126

Problemas Recientes de Desigualdades

donde n > 1 es un entero y a, b, c son las longitudes de los lados de un tri angulo con per metro uno. Problema 3.71 (IMO, 2003) Dada n > 2 y n umeros reales x1 x2 xn , muestre que 2 |xi xj | (n2 1) 3 2 (xi xj )2 ,

i,j

i,j

donde la igualdad se da si y s olo si x1 , x2 , ... , xn forman una progresi on aritm etica. Problema 3.72 (Lista corta Iberoamericana, 2004) Si los n umeros positivos x1 , x2 ,..., xn satisfacen que x1 + x2 + . . . + xn = 1, muestre que x1 x2 xn n2 + + + . x2 (x1 + x2 + x3 ) x3 (x2 + x3 + x4 ) x1 (xn + x1 + x2 ) 3

Problema 3.73 (Rep ublicas Checa y Eslovaca, 2004) Sea P (x) = ax2 + bx + c un polinomio cuadr atico con coecientes reales no negativos. Muestre que, para cualquier n umero positivo x, P (x)P 1 x (P (1))2 .

Problema 3.74 (Croacia, 2004) Muestre que la desigualdad a2 b2 c2 3 + + (a + b)(a + c) (b + c)(b + a) (c + a)(c + b) 4 es v alida para todos los n umeros reales positivos a, b, c. Problema 3.75 (Estonia, 2004) Sean a, b, c n umeros reales positivos tales que a2 + b2 + c2 = 3. Muestre que 1 1 1 + + 1. 1 + 2ab 1 + 2bc 1 + 2ca

Problemas Recientes de Desigualdades

127

Problema 3.76 (Ir an, 2004) Sean x, y , z n umeros reales para los cuales xyz = 1. Muestre que x4 + y 4 + z 4 + 3(x + y + z ) x2 x2 y 2 y 2 z 2 z 2 + + + + + . y z x z x y

Problema 3.77 (Corea, 2004) Sean R y r el circunradio y el inradio del tri angulo acut angulo ABC , respectivamente. Suponga que A es el angulo mayor del tri angulo. Sea M el punto medio de BC y sea X la intersecci on de las tangentes al circunc rculo de ABC en B y C . Muestre que AM r . R AX

Problema 3.78 (Moldovia, 2004) Muestre que para cualesquiera n umeros reales a, b, c 0, la siguiente desigualdad se cumple a3 + b3 + c3 a2 bc + b2 ca + c2 ab.

Problema 3.79 (Ucrania, 2004) Sean x, y , z n umeros reales positivos tales que x + y + z = 1. Muestre que xy + z + yz + x + zx + y 1 + xy + yz + zx.

Problema 3.80 (Ucrania, 2004) Sean a, b, c n umeros reales positivos tales que abc 1. Muestre que a3 + b3 + c3 ab + bc + ca.

Problema 3.81 (Rumania, 2004) Encuentre todos los n umeros reales positivos a, b, c que satisfacen las desigualdades 4(ab + bc + ca) 1 a2 + b2 + c2 3(a3 + b3 + c3 ).

128

Problemas Recientes de Desigualdades

Problema 3.82 (Rumania, 2004) Los n umeros reales a, b, c satisfacen a2 + 2 2 b + c = 3. Muestre que |a| + |b| + |c| abc 4.

Problema 3.83 (Rumania, 2004) Considere el tri angulo ABC y sea O un punto interior de ABC . Las rectas OA, OB , OC intersectan los lados del tri angulo en A1 , B1 , C1 , respectivamente. Sean R1 , R2 , R3 los radios de los circunc rculos de los tri angulos OBC , OCA, OAB , respectivamente, y sea R el radio del circunc rculo del tri angulo ABC . Muestre que OB1 OC1 OA1 R1 + R2 + R3 R. AA1 BB1 CC1

Problema 3.84 (Rumania, 2004) Sea n 2 un n umero entero y sean a1 , a2 , ..., an n umeros reales. Muestre que para cualquier subconjunto no vac o S de {1, 2, ..., n}, la siguiente desigualdad se cumple
2

ai
iS

1ij n

(ai + + aj )2 .

Problema 3.85 (APMO, 2004) Sean a, b, c n umeros reales positivos. Muestre que (a2 + 2)(b2 + 2)(c2 + 2) 9(ab + bc + ca). Problema 3.86 (Lista corta IMO, 2004) Sean a, b y c n umeros reales positivos tales que ab + bc + ca = 1. Muestre que 3 3 1 3 + 6(a + b + c) . 3 abc abc Problema 3.87 (IMO, 2004) Sea n 3 un n umero entero. Sean t1 , t2 , . . . , tn n umeros reales positivos tales que n2 + 1 > (t1 + t2 + + tn ) 1 1 1 + + + t1 t2 tn .

Problemas Recientes de Desigualdades

129

Muestre que ti , tj , tk son las longitudes de los lados de un tri angulo, para todo i, j , k, con 1 i < j < k n. Problema 3.88 (Jap on, 2005) Sean a, b y c n umeros reales positivos, tales que a + b + c = 1. Muestre que 3 3 a 1 + b c + b 3 1 + c a + a 1 + a b 1.

Problema 3.89 (Rusia, 2005) Sean x1 , x2 , . . . , x6 n umeros reales tales que 2 + + x2 = 6 y x + x + + x = 0. Muestre que x x x 1 . + x x2 1 2 6 1 2 6 6 2 1 2 Problema 3.90 (Reino Unido, 2005) Sean a, b, c n umeros reales positivos. Muestre que a b c + + b c a
2

(a + b + c)

1 1 1 + + a b c

Problema 3.91 (APMO, 2005) Sean a, b y c n umeros reales positivos tales que abc = 8. Muestre que a2 (1 + a3 )(1 + b3 ) + b2 (1 + b3 )(1 + c3 ) + c2 (1 + c3 )(1 + a3 ) 4 . 3

Problema 3.92 (IMO, 2005) Sean x, y , z n umeros reales positivos tales que xyz 1. Muestre que x5 y5 y2 z5 z2 x5 x2 + 5 + 5 0. 2 2 2 2 +y +z y +z +x z + x2 + y 2

Problema 3.93 (Balcanes, 2006) Sean a, b, c n umeros reales positivos, muestre que 1 1 3 1 + + . a(b + 1) b(c + 1) c(a + 1) 1 + abc Problema 3.94 (Estonia, 2006) Sea O el circuncentro de un tri angulo acut angu lo ABC y sea A , B y C los circuncentros de los tri angulos BCO, CAO y

130

Problemas Recientes de Desigualdades

ABO, respectivamente. Muestre que el area del tri angulo ABC es menor o igual que el area del tri angulo A B C . Problema 3.95 (Lituania, 2006) Sean a, b, c n umeros reales positivos, muestre que 1 1 1 1 1 1 1 + + + + . a2 + bc b2 + ca c2 + ab 2 ab bc ca

Problema 3.96 (Turqu a, 2006) Sean a1 , a2 , . . . , an n umeros reales positivos tales que 2 2 a1 + a2 + + an = a2 1 + a2 + + an = A. Muestre que
i=j

(n 1)2 A ai . aj A1

Problema 3.97 (Iberoamericana, 2006) Considere n n umeros reales a1 , a2 , . . . , an , no necesariamente distintos. Sea d la diferencia entre el m aximo y el m nimo valor de los n umeros y sea s = i<j |ai aj |. Muestre que (n 1)d s n2 d . 4

Determine las condiciones en los n n umeros para que se cumpla la igualdad. Problema 3.98 (IMO, 2006) Determine el m nimo n umero real M tal que la desigualdad ab(a2 b2 ) + bc(b2 c2 ) + ca(c2 a2 ) M (a2 + b2 + c2 )2 , se cumpla, para cualesquiera n umeros reales a, b, c. Problema 3.99 (Bulgaria, 2007) Encuentre todos los enteros positivos n tal que si a, b, c son n umeros reales no negativos con a + b + c = 3, entonces abc(an + bn + cn ) 3.

Problemas Recientes de Desigualdades

131

Problema 3.100 (Bulgaria, 2007) Si a, b, c son n umeros reales positivos, muestre que (a + 1)(b + 1)2 (b + 1)(c + 1)2 (c + 1)(a + 1)2 + + a + b + c + 3. 3 3 3 3 c2 a2 + 1 3 a2 b2 + 1 3 b2 c2 + 1 Problema 3.101 (China, 2007) Si a, b, c son las longitudes de los lados de un tri angulo con a + b + c = 3, encuentre el m nimo de a2 + b2 + c2 + 4abc . 3

Problema 3.102 (Grecia, 2007) Si a, b, c son las longitudes de los lados de un tri angulo, muestre que (a + b c)4 (b + c a)4 (c + a b)4 + + ab + bc + ca. b(b + c a) c(c + a b) a(a + b c) Problema 3.103 (Ir an, 2007) Si a, b, c son tres n umeros reales positivos, muestre que a+b b+c c+a > 1. + + ab bc ca Problema 3.104 (Mediterranea, 2007) Sean x, y , z n umeros reales tales que 1 xy + yz + zx = 1. Muestre que xz < 2 . Es posible mejorar la cota de 1 2? Problema 3.105 (Mediterranea, 2007) Sea x > 1 un n umero real positivo que no sea un entero. Muestre que [x] x + {x} [x] x + {x} + x + [x] {x} {x} x + [x] > 9 , 2

donde [x] y {x} representan la parte entera y la parte fraccionaria de x, respectivamente. Problema 3.106 (Per u, 2007) Sean a, b, c n umeros reales positivos tales que 1 1 a+b+c a +1 + . Muestre que b c a+b+c 2 3 + . a + b + c abc

132

Problemas Recientes de Desigualdades

Problema 3.107 (Rumania, 2007) Sean a, b, c n umeros reales positivos tales que 1 1 1 + + 1. a+b+1 b+c+1 c+a+1 Muestre que a + b + c ab + bc + ca.

Problema 3.108 (Rumania, 2007) Sea ABC un tri angulo acut angulo con AB = AC . Para todo punto P en el interior del tri angulo ABC , considere la circunferencia con centro en A y radio AP ; sean M y N las intersecciones de los lados AB y AC con la circunferencia, respectivamente. Determine la posici on de P de tal forma que la suma M N + BP + CP sea m nima. Problema 3.109 (Rumania, 2007) Los puntos M , N , P en los lados BC , CA, AB del tri angulo ABC , respectivamente, son tales que el tri angulo M N P es acut angulo. Sea x la longitud de la altura menor del tri angulo ABC y X la longitud de la altura mayor del tri angulo M N P . Muestre que x 2X . Problema 3.110 (APMO, 2007) Sean x, y , z n umeros reales positivos tales que x + y + z = 1. Muestre que x2 + yz 2x2 (y + z ) + y 2 + zx 2y 2 (z + x) + z 2 + xy 2z 2 (x + y ) 1.

Problema 3.111 (B altica, 2008) Si los n umeros reales positivos a, b, c satisfacen que a2 + b2 + c2 = 3, muestre que b2 c2 (a + b + c)2 a2 + + . 2 + b + c2 2 + c + a2 2 + a + b2 12 Bajo que circunstancias se tiene la igualdad? Problema 3.112 (Canad a, 2008) Sean a, b, c n umeros reales positivos tales que a + b + c = 1. Muestre que 3 a bc b ca c ab + + . a + bc b + ca c + ab 2

Problemas Recientes de Desigualdades

133

Problema 3.113 (Ir an, 2008) Encuentre el menor n umero real K tal que, para cualesquiera n umeros reales positivos x, y , z , se cumple la siguiente desigualdad x y + y z + z x K (x + y )(y + z )(z + x).

Problema 3.114 (Irlanda, 2008) Si los n umeros reales positivos a, b, c, d 2 2 2 2 satisfacen que a + b + c + d = 1. Muestre que a2 b2 cd + ab2 c2 d + abc2 d2 + a2 bcd2 + a2 bc2 d + ab2 cd2 3 . 32

Problema 3.115 (Irlanda, 2008) Sean x, y , z n umeros reales positivos, tales que xyz 1. Muestre que: (a) 27 (1 + x + y )2 + (1 + y + z )2 + (1 + z + x)2 , (b) (1 + x + y )2 + (1 + y + z )2 + (1 + z + x)2 3(x + y + z )2 . La igualdad es cierta si y s olo si x = y = z = 1. Problema 3.116 (Rumania, 2008) Si a, b, c son n umeros reales positivos, tales que ab + bc + ca = 3, muestre que 1 1+ a2 (b + c) + 1 1+ b2 (c + a) + 1 1 + c2 (a + b) 1 . abc

Problema 3.117 (Rumania, 2008) Determine el m aximo valor para el n umero real k si 1 1 1 + + k k, (a + b + c) a+b b+c c+a para cualesquiera n umeros reales a, b, c 0, que cumplen adem as que a+b+c = ab + bc + ca. Problema 3.118 (Serbia, 2008) Sean a, b, c n umeros reales positivos tales que a + b + c = 1. Muestre que a2 + b2 + c2 + 3abc 4 . 9

134

Problemas Recientes de Desigualdades

Problema 3.119 (Vietnam, 2008) Sean x, y , z n umeros reales distintos y no-negativos. Muestre que 1 1 4 1 . + + 2 2 2 (x y ) (y z ) (z x) xy + yz + zx Cu ando se da la igualdad? Problema 3.120 (IMO, 2008) (i) Si x, y , z son tres n umeros reales distintos a 1 y tales que xyz = 1. Muestre que y2 z2 x2 + + 1. (x 1)2 (y 1)2 (z 1)2 (ii) Muestre que la igualdad es cierta para un n umero innito de n umeros racionales x, y , z .

Cap tulo 4

Soluciones a los Ejercicios y Problemas

En este cap tulo presentamos las soluciones o sugerencias de los ejercicios y problemas que aparecen en este libro. En las secciones 1 y 2 damos las soluciones a los ejercicios de los cap tulos 1 y 2, respectivamente. En la secci on 3 las soluciones a los problemas del cap tulo 3. Le recomendamos al lector que no consulte este cap tulo sin antes haber intentado resolver los ejercicios y problemas el mismo.

4.1.

Soluciones a los ejercicios del cap tulo 1

Soluci on 1.1 Se sigue de la denici on de a < b y la propiedad 1.1.1 para el n umero a b. Soluci on 1.2 (i) Si a < 0, entonces a > 0. Use tambi en que (a)(b) = ab. (ii) (a)b > 0. (iii) a < b b a > 0, use ahora la propiedad 1.1.2. (iv ) Use 1 la propiedad 1.1.2. (v ) Si a < 0, entonces a > 0. (vi) a a = 1 > 0. (vii) Si a < 0, entonces a > 0. (viii) Use (vi) y la propiedad 1.1.3. (ix) Muestre que ac < bc y que bc < bd. (x) Use la propiedad 1.1.3 con a 1 > 0 y a > 0. (xi) Use la propiedad 1.1.3 con 1 a > 0 y a > 0. Soluci on 1.3 (i) a2 < b2 b2 a2 = (b + a)(b a) > 0. (ii) Si b > 0, entonces 1 b > 0, ahora use el ejemplo 1.1.4 (ii).

136

Soluciones a los Ejercicios y Problemas

Soluci on 1.4 Para (i), (ii) y (iii) use la denici on, y para (iv ) y (v ) recuerde que |a|2 = a2 . Soluci on 1.5 (i) x |x| y x |x|. (ii) Considere |a| = |a b + b| y |b| = |b a + a| y aplique la desigualdad del tri angulo. (iii) (x2 + xy + y 2 )(x 3 3 2 2 3 y ) = x y . (iv ) (x xy + y )(x + y ) = x + y 3 . Soluci on 1.6 Si a, b o c son cero, tenemos la igualdad. Entonces, suponga que |a| |b| |c| > 0, ya que la desigualdad es sim etrica en a, b y c. Dividiendo entre |a|, la desigualdad es equivalente a 1+ c c c c b b b b + 1+ 0. + 1+ + 1+ + a a a a a a a a
b a

c b 1 y a 1, se tiene que 1 + Como a Entonces, es suciente probar que

= 1+

b a

y 1+

c a

c = 1+ a .

b b c b c c + 1+ + + a a a a a a

+ 1+

c b 0. + a a

Ahora, use la desigualdad del tri angulo y el ejercicio 1.5. Soluci on 1.7 (i) Use que 0 b 1 y 1 + a > 0, para ver que 0 b(1 + a) 1 + a 0 b a 1 ab 0 ba 1. 1 ab

(ii) La desigualdad de la izquierda es clara. Como 1 + a 1 + b, se tiene que 1 1 1+b 1+a , ahora vea que, b a b a+b a + + = 1. 1+b 1+a 1+a 1+a 1+a (iii) Para la desigualdad izquierda use que ab2 ba2 = ab(b a) es producto de n umeros reales no negativos. Para la desigualdad derecha note que, b 1 b2 b b b2 , ahora termine as , ab2 ba2 ab2 b2 a2 = b2 (a a2 ) a a2 = Soluci on 1.8 Muestre, m as generalmente, que x < 1 que x > 2 1 + 1+x < 2. 1 1 1 ( a)2 . 4 2 4 2 1+
1 1+x

>

2y

Soluci on 1.9 ax + by ay + bx (a b)(x y ) 0.

4.1 Soluciones a los ejercicios del cap tulo 1 Soluci on 1.10 Suponga que x y . Luego, utilice el ejercicio anterior con 1 1 y2, y y x. Soluci on 1.11 Observe que

137 x2 ,

(a b)(c d) + (a c)(b d) + (d a)(b c) = 2(a b)(c d) = 2(a b)2 0. Soluci on 1.12 Se sigue de f (a, c, b, d) f (a, b, c, d) = (a c)2 (a b)2 + (b d)2 (c d)2 = 2(b c)(a d) > 0. = (b c)(2a b c) + (b c)(b + c 2d)

f (a, b, c, d) f (a, b, d, c) = (b c)2 (b d)2 + (d a)2 (c a)2 = 2(d c)(b a) > 0.

= (d c)(2b c d) + (d c)(c + d 2a)

Soluci on 1.13 Para que las expresiones est en bien denidas es necesario que 2 1 + 2 x . y x = 0 . Multiplique el numerador y el denominador por 1 + x 1 2 Realice simplicaciones, para obtener 2 2x + 1 < 7; ahora resuelva para x. Soluci on 1.14 Como 4n2 < 4n2 + n < 4n2 + 4n + 1, se tiene que, 2n < 2 4n + n < 2n + 1. Luego, su parte entera es 2n y lo que hay que mostrar es 1 , que es inmediato al tomar cuadrados. entonces 4n2 + n < 2n + 4 Soluci on 1.15 Como (a3 b3 )(a2 b2 ) 0, se tiene que a5 + b5 a2 b2 (a + b), luego ab ab abc2 c = . = a5 + b5 + ab a2 b2 (a + b) + ab a2 b2 c2 (a + b) + abc2 a+b+c An alogamente, a5
bc b5 +c5 +bc

a a+b+c

ca c5 +a5 +ca

b a+b+c .

Por lo que,

ab bc ca c a b + 5 + 5 + + , 5 5 5 + b + ab b + c + bc c + a + ca a+b+c a+b+c a+b+c


c a+b+c

pero,

a a+b+c

b a+b+c

c+a+b a+b+c

= 1.

Soluci on 1.16 Considere p(x) = ax2 + bx + c, por hip otesis p(1) = a + b + c y b p(1) = a b + c son no negativos. Como a > 0, el m nimo de p se logra en 2a

138
2

Soluciones a los Ejercicios y Problemas

b b < 0. Si x1 , x2 son las ra ces, se tiene que a = (x1 + x2 ) y su valor es 4ac 4a c a+b+c ab+c y a = x1 x2 , por lo que a = (1 x1 )(1 x2 ), a = (1 + x1 )(1 + x2 ) c y a a = 1 x1 x2 . Observe que, (1 x1 )(1 x2 ) 0, (1 + x1 )(1 + x2 ) 0 y 1 x1 x2 0, garantizan que 1 x1 ,x2 1.

Soluci on 1.17 Si las tres desigualdades ocurren, debe suceder que a, b y c son 1 menores que 1, y que a(1 b)b(1 c)c(1 a) > 64 . Por otro lado, para 0 x 1 1 1 . siempre sucede que x(1 x) 4 , se tiene que a(1 b)b(1 c)c(1 a) 64 Soluci on 1.18 Use la desigualdad M G M A con a = 1, b = x.
1 . Soluci on 1.19 Use la desigualdad M G M A, con a = x y b = x

Soluci on 1.20 Use la desigualdad M G M A, con a = x2 y b = y 2 . Soluci on 1.21 En el ejercicio anterior sume x2 + y 2 de ambos lados.
x +y y Soluci on 1.22 Use la desigualdad M G M A con a = x+ x , b = y y use tambi en la M G M A, para x y y . O bien, reduzca al ejercicio 1.20. b Soluci on 1.23 Use la desigualdad M G M A, con ax y x .

Soluci on 1.24 Use la desigualdad M G M A, con Soluci on 1.25 0 < b a.


a+b 2 2 ( a b) ab = = 2 (ab)2 , 2( a+ b)2

a b

b y a .

simplique y acote usando

Soluci on 1.26 x + y 2 xy. Soluci on 1.27 x2 + y 2 2xy . Soluci on 1.28 xy + zx 2x yz . Soluci on 1.29 Vea el ejercicio 1.27. Soluci on 1.30 Soluci on 1.31 Soluci on 1.32
1 x

1 y

yz x

2 . xy

xy z

xy 2 z zx

= 2y .

x2 +(y 2 +z 2 ) 2

x y2 + z2 .

4.1 Soluciones a los ejercicios del cap tulo 1 Soluci on 1.33 x4 + y 4 + 8 = x4 + y 4 + 4 + 4 4 4 x4 y 4 16 = 8xy . Soluci on 1.34 (a + b + c + d) 4 4 a b c d,
a b b +c + c d 1 a

139

1 b

1 c

1 d

44

1 abcd.

Soluci on 1.35

d a

44

abcd bcda

= 4.
1 x1

Soluci on 1.36 (x1 + + xn ) n n x1 xn , Soluci on 1.37


a1 b1

+ +

1 xn

nn

1 x1 xn .

a2 b2

+ +

an bn

nn a

a1 an b1 bn

= n.

Soluci on 1.38 an 1 > n a na


n1 2

n+1 2 n1 2

n1 2

(a 1) an1 + + 1 >
1+a++an1 n

(a 1)

1+a++an1 n 1+a 2

>a

, pero +

>

(n1)n 2

=a

n1 2

Soluci on 1.39 1 =

1+b 2

1+c 2

a b c = abc.
a3 b a3 b

Soluci on 1.40 Aplicando la desigualdad M G M A, se tiene, 3


3

b3 c

a3 b

Por lo que,

b3 c

bc = 3ab. An alogamente,
3 2( ab

b3 c

c3 a

b3 c

c3 a)+

Segunda Soluci on. Tambi en se puede resolver aplicando el ejercicio 1.107. Soluci on 1.41 Si abc = 0, el resultado es claro. Si abc > 0, se tiene que ab bc ca 1 + + = c a b 2 a b c + c b +b c a +c + a c a b + b a

(ab + bc + ca) 3(ab + bc + ca).

+ ca 3bc y

c3 a

+ bc

+ ab 3ca.

1 (2a + 2b + 2c) 2 y el resultado se sigue. Soluci on 1.42 Aplique la desigualdad M G M A dos veces, a2 b + b2 c + c2 a 3abc, ab2 + bc2 + ca2 3abc. Soluci on 1.43
1+ab 1+a

abc+ab 1+a

= ab

1+c 1+a

. + bc 1+a 1+b + ca 1+b 1+c

1 + ab 1 + bc 1 + ca + + = ab 1+a 1+b 1+c 3


3

1+c 1+a

(abc)2 = 3.

140 Soluci on 1.44 que


1 a+b

Soluciones a los Ejercicios y Problemas +


1 b+c

1 c +a

(a + b + c) 9 2 , es equivalente a mostrar (a + b + b + c + c + a) 9,
1 a

1 1 1 + + a+b b+c c+a

lo cual se sigue del ejercicio 1.36. Para la otra desigualdad use Vea el ejercicio 1.22. Soluci on 1.45 Note que,
1 (1 + 1) + (1 + 1 n + Hn 2 ) + + (1 + n ) = . n n

1 b

4 a+b .

Ahora aplique la desigualdad M G M A.


1yi 1 1 Soluci on 1.46 Dena, yi = 1+ x i xi = y i 1 = yi . Ahora, observe que y1 + . . . + yn = 1 implica que 1 yi = j =i yj , luego, j =i yj

(n 1)

j =i y j

1 n1

y
j =i y j

xi =
i i

1 yi yi

i i

yi

(n 1)n

j =i y j

1 n1

yi
i 1ai ai ,

= (n1)n .

Soluci on 1.47 Dena an+1 = 1 (a1 + + an ) y xi = 1, ..., n + 1. Aplique directamente el ejercicio 1.46. Soluci on 1.48
n i=1 n 1 i=1 1+ai n

para i =

=1
n i=1

n ai i=1 1+ai n

= n 1. Observe que
n n

ai (n 1) =
i,j

i=1

1 = ai

ai aj = (1 + aj ) ai

i>j

1 1 ai ai ai 1 + ai 1 + ai i=1 i=1 i=1 ( ai aj 1)( ai aj )2 ( ai + aj ) . (1 + ai )(1 + aj ) ai aj

i j 1 1 Como 1 1+ ai + 1+aj = 1+ai +aj +ai aj , tenemos que ai aj 1. Por lo tanto, los t erminos de la u ltima suma son positivos.

2+a +a

Soluci on 1.49 Sean Sa =


n

a2 n i i=1 ai +bi

y Sb =
n i=1

b2 n i i=1 ai +bi . n

Sa Sb =

i=1

2 a2 i bi = ai + bi

ai

bi = 0,
i=1

4.1 Soluciones a los ejercicios del cap tulo 1 luego Sa = Sb = S. Se tiene entonces,
n

141

2S =
i=1

2 1 a2 i + bi ai + bi 2

n i=1

(ai + bi )2 = ai + bi

ai ,
i=1

de donde la desigualdad se sigue despu es de usar el ejercicio 1.21. Soluci on 1.50 Como la desigualdad es homog enea1 se puede suponer, sin p erdida de generalidad, que abc = 1. Haciendo x = a3 , y = b3 y z = c3 , la desigualdad es equivalente a, 1 1 1 + + 1. x+y+1 y+z+1 z+x+1 Sean A = x + y + 1, B = y + z + 1 y C = z + x + 1, luego 1 1 1 + + 1 (A 1)(B 1)(C 1) (A + B + C ) + 1 0 A B C (x + y )(y + z )(z + x) 2(x + y + z ) 2 (x + y + z )(xy + yz + zx 2) 3. Ahora, use que
1 2 x+y+z xy + yz + zx (xzy ) 3 y (xyz ) 3 . 3 3 Segunda Soluci on. Use las ideas de la soluci on del ejercicio 1.15. Inicie de que (a2 b2 )(a b) 0 para garantizar que a3 + b3 + abc ab(a + b + c), luego

a3

c 1 . 3 + b + abc abc(a + b + c)

Soluci on 1.51 Note que abc 1 +1 a 1 +1 b 1 +1 c

a+b+c 3 3

1 27 .

= 1+

1 1 1 1 1 1 1 + + + + + + a b c ab bc ca abc 3 3 1 1+ + + 3 abc 3 2 abc (abc) = 1 1+ 3 abc


3

43 .

Una funci on f (a, b, . . .) es homog enea si para t R se tiene que f (ta, tb, . . .) = tf (a, b, . . .). Luego, una desigualdad de la forma f (a, b, . . .) 0, para el caso de una funci on homog enea es equivalente a f (ta, tb, . . .) 0 para cualquier t > 0.

142

Soluciones a los Ejercicios y Problemas

b+c b+c c 8. Use Soluci on 1.52 La desigualdad es equivalente a b+ a b c ahora, la desigualdad M G M A para cada t ermino del producto, y la desigualdad se resuelve inmediatamente.

Soluci on 1.53 Note que, b c (a + 1)(b + 1)(c + 1) 2 a + + = (a + 1)(b + 1) (b + 1)(c + 1) (c + 1)(a + 1) (a + 1)(b + 1)(c + 1) 2 3 =1 , (a + 1)(b + 1)(c + 1) 4 que es equivalente a (a + 1)(b + 1)(c + 1) 8, y esta u ltima desigualdad es b+1 c+1 a+1 a b c = 1. inmediata de la desigualdad 2 2 2 Soluci on 1.54 Vea que es similar al ejercicio 1.52. Soluci on 1.55 Aplique la desigualdad entre la media aritm etica y la media arm onica para obtener 2ab = a+b
1 a

2 +

1 b

a+b . 2

Puede concluir tambi en que si la igualdad se alcanza entonces a = b = c. Soluci on 1.56 Utilice primero que (a + b)2 4ab, luego tome en cuenta que
n i=1

1 4 ai bi

n i=1

1 . (ai + bi )2

Ahora, use el ejercicio 1.36, para mostrar que


n n

(ai + bi )2
i=1 i=1

1 n2 . (ai + bi )2

Soluci on 1.57 Por la desigualdad M G M A, se tiene que xy + yz 2y xz . Sumando desigualdades similares se obtiene 2(xy +yz +zx) 2(x yz +y zx+ z xy ). Nuevamente, por M G M A, se tiene que x2 + x2 + y 2 + z 2 4x yz . Sumando los resultados similares, una vez m as, se llega a que x2 + y 2 + z 2 x yz + y zx + z xy . Ahora, sumando ambas desigualdades, se obtiene el 2 resultado (x+y3+z ) x yz + y zx + z xy .

4.1 Soluciones a los ejercicios del cap tulo 1

143

Soluci on 1.58 La desigualdad M G M A lleva a que x4 + y 4 2x2 y 2 . Usando 2 2 2 M G M A nuevamente se tiene que 2x y + z 8xyz . O bien, directamente se tiene que 4 4 4 z2 z2 4 x y z x4 + y 4 + + 4 = 8xyz. 2 2 4 Soluci on 1.59 Por M G M A, se tiene x2 y2 + 2 y1 x1 La u ltima desigualdad se deduce de xy (x 1)(y 1) 8.

Segunda Soluci on. Sean a = x1, b = y 1, que son n umeros positivos, enton2 2 ces la desigualdad que se quiere demostrar es equivalente a (a+1) + (b+1) 8. b a Ahora bien, por M G M A, tenemos que (a + 1)2 4a y (b + 1)2 4b. Luego, 2 (a+1)2 b + (b+1) 4 a ltima desigualdad es consecuencia del b a b + a 8. La u ejercicio 1.24. Soluci on 1.60 Observe que (a, b, c) y (a2 , b2 , c2 ) se ordenan de la misma forma, use la desigualdad (1.2). Soluci on 1.61 Por el ejercicio anterior a3 + b3 + c3 a2 b + b2 c + c2 a.
1 1 1 1 1 1 Observe que ( a , b, c) y (a 2 , b2 , c2 ) se ordenan de la misma forma. Entonces, usando la desigualdad (1.2), se obtiene

x x 1

2, ya que (x 2)2 0.

(ab)3 + (bc)3 + (ca)3 =

1 1 1 + + a3 b3 c3 1 1 1 1 1 1 + 2 + 2 2 a c b a c b a b c = + + c a b = a2 b + b2 c + c2 a.

Sumando las dos desigualdades, se obtiene el resultado. Soluci on 1.62 Use la desigualdad (1.2) con (a1 , a2 , a3 ) = (b1 , b2 , b3 ) = a b c b c a b , c , a y (a1 , a2 , a3 ) = c , a , b .

144

Soluciones a los Ejercicios y Problemas

Soluci on 1.63 Use la desigualdad (1.2) con (a1 , a2 , a3 ) = (b1 , b2 , b3 ) = 1 1 1 1 1 1 , , a b c y (a1 , a2 , a3 ) = b , c , a . Soluci on 1.64 Suponga que a b c, y considere (a1 , a2 , a3 ) = (a, b, c). Use la desigualdad del reacomodo (1.2) dos veces con (a 1 , a2 , a3 ) = (b, c, a) y (c, a, b), respectivamente. Desde luego, se est a utilizando (b1 , b2 , b3 ) = 1 1 1 , , b+ca c+ab a+bc .

Soluci on 1.65 Utilice la misma idea que en el ejercicio anterior, pero con n variables. Soluci on 1.66 Use el ejercicio anterior y el hecho que
s sa1

=1+

a1 sa1 .

Soluci on 1.67 Aplique el ejercicio 1.65 a la sucesi on a1 , . . ., an , a1 , . . ., an . Soluci on 1.68 Aplique el ejemplo 1.4.11. Soluci on 1.69 Note que 1 = (a2 + b2 + c2 ) + 2(ab + bc + ca), y use el ejercicio anterior como sigue a+b+c 1 = 3 3 por lo tanto, evidente.
1 3

a2 + b2 + c2 3

2 a2 + b2 + c2 . Luego, 2(ab + bc + ca) 3 , y el resultado es

Segunda Soluci on. La desigualdad es equivalente a 3(ab + bc+ ca) (a + b + c)2 pero esta se reduce a ab + bc + ca a2 + b2 + c2 . Soluci on 1.70 Sea G = n x1 x2 xn la media geom etrica de los n umeros x1 x2 xn 1 x1 x2 , , , . dados y (a1 , a2 , ..., an ) = x n 2 G G G Utilice el corolario 1.4.2, se tiene que n luego,
1 x1

an1 an G G G G a1 a2 + + + + = + + + + , a2 a3 an a1 x2 x3 xn x1 n + + G.

1 x2

1 xn

4.1 Soluciones a los ejercicios del cap tulo 1 Tambi en, por el corolario 1.4.2, n entonces a2 an x1 x2 xn a1 + + + = + + + , an a1 an1 G G G G

145

x1 + x2 + + xn . n Las igualdades ocurren si y s olo si a1 = a2 = = an , es decir, si y s olo si x1 = x2 = = xn . Soluci on 1.71 La desigualdad es equivalente a
n1 n1 n1 a1 + a2 + + an

a1 an a1 an a1 an + + + , a1 a2 an
n i=1 1

la cual se verica usando la desigualdad del reacomodo varias veces.


ai Soluci on 1.72 Primero note que n i=1 1ai = Por la desigualdad M G M A se obtiene n 1 i=1 1ai

ai .

1 n

n i=1

1 1 ai

n
n

i=1

1 = 1 ai 1 ai ) =

1
n

n i=1 (1

ai )

1 n

n i=1 (1

n . n1

M as a un, la desigualdad de Cauchy-Schwarz sirve para mostrar que


n i=1

1 ai

n i=1

(1 ai ) n =

n(n 1)

y
i=1

ai

n.

1 < 4a+1+1 Soluci on 1.73 (i) 4a + + 1. (ii) Use la desigualdad de 2 = 2a Cauchy-Schwarz con u = ( 4a + 1, 4b + 1, 4c + 1) y v = (1, 1, 1). Soluci on 1.74 Suponga que a b c d (los otros casos son an alogos). Entonces, si A = b + c + d, B = a + c + d, C = a + b + d y D = a + b + c, se 1 1 1 1 B C D . Aplique la desigualdad de Tchebyshev dos veces tiene que A para mostrar que a3 b3 c3 d3 1 + + + (a3 + b3 + c3 + d3 ) A B C D 4 1 1 1 1 + + + A B C D

146

Soluciones a los Ejercicios y Problemas

1 2 (a + b2 + c2 + d2 )(a + b + c + d) 16 1 2 (a + b2 + c2 + d2 ) 16

1 1 1 1 + + + A B C D 1 1 1 1 + + + A B C D .

A+B+C +D 3

Ahora, use la desigualdad de Cauchy-Schwarz para ver que a2 + b2 + c2 + d2 ab + bc + cd + da = 1


1 + y use la desigualdad (A + B + C + D)( A 1 B

1 C

1 D)

16.

Soluci on 1.75 Use la desigualdad del reacomodo con a b c a 3 3 , (b1 , b2 , b3 ) = , 3 (a1 , a2 , a3 ) = 3 , b c a b


y permutaci on (a 1 , a2 , a3 ) =
3

b c

c a

b c,

c a,

a b

, para obtener b2 + ca c2 . ab

a b c + + b c a Finalmente use que abc = 1.

a2 + bc

Segunda Soluci on. Aplique la desigualdad entre M G M A de la siguiente manera: 2 1 a a b 3 3 a 3 aab + + = = a3 = a. 3 b b c bbc bc An alogamente, desigualdades.
1 3 b c

+b c +

c a

by

1 3

c a

c a

a b

c. Ahora sume las tres

Soluci on 1.76 Por hip otesis, para toda k, se tiene que s 2xk > 0. Por la desigualdad de Cauchy-Schwarz
n k =1

x2 k s 2xk

n k =1

(s 2xk )

xk
k =1

= s2 .

4.1 Soluciones a los ejercicios del cap tulo 1 Pero 0 <


n k =1 (s

147

2xk ) = ns 2s, por lo que


n k =1

x2 s k . s 2xk n2
1 2 x

Soluci on 1.77 La funci on f (x) = x +

es convexa en R+ .

b c Soluci on 1.78 La funci on f (a, b, c) = b+a c+1 + a+c+1 + a+b+1 + (1 a)(1 b)(1 c) es convexa en cada una de las variables, luego su m aximo se alcanza en los extremos.

Soluci on 1.79 Si x = 0, entonces la desigualdad se reduce a 1 + 1

1+y 2

que es verdadera ya que y 0. Por simetr a, la desigualdad es cierta para y = 0. Suponga ahora que 0 < x 1 y 0 < y 1. Sea u 0 y v 0 tales que x = eu y y = ev , entonces la desigualdad se convierte en 1 1 + 1 + e2v 1 + e2u esto es, f (u) + f (v ) f 2 2 1 + e(u+v) , ;

2,

u+v 2
2x

12e donde f (x) = 1+1 , la funci on es c oncava en . Como f (x) = e4x (1+ e2x )5/2 e2x el intervalo [0, ). Por lo tanto, la desigualdad anterior es verdadera.

Soluci on 1.80 Encuentre f (x). Soluci on 1.81 Utilice log(sen x) o bien sen A sen B = sen A+B AB + 2 2 sen A+B AB 2 2 .

Soluci on 1.82 (i) Si 1+ nx 0, la desigualdad es evidente ya que (1+ x)n 0. Suponga que (1+nx) > 0. Aplique M GM A a los n umeros (1, 1, . . . , 1, 1+nx) con (n 1) unos. (ii) Sean a1 , . . ., an n umeros positivos y dena, para cada j = 1, . . . n, j =
a1 +...+aj . Aplique la desigualdad j j j j1 (j 1), lo que implica j j j j 1 j

de Bernoulli para mostrar que

j j 1

j (j 1) j 1

j 1 j 1 = j 1 (jj (j 1)j 1 ) = aj j 1 .

148

Soluciones a los Ejercicios y Problemas

n2 n a n1 a a Luego, n n n1 n n1 n2 an an1 a1 .

Soluci on 1.83 Si x y z , se tiene que xn (x y )(x z ) y n (x y )(y z ) n y z (z x)(z y ) 0. Soluci on 1.84 Note que x(x z )2 + y (y z )2 (x z )(y z )(x + y z ) 0 si y s olo si x(xz )(xy )+y (y z )(y x)+z (xz )(y z ) 0. La desigualdad ahora se sigue de la desigualdad de Sch ur. O bien, la u ltima expresi on es sim etrica en x, y y z , por lo que se puede suponer x z y , ahora regrese a la desigualdad original, donde claramente x(x z )2 + y (y z )2 0 (x z )(y z )(x + y z ). Soluci on 1.85 La desigualdad es homog enea, por lo que se puede suponer que x a + b + c = 1. Ahora los sumandos de la izquierda son de la forma (1 x)2 ,
x 4+2x es convexa, ya que f (x) = (1 (1x)2 x)4 b c a+b+c a = 3f Jensen, (1a)2 + (1b)2 + (1c)2 3f 3 1 3

y la funci on f (x) = desigualdad de

> 0. Por la =
3 2 2 .

Soluci on 1.86 Como (a + b + c)2 3(ab + bc + ca), se puede deducir que 3 9 1 + ab+bc a verdadera si +ca 1 + (a+b+c)2 . Entonces, la desigualdad ser 1+ 6 9 . (a + b + c)2 (a + b + c)
3 a+b+c z= 1 c; 2

1 ,y= 1 se sigue inmediataAhora bien, si abc = 1, considere x = a b y mente que xyz = 1. Luego, la desigualdad es equivalente a

Pero esta u ltima desigualdad se sigue de que 1

0.

1+

3 6 xy + yz + zx x+y+z

que es la primera parte del ejercicio. Soluci on 1.87 Utilice la convexidad de la funci on f (x) = xr , para r 1 r 2 (su segunda derivada es r (r 1)x ). Primero suponga que r > s > 0. La r desigualdad de Jensen para funciones convexas f (x) = x s aplicada a xs 1, . . . , dice que xs n r s s r s w1 xr 1 + + wn xn (w1 x1 + + wn xn )

esima potencia de ambos lados de la desigualdad se obtiene la y tomando la 1 r - desigualdad deseada.

4.1 Soluciones a los ejercicios del cap tulo 1


r

149

Ahora, suponga que 0 > r > s. Entonces f (x) = x s es c oncava, es decir, la desigualdad de Jensen es v alida en el otro sentido. Sin embargo, si toma la esima la desigualdad se invierte nuevamente. potencia 1 r - r Finalmente, en el caso r > 0 > s, f (x) = x s es nuevamente convexa, y esima potencia en ambos lados de la desigualdad, esta se preserva. tomando la 1 r -
c Soluci on 1.88 (i) Aplique la desigualdad de H older a los n umeros xc 1 , . . . , xn , c , . . . , y c con a = a and b = b . y1 n c c (ii) Para probar esto siga la demostraci on del ejemplo 1.5.9. Lo u nico que tiene 1 b 1 c 1 a que probar adem as es que xi yi zi a xi + b yi + c zi , pero esto se sigue de la parte (i) del mismo ejemplo.

Soluci on 1.89 Por la simetr a de las variable en la desigualdad se puede suponer b+c b+c y (ii) b a+3 . que a b c. Se tienen dos casos, (i) b a+3 a+b+c Caso (i) b 3 . b+c c b+c c Se tiene que a+3 a+ c, y luego es cierto que a+3 b+ 2 2 c. Entonces, existen , [0, 1] tales que c+a = c + (1 ) 2 a+b+c 3 and b+c = c + (1 ) 2 a+b+c 3 .

Sumando estas igualdades, se obtiene que a + b + 2c = (+)c+(2) 2 Luego, a + b 2c = (2 ) 2 a + b 2c 3 . a+b+c 3 = (2) a + b 2c +2c. 3

1 Por lo tanto, 2 ( + ) = 3 2 y ( + ) = 2 . Ahora bien, como f es una funci on convexa, se tiene que

f f f

a+b 2 b+c 2 c+a 2

1 (f (a) + f (b)) 2 a+b+c 3 a+b+c 3

f (c) + (1 )f f (c) + (1 )f

150

Soluciones a los Ejercicios y Problemas

entonces, sumando estas desigualdades se obtiene que f a+b 2 +f b+c 2 +f c+a 2 1 (f (a) + f (b) + f (c)) 2 3 a+b+c + f . 2 3
a+c 2

b+c . Caso (ii) b a+3 Es an alogo al caso (i), utilizando el hecho que a a+b+c 3 .

a+b+c 3

ya

a+b 2

Soluci on 1.90 Si alguna de las variables a, b o c es cero, la desigualdad es evidente. Aplicando la desigualdad de Popovicius, al ejercicio anterior, y utilizando la funci on f : R R+ denida por f (x) = exp(2x), que es convexa ya que f (x) = 4 exp(2x) > 0, se obtiene que exp(2x) + exp(2y )+ exp(2z ) + 3 exp( 2(x + y + z ) ) 3 2 [exp(x + y ) + exp(y + z ) + exp(z + x)] =

=2 [exp(x) exp(y ) + exp(y ) exp(z ) + exp(z ) exp(x)] . Deniendo a = exp(x), b = exp(y ), c = exp(z ), se puede reescribir la desigualdad anterior como 3 a2 + b2 + c2 + 3 a2 b2 c2 2(ab + bc + ca). Para la segunda parte aplique la desigualdad AM GM de la siguiente forma 3 2abc + 1 = abc + abc + 1 3 a2 b2 c2 . Soluci on 1.91 Aplicando la desigualdad de Popoviciu a la funci on convexa 1 1 9 4 4 4 1 se obtiene la desigualdad, a +1 + + + f (x) = x + x b c a+b+c b+c c+a + a+b . Ahora, multiplique ambos lados de la desigualdad por (a + b + c) para terminar la demostraci on. Soluci on 1.92 Observe que por la ecuaci on (1.8), se obtiene 1 1 1 x2 + y 2 + z 2 |x||y | |y ||z | |z ||x| = (|x| |y |)2 + (|y | |z |)2 + (|z | |x|)2 2 2 2 lo cual es claramente mayor o igual a cero. Por lo tanto, |xy + yz + zx| |x||y | + |y ||z | + |z ||x| x2 + y 2 + z 2 .

4.1 Soluciones a los ejercicios del cap tulo 1

151

Segunda Soluci on. Aplique la desigualdad de Cauchy-Schwarz con (x, y, z ) y (y, z, x). Soluci on 1.93 La desigualdad es equivalente a tener ab + bc + ca a2 + b2 + c2 , la cual se sabe que es verdadera. Vea el ejercicio 1.27. Soluci on 1.94 Observe que, si a + b + c = 0, entonces se sigue de la ecuaci on 3 3 3 (1.7) que a + b + c = 3abc. Como (x y ) + (y z ) + (z x) = 0, se obtiene la factorizaci on (x y )3 + (y z )3 + (z x)3 = 3(x y )(y z )(z x). Soluci on 1.95 Suponga, sin p erdida de generalidad, que a b c. Entonces, necesita demostrar que a3 + b3 + c3 + 3abc 0. Como a3 + b3 + c3 + 3abc = (a)3 + b3 + c3 3(a)bc, factorice, la u ltima expresi on, como 1 (a + b + c)((a + b)2 + (a + c)2 + (b c)2 ). 2 La conclusi on se sigue de la desigualdad del tri angulo, b + c > a. Soluci on 1.96 Sea p = |(x y )(y z )(z x)|. Utilizando la desigualdad M G M A del lado derecho de la identidad (1.8), se tiene que x2 + y 2 + z 2 xy yz zx 3 2
3

p2 .

(4.1)

Ahora bien, como |x y | x + y , |y z | y + z , |z x| z + x, se tiene que 2(x + y + z ) |x y | + |y z | + |z x|. Aplicando nuevamente la desigualdad M G M A, se obtiene que 2(x + y + z ) 3 3 p, (4.2)

y el resultado se sigue de las desigualdades (4.1) y (4.2).

152

Soluciones a los Ejercicios y Problemas

Soluci on 1.97 Utilice la identidad (1.7), factorice, la condici on x3 + y 3 + z 3 3xyz = 1, y se obtiene (x + y + z )(x2 + y 2 + z 2 xy yz zx) = 1. (4.3)

Sean A = x2 + y 2 + z 2 y B = x + y + z . Observe que B 2 A = 2(xy + yz + zx). Por la identidad (1.8), se tiene que B > 0. La ecuaci on 4.3 se transforma en B A luego 3A = B 2 + obtener
2 B.

B2 A 2

= 1,

Como B > 0, aplique la desigualdad M G M A, para

1 1 2 = B2 + + 3, B B B es decir, A 1. El m nimo A = 1 se alcanza, por ejemplo, con (x, y, z ) = (1, 0, 0). 3A = B 2 +

Soluci on 1.98 Por la desigualdad (1.11), se tiene que 1 1 4 16 (1 + 1 + 2 + 4)2 64 + + + = . a b c d a+b+c+d a+b+c+d Soluci on 1.99 Utilice la desigualdad (1.11) dos veces para obtener
b) 2 ( (a+ (a2 + b2 )2 (a + b)4 a4 b4 2 ) + = . a +b = 1 1 2 2 8 4 4
2

Soluci on 1.100 Escriba el lado izquierdo como ( 2)2 ( 2)2 ( 2)2 + + x+y y+z z+x y use la desigualdad (1.11). Soluci on 1.101 Escriba el lado izquierdo como x2 y2 z2 + + axy + bzx ayz + bxy azx + byz y ahora aplicando la desigualdad (1.11), se tiene y2 z2 (x + y + z )2 3 x2 + + , axy + bzx ayz + bxy azx + byz (a + b)(xy + zx + yz ) a+b

4.1 Soluciones a los ejercicios del cap tulo 1 donde la u ltima desigualdad se sigue de la ecuaci on (1.8). Soluci on 1.102 Escriba el lado izquierdo como b2 c2 b2 c2 a2 a2 + + + + + , a+b b+c c+a a+b b+c c+a luego use la desigualdad (1.11). Soluci on 1.103 (i) Escriba el lado izquierdo como x2 y2 z2 + + x2 + 2xy + 3zx y 2 + 2yz + 3xy z 2 + 2zx + 3yz y use la desigualdad (1.11) para obtener

153

x y z (x + y + z )2 + + 2 . x + 2y + 3z y + 2z + 3x z + 2x + 3y x + y 2 + z 2 + 5(xy + zx + yz ) Ahora, s olo falta probar la desigualdad 1 (x + y + z )2 , x2 + y 2 + z 2 + 5(xy + zx + yz ) 2 pero esta es equivalente a x2 + y 2 + z 2 xy + zx + yz . (ii) Como en la parte (i), escriba el lado izquierdo como w2 x2 y2 z2 + + + xw + 2yw + 3zw xy + 2xz + 3xw yz + 2yw + 3xy zw + 2xz + 3yz entonces, use la desigualdad (1.11) para obtener x y z w + + + x + 2y + 3z y + 2z + 3w z + 2w + 3x w + 2x + 3y (w + x + y + z )2 . 4(wx + xy + yz + zw + wy + xz ) Luego, la desigualdad que tiene que probar es (w + x + y + z )2 2 , 4(wx + xy + yz + zw + wy + xz ) 3 que es equivalente a 3(w2 + x2 + y 2 + z 2 ) 2(wx + xy + yz + zw + wy + xz ). Pero esta se deduce de la desigualdad M G M A aplicada seis veces en la forma x2 + y 2 2xy .

154

Soluciones a los Ejercicios y Problemas

Soluci on 1.104 Use la desigualdad (1.11) para obtener y2 z2 x2 + + (x + y )(x + z ) (y + z )(y + x) (z + x)(z + y ) (x + y + z )2 . x2 + y 2 + z 2 + 3(xy + yz + zx)

Tambi en, la desigualdad 3 (x + y + z )2 2 2 2 x + y + z + 3(xy + yz + zx) 4 es equivalente a x2 + y 2 + z 2 xy + yz + zx. Soluci on 1.105 Escriba el lado izquierdo como a2 b2 c2 d2 + + + a(b + c) b(c + d) c(d + a) d(a + b) y use la desigualdad (1.11) para obtener a2 b2 c2 d2 (a + b + c + d)2 + + + a(b + c) b(c + d) c(d + a) d(a + b) a(b + 2c + d) + b(c + d) + d(b + c) Observe, por otro lado, que (a + b + c + d)2 = (ac + bd) + (ab + ac + ad + bc + bd + cd) = a2 + b2 + c2 + d2 + 2ab + 2ac + 2ad + 2bc + 2bd + 2cd (ac + bd) + (ab + ac + ad + bc + bd + cd)

Demostrar que la u ltima expresi on es mayor que 2, es equivalente a demostrar 2 2 2 2 que a + c 2ac y b + d 2bd, las cuales son inmediatas por la desigualdad M G M A. Soluci on 1.106 Escriba el lado izquierdo como a2 b2 c2 d2 e2 + + + + ab + ac bc + bd cd + ce de + ad ae + be y use la desigualdad (1.11) para obtener a2 b2 c2 d2 e2 (a + b + c + d + e)2 + + + + . ab + ac bc + bd cd + ce de + ad ae + be ab

4.1 Soluciones a los ejercicios del cap tulo 1 Como (a + b + c + d + e)2 = muestre que 2 lo cual es equivalente a 2 La u ltima desigualdad se sigue de a2 a2 ab. ab. a2 + 4 ab 5 ab, a2 + 2 ab,

155

Soluci on 1.107 (i) Use la desigualdad de Tchebyshev con los n umeros (a b2 c2 a2 b c) y ( x y z ) para tener que 1 3 a3 b3 c3 + + x y z
a2 x

b2 y

c2 z

a+b+c , 3

entonces, por la desigualdad (1.11), se tiene que (a + b + c)2 a2 b2 c2 + + . x y z x+y+z Por lo tanto, (a + b + c)2 a + b + c a3 b3 c3 + + . x y z x+y+z 3
1 3

(ii) Por el ejercicio 1.88, se tiene que a3 b3 c3 + + x y z (1 + 1 + 1) 3 (x + y + z ) 3 a + b + c.


1 1

Elevando al cubo ambos lados de la desigualdad y dividiendo ambos lados por 3(x + y + z ) se obtiene el resultado. Soluci on 1.108 Use la desigualdad u til (1.11) y obtendr a que
2 x2 x2 x2 n 1 + + xn 1 = + + x1 + + x n x1 + + x n x1 + + xn

x1 + + x n (x1 + + xn )2 = . n(x1 + + xn ) n
kn t

Luego, es suciente probar que x1 + + xn n x1 x n .

156

Soluciones a los Ejercicios y Problemas

Como k = m ax {x1 , . . . , xn } min {x1 , . . . , xn } = t, se tiene que kn t n x1 ++xn y como 1 , ya que todos los x son enteros positivos, es suciente i n probar que x1 + + x n n x1 x n , n que es equivalente a la desigualdad M G M A. Como todas las desigualdades intermedias que se han usado son desigualdades v alidas cuando x1 = = xn se concluye que esto sucede en este caso. Soluci on 1.109 Con la sustituci on a = x y, b = la desigualdad como,
y z z yc= x , se puede reescribir

b3 c3 a3 + + 1, a3 + 2 b3 + 2 c3 + 2 con la condici on extra que, abc = 1. Para probar esta u ltima desigualdad se puede usar la condici on extra como sigue a3 b3 c3 + 3 + 3 +2 b +2 c +2 = = a3 b3 c3 + 3 + 3 + 2abc b + 2abc c + 2abc a2 b2 c2 + + a2 + 2bc b2 + 2ca c2 + 2ab (a + b + c)2 = 1. a2 + b2 + c2 + 2bc + 2ca + 2ab a3

a3

Y la desigualdad anterior se deduce de la desigualdad (1.11).


b c Soluci on 1.110 Con la sustituci on x = a b , y = c , z = a , la desigualdad toma la forma b c 3 a + + . b+c c+a a+b 2

Y esta u ltima desigualdad es la desigualdad de Nesbitt, ejemplo 1.4.8.


a1 a2 3 , x2 = a Soluci on 1.111 Use la sustituci on x1 = a a2 , . . ., xn = an . Como 1 a1 1 1 alogamente para los otros sumandos 1+x1 +x1 x2 = 1+ a2 + a2 a3 = a1 +a2 +a3 y an

del lado izquierdo de la desigualdad. Por lo que se tiene que la desigualdad es ahora equivalente a a2 an a1 + + + > 1. a1 + a2 + a3 a2 + a3 + a4 an + a1 + a2

a1

a1 a2

4.1 Soluciones a los ejercicios del cap tulo 1

157

Pero esta desigualdad es ahora evidente. Unicamente observe que para toda i = 1, ..., n se cumple que ai + ai+1 + ai+2 < a1 + a2 + + an .
1 1 , y = 1 on Soluci on 1.112 Con la sustituci on x = a b y z = c , la condici ab + bc + ca = abc, se transforma en x + y + z = 1 y la desigualdad original es equivalente a

x4 + y 4 y 4 + z 4 z 4 + x4 + + 1 = x + y + z. x3 + y 3 y 3 + z 3 z 3 + x3 Por la desigualdad de Tchebyshev, se garantiza que x4 + y 4 x3 + y 3 x + y , 2 2 2 por lo que, x4 + y 4 y 4 + z 4 z 4 + x4 x+y y+z z+x + + . + 3 + 3 3 3 3 3 x +y y +z z +x 2 2 2

Soluci on 1.113 La desigualdad de la izquierda se sigue de aplicar la desigualdad ca ab (1.11). Para la desigualdad de la derecha, la sustituci on, x = bc a,y = b ,z = c nos permite reescribir la desigualdad como x+y+z yz + zx + xy . 3 3 Elevando al cuadrado ambos lados y se obtiene que 3(xy + yz + zx) (x + y + z )2 , la cual es v alida si y s olo si (xy + yz + zx) x2 + y 2 + z 2 , pero esta desigualdad es ya conocida. Soluci on 1.114 Note que a2 b2 c2 + + 0 33 a+1 b+1 c+1 1 1 1 + + a+1 b+1 c+1 1 1 1 + + . 1 a+1 b+1 c+1 0

158 Utilizando la sustituci on, a =


2x y ,

Soluciones a los Ejercicios y Problemas b=


2x y 2y z ,

c=

2z x,

se obtiene

1 1 1 + + = a+1 b+1 c+1

1 1 1 + 2y + 2z +1 x +1 z +1 z x y + + = 2x + y 2y + z 2z + x z2 x2 y2 + + = 2 2 2xy + y 2yz + z 2zx + x2 (x + y + z )2 = 1. 2xy + y 2 + 2yz + z 2 + 2zx + x2

Para la u nica desigualdad que aparece en este c alculo, se aplic o la desigualdad (1.11). Soluci on 1.115 Observe que 2 2 [5, 0, 0] = (a5 + b5 + c5 ) (a3 bc + b3 ca + c3 ab) = [3, 1, 1], 6 6 donde se us o el teorema de Muirhead. Soluci on 1.116 Utilizando la f ormula de Her on para el area de un tri angulo, podemos reescribir la desigualdad como sigue a2 + b2 + c2 4 3 Pero, esta es equivalente a (a2 + b2 + c2 )2 3[((a + b)2 c2 )(c2 (b a)2 )] (a + b + c) (a + b c) (a + c b) (b + c a) . 2 2 2 2

= 3(2c2 a2 + 2c2 b2 + 2a2 b2 (a4 + b4 + c4 )),

es decir, a4 + b4 + c4 a2 b2 + b2 c2 + c2 a2 , que en t erminos del teorema de Muirhead es equivalente a probar que [4, 0, 0] [2, 2, 0]. Segunda Soluci on. Utilizando la sustituci on x = a + b c, y = a b + c, z = a + b + c, obtenemos que x + y + z = a + b + c; entonces, utilizando la f ormula de Her on, tenemos 4(ABC ) = (a + b + c)(xyz ) (a + b + c) (a + b + c)2 (x + y + z )3 . = 27 3 3

4.1 Soluciones a los ejercicios del cap tulo 1

159

Ahora, s olo se tiene que probar que (a + b + c)2 3(a2 + b2 + c2 ). Aplicando, el teorema de Muirhead, se deduce esta desigualdad, ya que [1, 1, 0] [2, 0, 0]. Soluci on 1.117 Note que, b c 9 a + + (a + b)(a + c) (b + c)(b + a) (c + a)(c + b) 4(a + b + c) 8(ab + bc + ca)(a + b + c) 9(a + b)(b + c)(c + a) 24abc + 8 (a2 b + ab2 ) 9 (a2 b + ab2 ) + 18abc

6abc a2 b + ab2 + b2 c + bc2 + c2 a + ca2 [1, 1, 1] [2, 1, 0] Soluci on 1.118 La desigualdad es equivalente a a3 + b3 + c3 ab(a + b c) + bc(b + c a) + ca(c + a b). Dena x = a + b c, y = b + c a, z = a + c b, para obtener a = y y +z b = x+ 2 , c = 2 . Entonces, la desigualdad que se tiene que probar es
x +z 2 ,

1 1 ((x+y )3 +(y +z )3 +(z +x)3 ) ((x+y )(x+z )x+(x+y )(y +z )y +(x+z )(y +z )z ), 8 4 que es equivalente a 3(x2 y +y 2 x+y 2 z +z 2 y +z 2 x+x2 z ) 2(x2 y +y 2 x+y 2 z +z 2 y +z 2 x+x2 z )+6xyz o x2 y + y 2 x + y 2 z + z 2 y + z 2 x + x2 z 6xyz y aplicando el teorema de Muirhead se obtiene el resultado cuando x, y , z son no-negativos. Si uno de ellos es negativo (y no puede ser m as de uno al mismo tiempo), se tiene que x2 (y + z ) + y 2 (x + z ) + z 2 (x + y ) = x2 2c + y 2 2a + z 2 2b 0 pero 6xyz es negativo, lo cual concluye la prueba. Soluci on 1.119 Observe que b3 c3 a3 + + a+b+c b2 bc + c2 c2 ca + a2 a2 ab + b2

160 que es equivalente a la desigualdad

Soluciones a los Ejercicios y Problemas

a3 (b + c) b3 (c + a) c3 (a + b) + 3 + 3 a + b + c, b3 + c3 c + a3 a + b3 que a su vez es equivalente a a3 (b+c)(a3 +c3 )(a3 +b3 )+b3 (c+a)(b3 +c3 )(a3 +b3 )+c3 (a+b)(a3 +c3 )(b3 +c3 ) (a + b + c)(a3 + b3 )(b3 + c3 )(c3 + a3 ). Se puede reescribir esta u ltima desigualdad en t erminos del teorema de Muirhead como [9, 1, 0] + [6, 4, 0] + [6, 3, 1] + [4, 3, 3] = 1 1 [1, 0, 0] [6, 3, 0] + [3, 3, 3] 2 3 [7, 3, 0] + [6, 4, 0] + [6, 3, 1] + [4, 3, 3]

[9, 1, 0] [7, 3, 0] lo cual es evidente utilizando Muirhead. Soluci on 1.120 Suponga que a b c, luego 1 1 1 . (1 + b) (1 + c) (1 + c) (1 + a) (1 + a) (1 + b) Use Tchebyshev para ver que b3 c3 a3 + + (1 + b) (1 + c) (1 + a) (1 + c) (1 + a) (1 + b) 1 1 1 1 (a3 + b3 + c3 ) + + 3 (1 + b)(1 + c) (1 + a)(1 + c) (1 + a)(1 + b) 1 3 + (a + b + c) = (a3 + b3 + c3 ) . 3 (1 + a)(1 + b)(1 + c)
1 3 b+c 3 Finalmente, use que 3 (a + b3 + c3 ) ( a+3 ) , 3+a+b+c 3 3 a+b+c 3

c)

para ver que

1 y (1 + a)(1 + b)(1 +
3

1 3 3 + (a + b + c) (a + b3 + c3 ) 3 (1 + a)(1 + b)(1 + c) Para la u ltima desigualdad, observe que

a+b+c 3
a+b+c 3 b+c + a+3

6 (1 +
a+b+c 3 3 )

6 . 8

1 . 2

4.2 Soluciones a los ejercicios del cap tulo 2

161

Segunda Soluci on. Multiplicando por el denominador com un y desarrollando ambos lados, la desigualdad deseada es equivalente a a(a4 + b4 + c4 + a3 + b3 + c3 ) 3(1 + a + b + c + ab + bc + ca + abc). Como 4(a4 + b4 + c4 + a3 + b3 + c3 ) = 4(3[4, 0, 0] + 3[3, 0, 0]) y 3(1 + a + b + c + ab + bc + ca + abc) = 3([0, 0, 0] + 3[1, 0, 0] + 3[1, 1, 0] + [1, 1, 1]), la desigualdad es equivalente a 4[4, 0, 0] + 4[3, 0, 0] [0, 0, 0] + 3[1, 0, 0] + 3[1, 1, 0] + [1, 1, 1]. Ahora, observemos que [4, 0, 0]
4 4 4 4 4 4 , , = a 3 b 3 c 3 = 1 = [0, 0, 0], 3 3 3

donde tenemos que abc = 1. Tambi en 1 1 3[4, 0, 0] 3[2, 1, 1] = 3 (a2 bc + b2 ca + c2 ab) = 3 (a + b + c) = 3[1, 0, 0] 3 3 y 3[3, 0, 0] 3 4 4 1 , , 3 3 3
4 4 1 4 4 1 1 4 4 1 = 3 (a 3 b 3 c 3 + b 3 c 3 a 3 + c 3 a 3 b 3 3 1 = 3 (ab + bc + ca) = 3[1, 0, 0]. 3

Finalmente, [3, 0, 0] [1, 1, 1]. Sumando estas desigualdades, obtenemos la desigualdad deseada.

4.2.

Soluciones a los ejercicios del cap tulo 2

Soluci on 2.2 (i) c < a + b c < a + b +2 ab = ( a+ b)2 c < a + b. (ii) Con 2, 3 y 4 se puede construir un tri angulo y con 4, 9 y 16 no es posible construirlo.

Soluci on 2.1 (i) Trace un segmento BC de longitud a, con centro en B un c rculo de radio c y con centro en C un c rculo de radio b, Bajo qu e circunstancias se intersectan? (ii) Se sigue de (i). bc a c b , y = a+2 , z = b+c (iii) a = x + y , b = y + z , c = z + x x = a+2 2 .

162

Soluciones a los Ejercicios y Problemas

1 1 1 (iii) a < b < c a + b < a + c < b + c b+ c < c+a < a+b , por lo 1 1 1 que ser a suciente que ver que a+b < b+c + c+a , pero es m as f acil ver que 1 1 1 c < b+c + c+a .

Soluci on 2.3 Utilice el hecho de que si a, b, c son las longitudes de los lados de un tri angulo, el angulo que se opone al lado de longitud c es o bien recto, agudo u obtuso dependiendo si c2 es igual, menor o mayor a a2 + b2 , respectivamente. Suponga entonces que a b c d e, y que las ternas de segmentos de longitud (a, b, c) y (c, d, e) no forman un tri angulo acut angulo. Luego, como c2 a2 + b2 y e2 c2 + d2 , se puede concluir que e2 a2 + b2 + d2 a2 + b2 + c2 a2 + b2 + a2 + b2 = (a + b)2 + (a b)2 (a + b)2 , por lo cual a + b e, lo que nos lleva a una contradicci on. Soluci on 2.4 Como A > B entonces BC > CA. Utilizando la desigualdad del tri angulo AB < BC + CA y la armaci on anterior se tiene que AB < 2BC . Soluci on 2.5 (i) Sea O el punto de intersecci on de las diagonales AC y BD. Aplique la desigualdad del tri angulo a los tri angulos ABO y CDO. Sumando las desigualdades, tenemos AB + CD < AC + BD. Por otro lado, por hip otesis se tiene que AB + BD < AC + CD. Sumando estas dos u ltimas desigualdades obtiene que AB < AC . (ii) Sea DE paralela a BC ; entonces EDA < BCD < A, luego DE > 1 1 2 AD . Por lo tanto, 2 AD < DE < BC , vea el ejercicio anterior. Soluci on 2.6 Cada di es menor que la suma de la longitud de dos lados. Use tambi en el hecho que, en un cuadril atero convexo la suma de la longitud de dos lados opuestos es menor que la suma de la longitud de las diagonales. Soluci on 2.7 Aplique la desigualdad del tri angulo a los tri angulos ABA y 1 1 AA C para mostrar que c < ma + 2 a y b < ma + 2 a. Soluci on 2.8 Si , , son los angulos del tri angulo en A, B y C , respectivamente, y si 1 = BAA y 2 = A AC entonces, por D2, > 1 y > 2 . Luego, 180o = + + > 1 + 2 + = 2. O bien, si trazamos un c rculo de di ametro BC , A deber a estar fuera del c rculo y entonces BAC < 90 . Soluci on 2.9 Construya un paralelogramo ABDC , con una diagonal BC y otra AD la cual es igual al doble de AA y use la desigualdad D2, en el tri angulo ABD. Soluci on 2.10 Complete un paralelogramo como en la soluci on anterior para c +a a+b b+c alogamente, mb < 2 y mc < 2 . Para probar la mostrar que ma < 2 . An

4.2 Soluciones a los ejercicios del cap tulo 2

163

desigualdad de la izquierda, sean A , B , C los puntos medios de los lados BC , CA y AB , respectivamente. A B

Prolongue el segmento C B hasta un punto A de manera que C A = BC . Aplique el resultado anterior al tri angulo AA A cuyos lados tienen longitud ma , mb y mc . Soluci on 2.11 Considere el cuadril atero ABCD y sea O un punto en el exterior del cuadril atero tal que AOB sea semejante a ACD, entonces tambi en OAC y BAD son semejantes. Si O, B y C son colineales se tiene la igualdad, si no son colineales se tiene una desigualdad2 . Soluci on 2.12 Sean a = AB , b = BC , c = CD, d = DA, m = AC y n = BD. Sea R el radio del circunc rculo de ABCD. Se tiene que3 m(ab + cd) , 4R n(bc + ad) (ABCD) = (BCD) + (DAB ) = , 4R (ABCD) = (ABC ) + (CDA) = donde (ABCD) denota el area del cuadril atero ABCD. Por lo tanto, bc + ad m = >1 n ab + cd bc + ad > ab + cd (d b)(a c) > 0.

Soluci on 2.13 Con centro en A haga una rotaci on de 60 del tri angulo ABP. El punto B se transforma en C y sea P el transformado de P. El tri angulo P P C tiene lados P P = P A, P C = P B y P C , como el que se desea.
2 3

Ver [6], p ag. 136 o [1], p ag. 128. Ver [6], p ag. 97 o [9], p ag. 13.

164 A P P B C

Soluciones a los Ejercicios y Problemas

C P

Segunda Soluci on. Use la desigualdad de Ptolomeo (ejercicio 2.11) en los cuadril ateros ABCP , ABP C y AP BC ; despu es de cancelar factores comunes tendr a que: P B < P C + P A, P A < P C + P B y P C < P A + P B , respectivamente. Esto garantiza la existencia del tri angulo. Tercera Soluci on. Otra soluci on para el caso en que P este dentro de ABC. Sea P el punto donde AP corta al lado BC. Use ahora que, AP < AP < AB = BC < P B + P C. An alogamente obtenga las desigualdades P B < P C + P A y P C < P A + P B.

Soluci on 2.14 Sean a = AB , b = BC , x = AC , y = BD. Recuerde que en un paralelogramo se cumple la identidad 2(a2 + b2 ) = x2 + y 2 . Suponga adem as, sin perder generalidad, que a b. Es claro que 2b < (x + y ), por lo que (2b)2 < (x + y )2 = x2 + y 2 + 2xy = 2(a2 + b2 ) + 2xy . Luego, reduciendo se obtiene que 2(b2 a2 ) < 2xy . Soluci on 2.15 (i) Prolongue las medianas AA , BB y CC hasta que corten al circunc rculo en A1 , B1 y C1 , respectivamente. Use la potencia de A para 2 en, use el hecho de que ma + A A1 2R y establecer que A A1 = 4a ma . Tambi
2 2 2

2(b +c )a 2 , es decir, 4m2 que la longitud de la mediana satisface m2 a+a = a = 4 2 2 2(b + c ). Hay expresiones an alogas para mb y mc . (ii) Use la desigualdad de Ptolomeo en los cuadril ateros AC GB , BA GC y CB GA , donde G denota el centroide. Por ejemplo, para el primer cuadril atero 2 b mc c mb 2 ab m + ac m . se obtiene que 3 ma a + , entonces 2 m a a c b 2 2 3 2 3 2 2 2 2 Soluci on 2.16 Use la f ormula 4m2 b + b = 2(c + a ), para ver que mb 3 2 2 2 angulo pruebe que mc = 4 (c b ). Ahora bien, usando la desigualdad del tri 3 mb + mc < 2 (b + c). De aqu puede deducir la desigualdad de la izquierda.

4.2 Soluciones a los ejercicios del cap tulo 2

165

La desigualdad de la derecha se puede deducir de la primera cuando se aplica al tri angulo de lados4 de longitud ma , mb y mc . Soluci on 2.17 Sean a, b y c la longitud de los lados del tri angulo ABC . Si E y F son las proyecciones de Ia sobre las rectas AB y CA, respectivamente, es claro que, si ra es el radio del exc rculo, se tiene que ra = Ia E = EA = AF = F Ia = s, donde s es el semiper metro de ABC. Adem as, si ha es la altura del ha AD = . Como ah tri angulo ABC desde A, entonces DI a = bc, se tiene que ra a AD ha bc = = = DIa ra as abc 4R 4Rr a2 1 rs = 4Rr , a2

donde r y R son el inradio y circunradio de ABC , respectivamente. a c AD = b+c = b+ a que Como 2R = a y 2r = b + c a, entonces DI a a 1. Bastar a b+c 2 2 vea que a 2 o, equivalentemente, que 2bc a , pero bc = b c2 2 b2 +c2 =a 2 2 . Soluci on 2.18 Simplicando y utilizando el ejercicio 1.27, la primera desigualdad es equivalente a ab + bc + ca a2 + b2 + c2 . Para la segunda desarrolle (a + b + c)2 y utilizando la desigualdad del tri angulo se obtiene que a2 < a(b + c). Soluci on 2.19 Utilice la sugerencia anterior. Soluci on 2.20 Si desarrolla la expresi on regresar a al ejercicio anterior. Soluci on 2.21 La primera desigualdad es la desigualdad de Nesbitt, ejemplo b+c , entonces 1.4.8. Para la segunda desigualdad use el hecho de que a + b > a+2 2c c < . a+b a+b+c Soluci on 2.22 Observe que a2 (b + c a) + b2 (c + a b) + c2 (a + b c) 2abc = (b + c a) (c + a b) (a + b c); ahora vea el ejemplo 2.2.3. Soluci on 2.23 Observe que a b2 + c2 a2 +b c2 + a2 b2 + c a2 + b2 c2 = vea ahora el ejercicio 2.22.
4

a2 (b + c a) + b2 (c + a b) + c2 (a + b c) ,

Ver la soluci on del ejercicio 2.10.

166

Soluciones a los Ejercicios y Problemas

Soluci on 2.24 Use la transformaci on de Ravi con a = y + z , b = z + x, c = x + y para ver primero que a2 b(a b)+ b2 c(b c)+ c2 a(c a) = 2(xy 3 + yz 3 + zx3 ) 2(xy 2 z + x2 yz + xyz 2 ). Luego, la desigualdad es equivalente a desigualdad (1.11). Soluci on 2.25 ab bc ca + + a+b b+c c+a = < ab bc ca a+b b+c c+a cab 1 , (a + b)(b + c)(c + a) 8
x2 y

y2 z

+ zx x + y + z . Ahora, use la

para la u ltima desigualdad, vea la soluci on del ejemplo 2.2.3. Soluci on 2.26 Por el ejercicio 2.18 3(ab + bc + ca) (a + b + c)2 4(ab + bc + ca). Entonces, como ab + bc + ca = 3, se sigue que 9 (a + b + c)2 12, de donde se tiene el resultado. Soluci on 2.27 Use la transformaci on de Ravi, a = y + z, b = z + x, c = x + y. Por la desigualdad M G M A, y la desigualdad de Cauchy-Schwarz, 1 1 1 + + a b c = 1 1 1 + + y+z z+x x+y

1 1 1 1 + + 2 yz xy zx x+ y+ z = 2 xyz 3 x+y+z 2 xyz 3 3 x+y+z = . = 2 xyz 2r

Para la u ltima identidad vea el nal de la demostraci on del ejemplo 2.2.4.

4.2 Soluciones a los ejercicios del cap tulo 2 Soluci on 2.28 (i) Se sigue de las equivalencias siguientes, (s a)(s b) < ab s2 s(a + b) < 0 a + b + c < 2(a + b) c < a + b.

167

(ii) Use la transformaci on de Ravi, a = y + z , b = z + x, c = x + y , para ver que la desigualdad es equivalente a, 4(xy + yz + zx) (y + z )(z + x) + (z + x)(x + y ) + (x + y )(y + z ), y para justicar la u ltima desigualdad basta ver que, xy + yz + zx x2 + y 2 + z 2 , que se sabe del ejercicio 1.27. Otra manera de obtener (ii). La desigualdad es equivalente a, 3s2 2s(a + b + c) + (ab + bc + ca) ab + bc + ca . 4

Y esta a su vez es equivalente a 3(ab + bc + ca) 4s2 , que se reescribe como 3(ab + bc + ca) (a + b + c)2 , la cual se sigue tambi en del ejercicio 1.27. Soluci on 2.29 La ley de los cosenos ayuda a ver que 2ab cos C 2ac cos B a2 + b2 c2 a2 b2 + c2 = = 2a

(b cos C )(c cos B ) b cos C + c cos B = a2 . 2a 2

Soluci on 2.30 Por la desigualdad de Cauchy-Schwarz, para cualesquiera x, y , z , w 0, se tiene que xy + zw (x + z )(y + w). De donde, a2 + b2 c2 a2 b2 + c2 = + 1 2 1 2 a2 + b2 c2 a2 b2 + c2

c clica

c clica

c2 + a2 b2
c clica

c2 a2 + b2 ac.
c clica

(2a2 )(2c2 ) =

168

Soluciones a los Ejercicios y Problemas

Soluci on 2.31 Considere n umeros positivos x, y , z , con a = y + z , b = z + x y c = x + y . Las desigualdades son equivalentes a demostrar que y+z z+x x+y + + 3 2x 2y 2z Para la primera desigualdad use que desigualdad de Nesbitt. y
y x

2x 2y 2z + + 3. y+z z+x x+y +


x y

2 y para la segunda use la

Soluci on 2.32 Como en los tri angulos con la misma base la raz on de sus alturas es igual a la raz on de sus areas, se tiene que, PS (P BC ) (P CA) (P AB ) (ABC ) PQ PR + + = + + = = 1. AD BE CF (ABC ) (ABC ) (ABC ) (ABC ) Utilice la desigualdad (2.3) de la secci on 2.3.
1 Soluci on 2.33 (i) Recuerde que (S1 + S2 + S3 )( S + 1 1 S2

1 S3 )

(ii) Los v ertices de los tri angulos forman un hex agono que se divide en 6 tri angulos de areas S1 , S2 , S3 , T1 , T2 , T3 , donde los tri angulos de areas Si y Ti tienen un angulo com un. Utilice la f ormula de area que involucra al seno del angulo para demostrar que S1 S2 S3 = T1 T2 T3 . Use despu es la desigualdad M G M A, como sigue S 1 1 1 + + S1 S2 S3 (S1 + S2 + S3 + T1 + T2 + T3 ) 18 6 S1 S2 S3 T1 T2 T3 = 18. 3 S1 S2 S3 1 1 1 + + S1 S2 S3

9.

La igualdad se da cuando el punto O es el centroide del tri angulo y las l neas que pasan por O son las medianas del tri angulo, es decir, en el caso en que S . S1 = S2 = S3 = T1 = T2 = T3 = 1 6 Soluci on 2.34 Si P = G es el centroide, la igualdad es clara, ya que AG GL = BG CG GM = GN = 2. BP CP AL BM CN Por otro lado, si AP P L + P M + P N = 6, se tiene que P L + P M + P N = 9. (P BC ) P M (P CA) (P AB ) L PN No es dif cil ver que P AL = (ABC ) , BM = (ABC ) y CN = (ABC ) , por lo que
PL AL

PM BM

PN CN

= 1. Lo que lleva a que PL PM PN + + AL BM CN = 9.

AL BM CN + + PL PM PN

4.2 Soluciones a los ejercicios del cap tulo 2

169

Por la desigualdad (2.3), se sabe que esta igualdad ocurre solamente cuando BM CN AL P L = P M = P N = 3, lo que garantiza que P sea el centroide. Soluci on 2.35 (i) Como HD = DD , HE = EE y HF = F F , donde H es el ortocentro5 . Luego, la soluci on se sigue de la parte (i) del ejemplo 2.3.4.
AD +DD = 1 + HD en, al ver la soluci on del (ii) Como AD AD = AD AD , se tiene tambi BE CF HD HE HF AD ejemplo 2.3.4, que AD + BE + CF = 1 + AD + 1 + BE + 1 + CF = 4.

Como

AD AD

BE BE

CF CF

AD AD

BE BE

CF CF

9 se sigue el resultado.

Soluci on 2.36 Como se se nal o en el ejemplo 2.3.5, la longitud de la bisectriz interna del angulo A cumple
2 la = bc 1

a b+c

4bc (s(s a)). (b + c)2

2 s(s a), y l l s (s a)(s b) Como 4bc (b + c)2 , se tiene que la a b (sa)+(sb) c s = s 2 . Por lo tanto, la lb lc s s(s a)(s b)(s c) = s(sr ), 2 b+c 2 +l2 +l2 s(sa)+s(sb)+s(sc) = s2 . la lb +lb lc +lc la s a+2 = s2 , y la c b

Soluci on 2.37 Sean = AM B , = BN A, = AP C , y sea (ABC ) el area. Se tiene que abc 1 . (ABC ) = a AM sen = 2 4R De donde, Luego,
bc AM

= 2R sen . An alogamente,

ca BN

= 2R sen y

ab CP

= 2R sen .

bc ca ab + + = 2R(sen + sen + sen ) 6R. AM BN CP La igualdad se alcanza si M , N y P son los pies de las alturas. Soluci on 2.38 Sean A1 , B1 , C1 los puntos medios de los lados BC , CA, AB , respectivamente, y sean B2 , C2 las reecciones de A1 con respecto a AB y CA, respectivamente. Tambi en considere a D la intersecci on de AB con A1 B2 y E la intersecci on de CA con A1 C2 . Entonces, 2DE = B2 C2 C2 B1 + B1 C1 + C1 B2 = A1 B1 + B1 C1 + C1 A1 = s. Use que A1 DAE es un cuadril atero inscrito en una circunferencia de di ametro AA1 y use la ley de senos en ADE , para deducir que DE = AA1 sen A =
5

Consultar [6], p ag. 85 o [9], p ag. 37.

170

Soluciones a los Ejercicios y Problemas

ama ma sen A. Entonces, s 2DE = 2ma sen A = 2ma 2a R = R , es decir, ama sR. An alogamente, puede vericar que bmb sR y cmc sR.

Soluci on 2.39 La desigualdad es equivalente a 8(s a)(s b)(s c) abc, donde s es el semiper metro. Como (ABC ) = sr = abc s(s a)(s b)(s c), donde R y r denotan el 4R = circunradio y el inradio, respectivamente, se tiene que demostrar solamente que 8sr 2 abc. Es decir, que 8sr 2 4Rrs, que es equivalente a 2r R. Soluci on 2.40 El area del tri angulo ABC , satisface que (ABC ) = abc 4R = (a+b+c)r 1 1 1 1 1 , luego, ab + bc + ca = 2Rr R2 , donde R y r denotan el circunradio 2 y el inradio, respectivamente. Soluci on 2.41 Use el ejercicio 2.40 y la ley de los senos.
(sb)(sc) Soluci on 2.42 Use que6 sen A , donde s es el semiper metro. 2 = bc B C Expresiones semejantes para sen 2 y sen 2 , para ver que

sen

B C (s a)(s b)(s c) sr 2 r 1 A sen sen = = = , 2 2 2 abc abc 4R 8

donde R y r denotan el circunradio y el inradio, respectivamente. Soluci on 2.43 Sabe por la desigualdad (2.3), que (a + b + c) 1 1 1 + + a b c 9.

Como a + b + c 3 3R, se tiene que

1 1 1 3 + + . a b c R 3 3 = . 2 + B + C)

(4.4)

Usando nuevamente la desigualdad (2.3), se obtiene 1 + + 3 2A 2B 2C


Sea f (x) = log 2 x , como f (x) = de Jensen, 1 x2

2 (A

(4.5)

> 0, f es convexa. Usando la desigualdad 1 + + 3 2A 2B 2C

1 log + log + log 3 2A 2B 2C


6

log
=

sen2

A 2

1cos A 2

2 c2 a2 1 b +2 bc 2

a2 (bc)2 4bc

(sb)(sc) . bc

4.2 Soluciones a los ejercicios del cap tulo 2 Usando (4.5) y el hecho que log x es estrictamente creciente, se tiene que 1 log + log + log 3 2A 2B 2C 3 log . 2
1 b

171

(4.6)
1 c

1 Suponga que a b c, lo cual implica A B C . Entonces a log 2A log 2B log 2C . Usando la desigualdad de Tchebyshev,

1 1 1 log + log + log a 2A b 2B c 2C Entonces, por (4.4) y (4.6), se tiene

1 1 1 + + a b c

log

2A

+ log 2 B + log 2C 3

1 1 3 1 3 log + log + log log . a 2A b 2B c 2C R 2 Elevando a las potencias adecuadas y tomando rec procos, obtiene la desigualdad deseada. En todas las desigualdades anteriores, la igualdad se tiene si y s olo si a = b = c (esto es, la igualdad se tiene s olo para tri angulos equil ateros). Soluci on 2.44 Por la ley de los senos, se tiene que sen B sen C 1 sen A = = = , a b c 2R donde a, b, c son las longitudes de los lados del tri angulo y R es el circunradio. Entonces, sen2 A + sen2 B + sen2 C = = b2 c2 a2 + + 4R2 4R2 4R2 1 (a2 + b2 + c2 ) 4R2 1 9 9R2 = , 2 4R 4

donde la desigualdad se sigue de la desigualdad de Leibniz. Soluci on 2.45 Use la desigualdad de Leibniz y el hecho de que el area de un abc tri angulo est a dada por (ABC ) = 4R . Soluci on 2.46 Note que el inc rculo de ABC es el circunc rculo de DEF . Aplicando la desigualdad de Leibniz a DEF , se obtiene EF 2 + F D 2 + DE 2 9r 2 ,

172

Soluciones a los Ejercicios y Problemas

donde r es el inradio de ABC . Por otra parte, usando el teorema 2.4.3 se obtiene que s2 27r 2 , de donde EF 2 + F D 2 + DE 2 Soluci on 2.47 a2 b2 c2 + + hb hc hc ha ha hb = = a2 bc + b2 ca + c2 ab abc(a + b + c) = 2 4(ABC ) 4(ABC )2 2R abc(a + b + c) = 4. abc (a+b+c)r r 4
4R 2 A 2

s2 . 3

Soluci on 2.48 Recuerde que sen2 3 (vea el ejemplo 2.5.2). cos C 2 Soluci on 2.49 Observe que, 4 3(ABC )

1cos A 2

y use que cos A + cos B +

9abc 9 4 Rrs 2s 4 3rs 2 3s 9R R. a+b+c 2s 3 3

La u ltima desigualdad la se demostr o en el teorema 2.4.3. Soluci on 2.50 Use el ejercicio anterior y la desigualdad entre la media arm onica y la media geom etrica 3
1 ab

1 bc

1 ca

a2 b2 c2 .

Soluci on 2.51 Use el ejercicio anterior y la desigualdad M G M A 3 a2 b2 c2 a2 + b2 + c2 . 3


a+b+c 2

Soluci on 2.52 Primero observe que si s =

entonces

a2 + b2 + c2 (a b)2 (b c)2 (c a)2 =

= 4{(s b)(s c) + (s c)(s a) + (s a)(s b)}.

= a2 (b c)2 + b2 (c a)2 + c2 (a b)2

4.2 Soluciones a los ejercicios del cap tulo 2

173

Por lo que, si x = s a, y = s b, z = s c, entonces la desigualdad es equivalente a 3 xyz (x + y + z ) xy + yz + zx. Elevando al cuadrado y reduciendo t erminos, se tiene que la desigualdad anterior es equivalente a xyz (x + y + z ) x2 y 2 + y 2 z 2 + z 2 x2 . Deduzca esta u ltima de la desigualdad de Cauchy-Schwarz con (xy, yz, zx) y (zx, xy, yz ). Soluci on 2.53 Use el ejercicio 2.50 y la desigualdad 3 3 (ab)(bc)(ca) ab + bc + ca. Soluci on 2.54 Note que 3(a + b + c)abc 9abc ab + bc + ca a+b+c (a + b + c)2 3(ab + bc + ca) a2 + b2 + c2 ab + bc + ca, ahora use el ejercicio 2.49. Soluci on 2.55 Utilice (2.5), (2.6) y (2.7), observe que a2 + b2 + c2 + 4abc = 1 2 2 2r . Soluci on 2.56 Utilice las relaciones usadas en la soluci on al ejercicio 2.39 (b + c a)(c + a b)(a + b c) abc = = = 8(s a)(s b)(s c) abc 8s(s a)(s b)(s c) 4Rs( abc 4R ) 2r 8(rs)2 = . 4Rs(rs) R

174 Soluci on 2.57 Observe que

Soluciones a los Ejercicios y Problemas

b2 c2 1 a2 + + = b+ca c+ab a+bc 2 = = = = 1 2 s 2

sb sc sa a+ b+ c sa sb sc b c a + + sa sb sc s

b2 c2 a2 + + sa sb sc

s 2s3 2s(s2 + r 2 + 4rR) + 3(4Rrs) s 2 r2s 2s(R R 2s(R r ) 3 3rR 2) = = 3 3R, r r r

s (a + b + c)s2 2(ab + bc + ca)s + 3abc s 2 (s a)(s b)(s c)

las dos u ltimas desigualdades se derivan del hecho que R 2r (lo que implica que r 2R ) y que s 3 3r , respectivamente. Soluci on 2.58 Debe partir del lado de las ecuaciones donde se encuentra la relaci on entre las s y realice las operaciones. Soluci on 2.59 Si x1 , 1 x1 , x2 , 1 x2 , . . ., son las longitudes en que queda dividido cada lado por el punto correspondiente, se tiene que a2 + b2 + c2 + d2 = 1 1 2 1 2 2 2 (x2 i + (1 xi ) ). Muestre que 2 2(xi 2 ) + 2 = xi + (1 xi ) 1. Para la parte (ii), la desigualdad de la derecha se sigue de la desigualdad del tri angulo. Para la desigualdad de la izquierda use reexiones en los lados, como en la siguiente gura.

d b c

4.2 Soluciones a los ejercicios del cap tulo 2 Soluci on 2.60 Esta es igual a (ii) del problema anterior.

175

Soluci on 2.61 Si ABC es el tri angulo y DEF GHI es el hex agono con DE , F G, HI paralelos a BC , AB , CA, respectivamente, se tiene que el per metro del hex agono es 2(DE + F G + HI ). Sean X , Y , Z los puntos de tangencia del inc rculo con los lados BC , CA, AB , respectivamente, y sea p = a + b + c el per metro del tri angulo ABC . Dena x = AZ = AY , y = BZ = BX y z = CX = CY , se tiene entonces la relaci on AE + ED + DA 2x DE = = . a p p An alogamente, se tienen las otras dos relaciones 2z FG = , c p Luego, p(DEF GHI ) = 4(xa + yb + zc) p 4(a(s a) + b(s b) + c(s c)) 2s 4((a + b + c)s (a2 + b2 + c2 )) = 2s (a2 + b2 + c2 ) = 2(a + b + c) 4 (a + b + c) = HI 2y = . b p

1 (a + b + c)(a + b + c) por la desigualdad de Tchebyshev. pero, a2 + b2 + c2 3 2 Por lo tanto, p(DEF GHI ) 2(a + b + c) 4 3 (a + b + c) = 3 (a + b + c).

Soluci on 2.62 Considere el circunc rculo del tri angulo equil atero con lados de longitud 2. Los c rculos con centro en los puntos medios de los lados del tri angulo y radio 1 cubren al c rculo de radio 2. Si un c rculo de radio mayor que 2 3 rculos de radio 1, entonces uno de los tres c rculos 3 es cubierto por tres c cubre una cuerda de longitud mayor a 2. Soluci on 2.63 Tome el tri angulo acut angulo de lados de longitud 2r1 , 2r2 y 2r3 , si existe. Su circunradio es el buscado. Si no existe el tri angulo, la soluci on es el mayor radio entre r1 , r2 y r3 . Soluci on 2.64 Lema 1. Si un cuadrado de lado de longitud a est a dentro del rect angulo de lados de longitud c y d, entonces a m n {c, d}. Demostraci on.

176

Soluciones a los Ejercicios y Problemas

d a c Por los v ertices del cuadrado dibujamos rectas paralelas a los lados del rect angulo de tal manera que encierren al cuadrado como se muestra en la gura. Como las rectas paralelas forman un cuadrado dentro del rect angulo se tiene el resultado. Lema 2. La longitud de la diagonal del cuadrado inscrito en un tri angulo es menor o igual a la longitud de la bisectriz interna del angulo recto. Demostraci on. Sea ABC un tri angulo rect angulo con hipotenusa CA y sea P QRS el cuadrado inscrito. A

S T P
O V O

Podemos suponer que los v ertices P y Q pertenecen a los catetos del tri angulo rect angulo (en caso de no ser as , transladamos el cuadrado) y sea O la interesecci on de las diagonales P R y QS . Como BQOP es c clico (B = O = 90 ), se sigue que QBO = QP O = 45 , entonces O pertenece a la bisectriz interna del angulo B . Sea T la intersecci on de BO con RS , entonces QBT = QST = 45 , luego BQT S es c clico y el centro O del circunc rculo de BQT S es la intersecci on de las mediatrices de los segmentos SQ y BT , pero la mediatriz de SQ es P R, por lo tanto el punto O pertenece a P R y si V es el punto medio de BT , se tiene que V OO es un tri angulo rect angulo. Como O O > O V , se tiene que las cuerdas SQ y BT satisfacen SQ < BT , que es lo que se desea.

4.2 Soluciones a los ejercicios del cap tulo 2

177

Termine ahora la demostraci on del ejercicio. Sea ABCD el cuadrado de lados de longitud 1 y sea l la recta que separa los dos cuadrados. A D F
E

B
G H

b C

Si l es paralela a uno de los lados del ABCD, entonces aplique el lema 1. En caso contrario, l intersecta cada recta que determina uno de los lados del cuadrado ABCD. Suponga que A es el v ertice m as lejano a l. Si l corta a los lados de ABCD en E , F , G, H como en la gura, se tiene por el lema 2 que la suma de los cuadrados peque nos es menor o de las diagonales igual a AC , esto es 2(a + b) 2, luego el resultado. Soluci on 2.65 Si , , son los angulos centrales que abren las cuerdas de longitudes a, b, c, respectivamente, se tiene que a = 2 sen 2 , b = 2 sen 2 y c = 2 sen 2 . Luego, abc = 8 sen sen sen 8 sen3 2 2 2 ++ 6 = 8 sen3 (30 ) = 1.

La desigualdad se sigue del ejercicio 1.81. Soluci on 2.66 Una primera observaci on es ver que las diagonales son paralelas a los lados. Sea X el punto de intersecci on de las diagonales AD y CE . Ahora, se puede dividir el pent agono como (ABCDE ) = (ABC ) + (ACX ) + (CDE ) + (EAX ). Como ABCX es un paralelogramo, se tiene que (ABC ) = (CXA) = (CDE ). Sean a = (CDX ) = (EAX ) y b = ( DEX ), entonces se obtiene que a b =
AX XD a b

(CXA) a+b =( CDX ) = a , de donde para encontrar (ABCDE ).

1+ 5 2 .

Ahora, ya cuenta con los elementos

178

Soluciones a los Ejercicios y Problemas

Soluci on 2.67 Primero se tiene que probar sr = s1 R = (ABC ), donde s1 es el semiper metro del tri angulo DEF . Para deducir esta igualdad bastar a que vea que los radios OA, OB y OC son perpendiculares a EF , F D y DE , respectivamente. Use tambi en que R 2r . Soluci on 2.68 Suponga que el angulo m aximo es A y que este satisface que 60 A 90 , entonces la longitud de las alturas hb y hc son tambi en menores hb hc 3 a 1. Utilice ahora el hecho que (ABC ) = 2 sen A y que 2 sen A 1. El caso del tri angulo obtuso es m as f acil. Soluci on 2.69 Si ABCD es el cuadril atero con lados de longitud a = AB , b = BC , c = CD y d = DA. D cd B + cd sen ab+ (i) (ABCD) = (ABC ) + (CDA) = ab sen 2 2 2 . (ii) Si ABCD es el cuadril atero mencionado con lados de longitud a, b, c y d, considere el tri angulo BC D que resulta de reejar DCB con respecto a la mediatriz de BD. Los cuadril ateros ABCD y ABC D tienen la misma area pero el segundo tiene lados de longitud a, c, b y d, en este orden. Utilice ahora (i). bc cd da (iii) (ABC ) ab 2 , (BCD ) 2 , (CDA) 2 y (DAB ) 2 . Soluci on 2.70 En el ejemplo 2.7.6 se demostr o que PA PB PC R (pa + pb )(pb + pc )(pc + pa ). 2r

Utilice la desigualdad M G M A. Soluci on 2.71 (i) (ii) (iii)


PA p b +p c P A2 pb pc

P B2 pc pa

P C2 pa pb

33

P A2 P B 2 P C 2 pb pc pc pa pa pb

4R 2 r

12.

PB p c +p a

PC p a +p b

33 33

PA PB PC p b +p c p c +p a p a +p b

33

R 2r

3. 6.

PA pb pc

PB pc pa

PC pa pb

PA PB PC pb pc pc pa pa pb

33

4R r

Para las u ltimas desigualdades en (i) y (iii) se ha utilizado el ejercicio 2.70. Para la u ltima desigualdad en (ii) use el ejemplo 2.7.6. (iv ) Proceda como en el ejemplo 2.7.5, es decir, haga una inversi on en una circunferencia de centro P y radio d (arbitrario, por ejemplo d = pb ). Sean A , B , C los inversos de A, B , C . Sean p a , pb , pc las distancias de P a los lados B C , C A , A B , respectivamente. pa P B P C como sigue. Se tiene que Pruebe que p a = d2
p a B C = 2(P B C ) =

pa P B P C B C P B P C BC = , P A1 d2

4.2 Soluciones a los ejercicios del cap tulo 2

179

donde A 1 es el inverso de A1 , el pie de la perpendicular de P sobre BC . p c P A P B p b P C P A y p . An alogamente, p c = b = d2 d2 La desigualdad de Erd os-Mordell aplicada en el tri angulo A B C , garantiza que P A + P B + P C 2(pa + pb + pc ). Pero como P A P A = P B P B = P C P C = d2 , al sustituir se tiene que 1 1 pb pc pa 1 + + 2 + + PA PB PC PB PC PC PA PC PA y esta desigualdad es equivalente a P B P C + P C P A + P A P B 2(pa P A + pb P B + pc P C ). Finalmente, para concluir use el ejemplo 2.7.4. Soluci on 2.72 Si P es un punto interior o en el per metro del tri angulo ABC , utilice la demostraci on del teorema 2.7.2. Si ha es la longitud de la altura desde A, se tiene que el area del tri angulo ABC satisface que 2(ABC ) = aha = apa + bpb + cpc . Como ha P A + pa (a un si pa 0, esto es, si P es un punto que est a fuera del tri angulo, en distinto lado de BC que A), adem as la igualdad se da si P est a sobre la altura por A. Luego aP A + apa aha = apa + bpb + cpc , entonces aP A bpb + cpc . Puede aplicar esta desigualdad al tri angulo AB C sim etrico a ABC con respecto a la bisectriz interna del angulo A, donde aP A cpb + bpc , con igualdad cuando AP pase por O. An alogamente, bP B apc + cpa y cP C apb + bpa , por lo tanto PA + PB + PC b c + c b pa + c a pb + + a c a b + b a pc .

La igualdad se da cuando P sea el circuncentro O. Segunda Soluci on. Sean L, M y N los pies de las perpendiculares P sobre BC , CA y AB , respectivamente. Sean H y G la proyecciones ortogonales de B y C , respectivamente, sobre la recta M N . Luego BC HG = HN + N M + M G. Como BN H = AN M = AP M , los tri angulos rect angulos BN H y AP M M BN. De manera an aloga, se tiene que son semejantes, por lo que HN = P PA N CM . MG = P PA Por el teorema de Ptolomeo, aplicado a AM P N , se tiene que P A M N = AN P M + AM P N , por lo que MN = AN P M + AM P N , PA

180 de donde BC Por lo tanto,

Soluciones a los Ejercicios y Problemas

AN P M + AM P N PN PM BN + + CM. PA PA PA BC P A P M AB + P N CA.

b c + pc a . An alogamente para las otras dos desigualdades. Luego, P A pb a

Soluci on 2.73 Considere la sucesi on de reexiones del cuadril atero ABCD, como en la siguiente gura. P B A B
S

D
S

C
R

R Q

C A

Note que el per metro de P QRS es la longitud de la l nea quebrada P QR S P . Note tambi en que A B es paralela a AB , que la distancia m as corta es AA , como se puede ver, se logra si se proyecta O sobre los lados del cuadril atero. Soluci on 2.74 Primero note que (DEF ) = (ABC ) (AF E ) (F BD) (EDC ). Si x = BD, y = CE , z = AF , a x = DC , b y = EA y c z = F B , se tiene que, z (b y ) (F BD) x(c z ) (EDC ) y (a x) (AF E ) = , = y = . (ABC ) cb (ABC ) ac (ABC ) ba Luego, (DEF ) (ABC ) = 1 = y x z y x z 1 1 1 c b a c b a x y z x y z x y z 1 1 1 + =2 . a b c a b c a b c

4.2 Soluciones a los ejercicios del cap tulo 2

181

La u ltima igualdad se debe a que la concurrencia de las cevianas nos garantiza y z x ltimo producto es m aximo cuando que a x by cz = 1. Ahora bien, el u y z x un es 1 a = b = c y como los segmentos concurren el valor com 2 . Luego, se tiene que P es el centroide. Soluci on 2.75 Si x = P D, y = P E y z = P F , se tiene que 2(ABC ) = ax + by + cz . Por la desigualdad de Cauchy-Schwarz, (a + b + c)2
2

a b c + + x y z

(ax + by + cz ) .

a+b+c) b c a +y +z (2( Luego, x ABC ) . La igualdad se da cuando x = y = z , es decir, cuando P es el incentro.

Soluci on 2.76 Primero vea que BD2 + CE 2 + AF 2 = DC 2 + EA2 + F B 2 , usando que BD2 DC 2 = P B 2 P C 2 y relaciones similares. Ahora bien, (BD + DC )2 = a2 , de donde BD2 + DC 2 = a2 2BD DC . An alogamente, para los otros dos lados. Luego, BD2 + DC 2 + CE 2 + AE 2 + AF 2 + F B 2 = a2 + b2 + c2 2(BD DC + CE AE + AF F B ). As , la suma es m nima cuando (BD DC + CE AE + AF F B ) sea m axima. BD +DC 2 a 2 Pero BD DC = 2 y alcanza el m aximo cuando BD = DC . 2 An alogamente, CE = EA y AF = F B , por lo tanto P es el circuncentro. Soluci on 2.77 Como
3

tiene que P D P E P F Adem as, la igualdad ocurre si y s olo si aP D = bP E = cP F . Pero c P F = b P E (ABP ) = (CAP ) P est a sobre la mediana AA . De manera an aloga, vea que P se encuentra en las otras medianas, entonces P es el centroide. Soluci on 2.78 Usando la t ecnica para la demostraci on del teorema de Leibniz, 1 2 (a + b2 + c2 ), donde G es el verique que 3P G2 = P A2 + P B 2 + P C 2 3 centroide. Por lo tanto, se tiene que el punto optimo es P = G. Soluci on 2.79 El cuadril atero AP M N es c clico y est a inscrito en la circunferencia de di ametro AP . La cuerda M N subtiende siempre el angulo A (o 180 A) por lo que la longitud de M N depende proporcionalmente del radio de la circunferencia que circunscribe a AP M N . Tendr a la mayor circunferencia cuando el di ametro AP sea lo m as grande posible. Esto sucede cuando P es diametralmente opuesto a A. En este caso M y

(aP D)(bP E )(cP F )


8 (ABC )3 27 abc .

aP D +bP E +cP F 3

2(ABC ) , 3

se

182

Soluciones a los Ejercicios y Problemas

A N P

M B C

N coinciden con B y C , respectivamente. Por lo tanto, la cuerda m axima M N es BC . Soluci on 2.80 El circunc rculo de DEF es el c rculo de los nueve puntos de ABC , luego intersecta tambi en los puntos medios de los lados de ABC y pasa por L, M , N , los puntos medios de AH , BH , CH , respectivamente. Note que t2 a = AL AD , luego t2 a ha = = AL AD = AD AL = OA

R cos A 3R cos

3 A+B+C = 3R cos 60 = R. 3 2
t2 a ha

Observe que se puede probar un resultado m as fuerte

= R + r , utilizando

A F L E H

M O
A

4.2 Soluciones a los ejercicios del cap tulo 2 el hecho que cos A + cos B + cos C = Soluci on 2.81 (i) Note que pc pb pa + + ha hb hc = = Use ahora que pb pc pa + + ha hb hc ha hb hc + + pa pb pc 9. cpc apa bpb + + aha bhb chc 2(P BC ) + 2(P CA) + 2(P AB ) = 1. 2(ABC )
r R

183

+ 1, vea el lema 2.5.2.

(ii) Por la desigualdad M G M A se tiene 27 pa pb pc ha hb hc pa pc pb + + ha hb hc


3

= 1,

de donde se obtiene la u ltima igualdad de (i). (iii) Sean x = (P BC ), y = (P CA) y z = (P AB ). Observe que a(ha pa ) = aha apa = 2(y + z ) 4 yz . An alogamente, se tiene que b(hb pb ) 4 zx y c(hc pc ) 4 xy . Entonces, a(ha pa )b(hb pb )c(hc pc ) 64xyz = 8(apa bpb cpc ). Por lo tanto, (ha pa )(hb pb )(hc pc ) 8pa pb pc . Soluci on 2.82 Suponga que a < b < c, luego de todas las alturas del tri angulo ABC , AD es la mayor. Si E es la proyecci on de I sobre AD, bastar a ver que AE AO = R. Recuerde que la bisectriz del angulo A es tambi en bisectriz del angulo EAO. Si proyecta I en E sobre el di ametro AA , entonces AE = AE . Ahora vea que AE AO, mostrando que I se encuentra dentro del tri angulo acut angulo COF , donde F es la intersecci on de AA con BC . Para ver que COF es un tri angulo acut angulo, use que los angulos de ABC A, 1 C < 90 A. Use cumplen A < B < C entonces 1 B < 90 2 2 tambi en que COF = A + C B < 90 . Soluci on 2.83 Sea ABC un tri angulo con lados de longitud a, b y c. Utilice la f ormula de Her on para calcular el area de un tri angulo se tiene que (ABC ) = s(s a)(s b)(s c) donde s = a+b+c . 2 (4.7)

184

Soluciones a los Ejercicios y Problemas

Si s y c est an jos, entonces tambi en est a jo s c. Luego el producto de 16(ABC )2 es m aximo cuando (s a)(s b) sea m aximo. Esto es, si s a = s b, es decir, cuando a = b. Por lo tanto, el tri angulo es is osceles. Soluci on 2.84 Sea ABC un tri angulo con lados de longitud a, b y c. Como el per metro es jo, lo es tambi en el semiper metro, utilizando la ecuaci on (4.7), se tiene que 16(ABC )2 es m axima cuando (s a)(s b)(s c) es m aximo. El producto de estos tres n umeros es m aximo cuando (s a) = (s b) = (s c), es decir, cuando a = b = c. Por lo tanto, el tri angulo es equil atero. Soluci on 2.85 Si a, b, c son las longitudes de los lados del tri angulo, observe A+B +C , ya que a + b + c = 2R(sen A + sen B + sen C ) 6R sen 3 que la funci on sen x es c oncava. Adem as, se tiene la igualdad cuando sen A = sen B = sen C . Soluci on 2.86 La desigualdad (lm + mn + nl)(l + m + n) a2 l + b2 m + c2 n es equivalente a l2 + m2 c2 m2 + n2 a2 n2 + l2 b2 + + +30 lm mn ln 3 cos AP B + cos BP C + cos CP A + 0. 2 Ahora, use el hecho que cos + cos + cos + 2 cos + + cos 2 2
2 3 2

0 es equivalente a 2 0.

+ sen2

Soluci on 2.87 Considere el punto de Fermat F y sean p1 = F A, p2 = 1 (p1 p2 + p2 p3 + F B y p3 = F C , entonces observe primero que (ABC ) = 2 p3 p1 )sen 120 = a2 + b2 + c2 =
3 en, 4 (p1 p2 + p2 p3 + p3 p1 ). Tambi 2 2 2p2 1 +2p2 +2p3 2p1 p2 cos 120 2p2 p3 cos 120 2p3 p1 cos 120

2 2 = 2(p2 1 + p2 + p3 ) + p1 p2 + p2 p3 + p3 p1 . 2 2 Ahora usando tiene que a2 + b2 + c2 3(p1 p2 + p2 p3 + que x + y 2xy , se 4 p3 p1 ) = 3 3 3(ABC ) . Luego, a2 + b2 + c2 4 3(ABC ). 2 2 M as a un, la igualdad a2 + b2 + c2 4 3(ABC ) se cumple cuando p2 1 + p2 + p3 = p1 p2 + p2 p3 + p3 p1 , es decir, cuando p1 = p2 = p3 y el tri angulo es equil atero.

4.2 Soluciones a los ejercicios del cap tulo 2

185

Soluci on 2.88 Sean a, b, c las longitudes de los lados del tri angulo ABC . Dena, al igual que en el ejercicio anterior, sean p1 = F A, p2 = F B y p3 = F C . Recuerde que, por la soluci on del ejercicio anterior, se tiene 4 3(ABC ) = 3(p1 p2 + p2 p3 + p3 p1 ). Luego, lo u nico que debe probar es 3(p1 p2 + p2 p3 + p3 p1 ) (p1 + p2 + p3 )2
2 2 pero, esto es equivalente a p1 p2 + p2 p3 + p3 p1 p2 1 + p2 + p3 que es un resultado conocido, ver ejercicio 1.27.

Soluci on 2.89 Como en el problema de Fermat, hay dos casos, cuando ABC tiene todos sus angulos menores a 120 o cuando hay un angulo mayor de 120 . En el primer caso el m nimo de P A + P B + P C es CC , donde C es la imagen de A, al rotar la gura con centro en B un angulo de 60 y en direcci on positiva. Por la ley de los cosenos se tiene que (CC )2 = b2 + c2 2bc cos (A + 60 ) = b2 + c2 bc cos A + bc 3sen A 1 2 (a + b2 + c2 ) + 2 3(ABC ). = 2 Ahora, utilice que a2 +b2 +c2 4 3(ABC ) para (CC )2 4 3(ABC ). obtener Por el teorema 2.4.3 se tiene que (ABC ) 3 3r 2 , por lo tanto (CC )2 36r 2 . En el caso en que A 120 , el punto que resuelve el problema de FermatSteiner es el punto A, por lo que P A + P B + P C AB + AC = b + c. Por lo tanto, lo u nico que se tiene que probar es b + c 6r . M as a un, para esto puede xyz usar el hecho que b = x + z , c = x + y y r = x+y+z . Segunda Soluci on. Es claro que P A + pa ha , donde pa es la distancia de P al lado BC y ha es la longitud de la altura desde A. Luego, ha + hb + hc ltima (P A + P B + P C ) + (pa + pb + pc ) 3 2 (P A + P B + P C ), donde la u desigualdad se debe al teorema de Erd os-Mordell. 1 1 1 +h +h )= Ahora usando el ejercicio 1.36 se tiene que 9 (ha + hb + hc )( h a c b 1 3 (ha + hb + hc )( r ). Por lo que, 9r ha + hb + hc 2 (P A + P B + P C ) y de aqu el resultado. Soluci on 2.90 Primero note que (A1 B1 C1 ) = 1 2 A1 B1 A1 C1 sen B1 A1 C1 . Como P B1 CA1 es un cuadril atero c clico de di ametro P C , aplicando la ley de los senos se obtiene que A1 B1 = P C sen C . An alogamente A1 C1 = P B sen B .

186

Soluciones a los Ejercicios y Problemas

Llame Q a la intersecci on de BP con el circunc rculo del tri angulo ABC , luego, se tiene B1 A1 C1 = QCP . En efecto, como P B1 CA1 es un cuadril atero c clico se tiene que B1 CP = B1 A1 P . An alogamente, C1 BP = C1 A1 P . Entonces, B1 A1 C1 = B1 A1 P + C1 A1 P = B1 CP + C1 BP , pero C1 BP = ABQ = ACQ. Por lo tanto, B1 A1 C1 = B1 CP + ACQ = QCP . Otra vez, la ley de los senos garantiza que A
sen QCP sen BQC

PQ PC .

B1

C1
P

A1

(A1 B1 C1 ) = = = = =

1 A1 B1 A1 C1 sen B1 A1 C1 2 1 P B P C sen B sen C sen QCP 2 PQ 1 P B P C sen B sen C sen QCP 2 PC 1 P B P Q sen A sen B sen C 2 (R2 OP 2 )(ABC ) . 4R2

La u ltima igualdad se da ya que la potencia del punto P , con respecto al circunc rculo de ABC , es P B P Q = R2 OP 2 y el area del tri angulo ABC 2 est a dada por (ABC ) = 2R sen A sen B sen C . Luego, (A1 B1 C1 ) es m axima cuando P = O, es decir, cuando A1 B1 C1 es el tri angulo medial.

4.3 Soluciones a los problemas del cap tulo 3

187

4.3.

Soluciones a los problemas del cap tulo 3

Soluci on 3.1 Considere a = A1 A2 , b = A1 A3 y c = A1 A4 y aplique el teorema de Ptolomeo al cuadril atero A1 A3 A4 A5 para tener que ab + ac = bc, a a o equivalentemente que b + c = 1. B1 B2 A1 A2 Como los tri angulos A1 A2 A3 y B1 B2 B3 son semejantes B = A = a b y 1 B3 1 A3 de aqu se obtiene B1 B2 =
a2 b2
2 2 2 2 2

a c +a b + a = c2 = b2 c2 de ab + ac = bc y la desigualdad por la desigualdad (1.11). La desigualdad es estricta ya que b = c. 2 2 2 a 2 a2 Note que, a 2a +a = a b + c bc = 1 2 bc . b2 c2 Ley de los senos aplicada al tri angulo A1 A3 A4 lleva a que

2 + SC a2 alogamente C1 C2 = ac . Por lo que SBS = b . An A 2 2 2 (b+c) b +c 1 (b+c)2 > 2(b+c)2 = 2 . La tercera igualdad se sigue

a2 bc

= = = >

sen 2 sen 2 7 7 2 2 4 = 2 sen 7 sen 7 2sen 7 sen 7 cos 27 sen 2 sen 2 7 7 = ) cos 27 (1 + cos 27 )(1 cos 27 ) 2(1 cos2 27 2 cos 27 sen 2 1 7 2 = 2 2 2 ) 4 cos 7 (1 + cos 7 )sen 7 4 cos 7 (1 + cos 27 21 1 1 . = = 2 2 4 cos 2 (1 + cos ) 4 (1 + ) 4 4
2 2

Luego

a2 b2

a2 c2

2 = 1 2a bc < 1 ( 2 1) = 2 2.

Soluci on 3.2 Corte el tetraedro por las aristas DA, DB , DC y desd oblelo en el plano del tri angulo ABC . Las caras ABD, BCD y CAD tendr an por imagen a los tri angulos ABD1 , BCD2 y CAD3 . Vea que D1 , B y D2 son colineales as como tambi en lo son D3 , A y D1 , y que B y A son puntos medios 1 D2 D3 y por la desigualdad de D1 D2 y D3 D1 , respectivamente. Luego, AB = 2 del tri angulo D2 D3 CD3 + CD2 = 2CD. Por lo tanto AB CD, como se quer a. Soluci on 3.3 Sea S el area del tri angulo entonces se tienen las siguientes 2S S S 2S S , sen = , sen = 2 f ormulas sen = 2 bc ca ab , y r = s = a+b+c . Use estas f ormulas para obtener que la desigualdad a probar es equivalente a a b c + + bc ca ab (a + b + c) 9,

188

Soluciones a los Ejercicios y Problemas

la cual puede ser mostrada usando la desigualdad M G M A en cada factor del lado izquierdo. Soluci on 3.4 Suponga que los c rculos son de radio 1. Sea P el punto com un de los c rculos y sean A, B , C los otros puntos de intersecci on. El area de los gajos ser a m nima si el punto P est a dentro del tri angulo ABC (sino rote un c rculo 180 alrededor de P , y esto reducir a el area).

A P B C

El area de los gajos es igual a (sen + sen + sen ), donde , , son los angulos centrales que abren los arcos comunes de los c rculos. Es claro que + + = 180 . Como la funci on sen x es c oncava, el m nimo se logra cuando , lo que implica que los centros de las circunferencias forman = = = 3 un tri angulo equil atero. Soluci on 3.5 Sea I el incentro del tri angulo ABC y dibuje la recta por I perpendicular a IC . Sean D , E las interesecciones de esta recta con BC y CA, respectivamente. Primero pruebe que (CDE ) (CD E ) usando que el area de D DI es mayor que el area de EE I . Para esto observe que uno de los tri angulos DD I , EE I queda en el lado opuesto C con respecto a D E , si por ejemplo, es el tri angulo D DI entonces este tendr a area mayor o igual al area del tri angulo EE I , luego la desigualdad se sigue. Ahora, muestre que el area r r 2r 2 I = y que D , , para esto u ltimo note que CI = de (CD E ) es sen C sen C cos C luego
2 2

2r 2 1 2r 2 = (CD E ) = D E CI = 2r 2 . C C 2 sen C 2 sen 2 cos 2

Soluci on 3.6 La clave est a en notar que 2AX 3(AB + BX ), que se puede deducir aplicando el teorema de Ptolomeo (ejercicio 2.11) al cuadril atero c clico

4.3 Soluciones a los problemas del cap tulo 3

189

que resulta de pegar al tri angulo ABX un tri angulo equil atero AXO de lado 2 AX , as se AX y luego observar que el di ametro de tal circunferencia es 3 tendr a que AX (AB + BX ) = AX BO AX
2 AX . 3

Por lo tanto

2AD = 2(AX + XD ) 3(AB + BX ) + 2XD 3(AB + BC + CX ) + 3XD 3(AB + BC + CD). Soluci on 3.7 Tome el tri angulo A B C de area m axima de entre todos los tri angulos que se pueden formar con tres v ertices de los puntos dados; entonces su area satisface que (A B C ) 1. Construya otro tri angulo ABC que tenga a A B C como tri angulo medial; este tiene area (ABC ) = 4(A B C ) 4. En ABC se encuentran todos los puntos. En eferto, si alg un punto Q est a fuera del tri angulo ABC , estar a en uno de los semiplanos determinados por los lados y opuesto al semiplano donde est a el tercer v erticie. Por ejemplo, si Q est a en el semiplano determinado por BC , y opuesto a donde est a A, el tri angulo QB C es de area mayor a A B C , lo cual es una contradicci on.
1 1 Soluci on 3.8 Sea M = 1 + 2 + + n . Queremos probar que M es el valor m nimo deseado, resultado que se alcanza si hacemos x1 = x2 = = xn = 1. k Usando la desigualdad M G M A, se tiene que xk k +(k 1) = xk +1+ +1

xk k 1 1 = kxk , para toda k . Por lo tanto,

x1 +

3 x2 xn 1 n1 2 x3 + + + n x 1 + x2 + + x n = x1 +x2 + +xn n+M. 2 3 n 2 n

Por otro lado, la desigualdad entre la media arm onica y la media aritm etica nos lleva a n x1 + x2 + + xn 1 1 1 = 1. n x + x + + xn
1 1

Podemos concluir que la expresi on dada es al menos n n + M = M . Como hemos visto podemos encontrar M y es el m nimo deseado. Segunda Soluci on. Aplique la desigualdad M G M A con pesos, a los n umeros xj j , con pesos
1

tj = xj j j

j P 1 j

, para obtener 1 j
1 P 1 j

(x1 x2 ...xn )

1 . j

190

Soluciones a los Ejercicios y Problemas


1 x1

La u ltima desigualdad se sigue de n n

x1 n

1 xj

= n.

Soluci on 3.9 Note que AF E y BDC son tri angulos equil ateros. Sean C y F puntos fuera del hex agono y de manera que ABC y DEF sean tambi en tri angulos equil ateros. Como BE es la mediatriz del segmento AD, se sigue que C y F son los reejados de C y F en la recta BE . Use ahora que AC BG y EF DH son c clicos, para concluir que AG + GB = GC y DH + HE = HF .
1 2 (a + b2 + c2 ). Soluci on 3.10 El teorema de Leibniz garantiza que OG2 = R2 9 abc abc Como rs = 4R , se tiene que 2rR = a+b+c . Luego, lo que hay que demostrar

es que abc

(a+b+c) (a2 +b2 +c2 ) , 3 3

para esto use M G M A.

Soluci on 3.11 El lado izquierdo de la desigualdad se sigue de que 1 1 + x0 + x1 + + xi1 xi + + xn (1 + x0 + + xn ) = 1. 2 Para el lado derecho considere i = arcsen (x0 + + xi ), para i = 0, ..., n. Note que 1 + x0 + + xi1 xi + + xn = S olo resta ver ahora que sen i sen i1 = 2 cos
sen i sen i1 cos i1

= cos i1 .

1 + sen i1

1 sen i1

< 2 . Pero

i i+1 i + i+1 sen < (cos i1 )(i i1 ). 2 2

Para probar la desigualdad use que el coseno es decreciente y que sen , para 0 2 . Luego sen i sen i1 < cos i1 i i1 = n 0 = . 2

2 n n n 2 Soluci on 3.12 Si i=1 xi = 1 entonces 1 = ( i=1 xi ) = i=1 xi + 2 i<j xi xj . Por lo tanto, la desigualdad que se pide demostrar es equivalente a n 1 x2 i . n1 1 ai i=1

4.3 Soluciones a los problemas del cap tulo 3 Use la desigualdad de Cauchy-Schwarz para ver que
n 2 n

191

xi
i=1

i=1

x2 i 1 ai

n i=1

(1 ai ) .

2 Soluci on 3.13 Note primero que n i=1 xn+1 (xn+1 xi ) = (n 1)xn+1 . La desigualdad a demostrar se reduce a n i=1

xi (xn+1 xi )

n 1xn+1 .

Aplique Cauchy-Schwarz con ( x1 , . . . , xn ) y ( xn+1 x1 , . . . , xn+1 xn ). Soluci on 3.14 Recuerde que N tambi en es punto medio del segmento que une los puntos medios X , Y de las diagonales AC y BD. La circunferencia de di ametro OM pasa por X y Y , ya que OX y OY son perpendiculares a las diagonales respectivas, y ON es una mediana del tri angulo OXY . Soluci on 3.15 La desigualdad de la derecha se deduce de wx + xy + yz + zw = (w + y )(x + z ) = (w + y )2 0. Para la desigualdad de la izquierda, note que |wx + xy + yz + zw| = |(w + y )(x + z )| 1 (w + y )2 + (x + z )2 2 w2 + x2 + y 2 + z 2 = 1. Tambi en puede usar la desigualdad de Cauchy-Schwarz para obtener |wx + xy + yz + zw|2 (w2 + x2 + y 2 + z 2 )(x2 + y 2 + z 2 + w2 ) = 1. Soluci on 3.16 Para la desigualdad de la izquierda, reacomode as an + a2 a1 + a3 an1 + a1 a1 a2 a3 a2 a1 an + ++ = + + + ++ + , a1 a2 an a2 a1 a2 a3 an a1 ahora utilice que
x y

+a2 1 +a1 Llame Sn = ana . Pruebe por inducci on que + + + an an 1 Sn 3n. c +a a+b c Primero para n = 3, se necesita ver que b+ a + b + c 9. Si a = b = c, c c +a a+b entonces b+ a + b + c = 6 y la desigualdad es cierta. Suponga ahora que

y x 2. a1 +a3 a2 +a4 a2 + a3

192

Soluciones a los Ejercicios y Problemas

a b c y que no son todos iguales, entonces hay tres casos: a = b < c, a < b = c, a < b < c. Pero en cualquiera de ellos se tiene que a b y a < c. a+b b Luego 2c = c + c > a + b y a+ c < 2, y como c es entero positivo se tiene que c = a + b. c c +a a+b a+2b 2a+b b a b a Luego, b+ a + b + c = a + b + 1 = 3 + 2 a + 2 b . Como 2 a y 2 b b deben ser enteros positivos y como 2 a 2a b = 4 se tiene o bien que son 2 los dos n umeros o bien uno 1 y el otro 4. Esto quiere decir que la suma es a lo m as 8, que es menor que 9, luego el resultado. Continue con la inducci on, suponga que Sn1 3(n1). Considere {a1 , . . . , an }, si todos son iguales, Sn = 2n y la desigualdad es verdadera. Suponga entonces que hay al menos dos ai diferentes. Tome el m aximo de las ai s; sus vecinos (ai1 , ai+1 ) pueden ser iguales a este valor m aximo, pero como hay entre los ai dos diferentes, para alg un m aximo ai , se tiene que uno de sus vecinos es menor que ai . Sin perder generalidad suponga que an es m aximo y que alguno de sus vecinos an1 o a1 es menor que an . Luego, como 2an > an1 + a1 se tiene que an1 +a1 1 +a1 < 2 y entonces an = 1, por lo que an = an1 + a1 . Al sustituir an an este valor de an en Sn , obtendr a que Sn = an2 + an1 + a1 an1 + a1 an1 + a1 + a2 a2 + a3 + + + + = a1 a2 an1 an1 + a1 1+ an1 + a2 a2 + a3 an2 + a1 + + + + 1 + 1. a1 a2 an1

Como Sn1 3(n 1), se concluye que Sn 3n. Soluci on 3.17 Como el cuadril atero OBDC es c clico, utilice el teorema de DC BD Ptolomeo para demostrar que OD = R BC + BC , donde R es el circunradio de ABC . Por otro lado, como los tri angulos BCE y DCA son semejanAD DC AD tes, al igual que ABD y F BC , sucede que R BD BC + BC = R F C + EB . BE CF CF An alogamente, tenemos que OE = R BE y OF = R BE + AD . AD + CP Multiplicando estas igualdades y aplicando M G M A obtendr a el resultado. Otra forma de demostrar lo anterior es utilizando inversi on. Sean D , E y F los puntos de intersecci on de AO, BO y CO con los lados BC , CA y AB , respectivamente. Invierta los lados BC , CA y AB con respecto a (O, R), obtendr a los circunc rculos de los tri angulos OBC , OCA y OAB , respectivamente. Luego, OD OD = OE OE = OF OF = R2 . Si x = (ABO), y = (BCO) y z = (CAO), se tiene que z+x AO , = OD y BO x+y = OE z y CO y+z . = OF x

4.3 Soluciones a los problemas del cap tulo 3 Lo que implica, usando M G M A, que OE OF 8R3 .
R3 OD OE OF

193 8, por lo que, OD

Soluci on 3.18 Primero, observe que AY 2R y que ha AX , donde ha is la longitud de la altura sobre BC . Luego, deduzca que la sen 2 A = = 3 = 3 como ha = b sen C , hb = c sen A, hc = a sen B . Soluci on 3.19 Sin perder generalidad, podemos suponer x1 x2 xn . Como 1 < 2 < < n se tiene, por la desigualdad del reacomodo (1.2), A = x1 + 2x2 + + nxn nx1 + (n 1)x2 + + xn = B. Luego, |A + B | = |(n + 1) (x1 + + xn )| = n + 1, por lo que A + B = (n + 1). Ahora bien, si A + B = n + 1 se cumple que B n+1 2 A, y si A + B = (n + 1) sucede que B n+1 A . 2 n+1 o est a n entre B y A, ya que en caso contrario A Suponga ahora que n+1 2 2 n+1 o B estar an en el intervalo n+1 , . Luego, uno de los valores |A| o |B | 2 2 n+1 es menor o igual a 2 y termina el problema. n+1 Suponga por lo tanto, B n+1 2 < 2 A. Sea y1 , ..., yn una permutaci on de x1 , ..., xn tal que 1y1 + 2y2 + + nyn = C toma el valor m as grande con C n+1 2 . Tome i tal que y1 y2 yi y yi > yi+1 , y considere D = y1 + 2y2 + + iyi+1 + (i + 1)yi + (i + 2)yi+2 + + nyn
3

AX AY sen 2 A ha 2R sen 2 A 1 sen A ha = como a sen A a 2R hb hc ha a sen A b sen B c sen C

D C = iyi+1 + (i + 1)yi (iyi + (i + 1)yi+1 ) = yi yi+1 > 0.

Como |yi |, |yi+1 | n+1 2 , se tiene que D C = yi yi+1 n + 1; luego, D C + n + 1 y entonces C < D C + n + 1 n+1 2 . , pues C es la suma m a s grande que es menor a n+1 Por otro lado, D n+1 2 2 . n+1 n+1 n+1 Luego, 2 D 2 y as |D| 2 .

194

Soluciones a los Ejercicios y Problemas

Soluci on 3.20 Entre los n umeros x, y , z dos tienen el mismo signo (suponga y que son x y y ), como c = z x y + x es positivo, z es positivo.
2yz 2zx Note que a + b c = 2xy z , b + c a = x , c + a b = y son positivos. Rec procamente, si u = a + b c, v = b + c a y w = c + a b son 2yz 2zx positivos entonces haciendo u = 2xy z , v = x , w = y , podemos obtener que

a=

u+w 2

=x

y z

z y

, etc.

Soluci on 3.21 Primero vea que un hex agono ABCDEF centralmente sim etrico tiene lados opuestos paralelos. Esto garantiza que (ACE ) = (BDF ) = (ABCDEF ) . Ahora, si reejamos los v ertices del tri angulo P QR con respecto al 2 centro de simetr a del hex agono, obtenemos los puntos P , Q , R que forman el hex agono centralmente sim etrico P R QP RQ , inscrito en ABCDEF y de area 2(P QR).
4 1 3 3 Soluci on 3.22 Sean X = 4 i=1 Xi . i=1 xi , Xi = X xi ; es claro que X = 3 1 1 3 3 3 3 Por la desigualdad M G M A, 3 X1 x2 x3 x4 = x1 ; an alogamente, para los 4 1 otros ndices y esto implica que X i=1 xi . Por la desigualdad de Tchebyshev se obtiene 2 2 3 3 3 x2 + x2 x3 2 + x3 + x4 x1 + x2 + x3 + x4 1 + x 2 + x3 + x 4 1 4 4 4

Gracias a la desigualdad M G M A se tienen 1 y por lo tanto, X i=1 xi .

2 2 2 x2 1 +x 2 +x 3 +x 4 4

(x1 x2 x3 x4 )2 =
y 1 x 1 z 1 , , y x z

Soluci on 3.23 Use la desigualdad de Cauchy-Schwarz con u = x, y, z . yv= Soluci on 3.24 Si = ACM y = BDM , entonces
4. 4.

+ , Ahora, use el hecho de que tan tan tan tan3 + y + = 3 donde = Podemos utilizar otro m etodo es el cual usa el hecho de que la desigualdad +l 3 3, equivalente a (M CD) 3 3(M AB ) que a su vez es equivalente a hh donde l es la longitud del lado del cuadrado y h es la longitud de la altura desde M sobre AB . Ahora, encuentre la m axima h. L PM PN Soluci on 3.25 Primero note que P AL + BM + CN = 1. Ahora, use el hecho de que AL, BM y CN son menores que a.

M A M B M C M D

= tan tan

4.3 Soluciones a los problemas del cap tulo 3 Soluci on 3.26 Como
PB PA PB PA

195
2

QC QA

= 1.

QC QA

1 4

PB PA

QC QA

, es suciente probar que

Trace BB , CC paralelas a la mediana AA de tal forma que B y C est en en P Q. Los tri angulos AP G y BP B son semejantes, y tambi en los tri angulos QC BB CC B = y = . Use esto AQG y CQC son semejantes, entonces P PA AG QA AG junto con el hecho de que AG = 2GA = BB + CC . Soluci on 3.27 Sean el circunc rculo del tri angulo ABC y R su radio. Considere la inversi on en . Para cualquier punto P distinto de O, sea P su inverso. El inverso del circunc rculo de OBC es la recta BC , entonces A 1 , el inverso de A1 , es el punto de intersecci on entre el rayo OA1 y BC . Como7 P Q = R2 P Q OP OQ

para dos puntos P , Q (distintos de O) con inversos P , Q , se tiene R2 A A AA x+y+z AA1 1 1 = = = , OA1 OA OA1 OA1 OA y+z donde x, y , z denotan las areas de los tri angulos OBC , OCA y OAB , respectivamente. An alogamente, se tiene que BB1 CC1 x+y+z x+y+z y . = = OB1 z+x OC1 x+y Luego, BB1 CC1 AA1 + + = (x + y + z ) OA1 OB1 OC1
7

1 1 1 + + y+z z+x x+y

9 . 2

Vea [6], p ag. 132 o [9], p ag. 112.

196

Soluciones a los Ejercicios y Problemas

Para la u ltima desigualdad, vea el ejercicio 1.44. Soluci on 3.28 Note que el area del tri angulo GBC es, (GBC ) = 2(ABC ) ) aGL . An alogamente, GN = 2(ABC . 2 , de donde GL = 3a 3c Por lo tanto, (GN L) = = GL GN sen B 4(ABC )2 sen B = 2 18ac (ABC )2 b2 (ABC ) b2 4(ABC )2 b2 = . = (18abc)(2R) 9 4R2 (9R abc 4R )(4R)
(ABC ) c2 94R2 (ABC ) 3

An alogamente, (GLM ) =

y (GM N ) = =

(ABC ) a2 . 94R2

Por lo tanto,

(LM N ) 1 = (ABC ) 9

a2 + b2 + c2 4R2

R2 OG2 . 4R2

La desigualdad de la derecha es ahora inmediata. angulo es acut anguPara la otra desigualdad, note que OG = 1 3 OH . Como el tri lo, H est a dentro del tri angulo y lo m as alejado que H est a de O es R. Por lo tanto, 1 1 2 R2 9 R2 9 OH 2 R 4 2 (LM N ) = = > . 2 2 (ABC ) 4R 4R 9 27 Soluci on 3.29 La funci on f (x) = f (ab) + f (bc) + f (ca) 3 f
1 1+x

es convexa para x > 0. Luego,

ab + bc + ca 3 = 3 3 + ab + bc + ca 3 1 = , 3 + a2 + b2 + c2 2

la u ltima desigualdad se sigue de que ab + bc + ca a2 + b2 + c2 . Tambi en se puede partir de 1 1 1 9 9 3 + + = . 2 2 2 1 + ab 1 + bc 1 + ca 3 + ab + bc + ca 3+a +b +c 2 La primera desigualdad se sigue de la desigualdad (1.11) y la segunda se sigue del ejercicio 1.27. Soluci on 3.30 Dena x = b + 2c, y = c + 2a, z = a + 2b. La desigualdad deseada se transforma en x y + y x + y z + z y + x z +3 + x z y z x + + x y z 15,

4.3 Soluciones a los problemas del cap tulo 3 que se resuelve aplicando la desigualdad M G M A. Otra forma de probar la desigualdad es la siguiente:

197

b c a2 b2 c2 (a + b + c)2 a + + = + + . b + 2c c + 2a a + 2b ab + 2ca bc + 2ab ca + 2bc 3(ab + bc + ca) La desigualdad se sigue inmediatamente de la desigualdad (1.11). S olo falta probar la desigualdad (a + b + c)2 3(ab + bc + ca), que es una consecuencia de la desigualdad de Cauchy-Schwarz. Soluci on 3.31 Utilice la desigualdad (1.11) o bien utilice la desigualdad de Cauchy-Schwarz con a , b , c , d y ( a + b, b + c, c + d, d + a). a+b b+c c +d d+a Soluci on 3.32 Sean x = b + c a, y = c + a b y z = a + b c. La semejanza entre los tri angulos ADE y ABC nos da que per metro de ADE 2x DE = = . a per metro de ABC a+b+c
x(y +z ) Luego, DE = x +y +z , es decir, la desigualdad es equivalente a ver que x +y +z . Utilice la desigualdad M G M A. 4 x(y +z ) x +y +z

Soluci on 3.33 Tome F sobre AD con AF = BC y dena E el punto de intersecci on de BF con AC . Use la ley de los senos en los tri angulos AE F , BCE y BDF , para obtener AF sen F sen E sen F BD AE AE = = = = EC sen E BC sen B sen B FD EC entonces, E = E . Considere despu es, a G sobre BD con BG = AD y a H la intersecci on de GE con la paralela a BC por A. Use que los tri angulos ECG y EAH son semejantes, y el teorema de Menelao en el tri angulo CAD, con transversal EF B , para concluir que AH = DB . Tendr a ahora que BDAH es un paralelogramo, que BH = AD y que BHG es is osceles con BH = BG = AD > BE . Soluci on 3.34 Observe que ab + bc + ca 3abc si y s olo si Como 1 1 1 (a + b + c) + + 9, a b c
1 a

1 b

1 c

3.

198

Soluciones a los Ejercicios y Problemas

debe tenerse que (a + b + c) 3. Entonces 3(a + b + c) (a + b + c)2 = a3/2 a1/2 + b3/2 b1/2 + c3/2 c1/2 a3 + b3 + c3 1 1 1 + + a b c
2

3 a3 + b3 + c3 . Soluci on 3.35 Tome yi = desigualdad, es decir,


xi n1 ,

para i = 1, 2, . . . , n, y suponga falsa la

1 1 1 + + + > n 1. 1 + y1 1 + y2 1 + yn Luego 1 1 + yi >


j =i

1 1 + yj

=
j =i

yj 1 + yj ,

(n 1) n1

(1 + y1 ) (1 + yi ) (1 + yn )

y1 y i yn

donde y1 y i yn es el producto de las y s salvo la yi . Entonces


n i=1

1 y1 yn , > (n 1)n 1 + yi (1 + y1 ) (1 + yn )

y esto u ltimo lleva a que 1 > x1 xn , que es una contradicci on. Soluci on 3.36 Use la desigualdad de Cauchy-Schwarz con las colecciones x1 xn x1 y1 , . . . , xn yn para garantizar que y y1 , . . . , yn ( x1 + + x n ) 2 = Ahora use la condici on desigualdad u til (1.11).
2 x1 xn x1 y 1 + + xn y n y1 yn xn x1 + + ( x1 y 1 + + x n y n ) . y1 yn

xi y i

xi . Tambi en puede resolverse usando la

4.3 Soluciones a los problemas del cap tulo 3

199

Soluci on 3.37 Como abc = 1, tenemos que (a 1)(b 1)(c 1) = a + b + 1 1 +1 alogamente, c (a b + c ). An (an 1)(bn 1)(cn 1) = an + bn + cn 1 1 1 + n+ n n a b c .

La armaci on se sigue del hecho de que el lado izquierdo de cada una de las identidades tiene el mismo mismo signo. Soluci on 3.38 Haga la demostraci on por inducci on en n. El caso n = 1 es claro. Suponga el resultado cierto para n y vea que es v alida la desigualdad para n + 1. Como se tiene que n < n2 + i < n + 1, para i = 1, 2, ..., 2n, sucede que n2 + i = Luego
(n+1)2 n2 (n+1)2

n2 + i n <

n2 + i +

i 2n

n=

i . 2n

j
j =1

=
j =1

j +
j =n2 +1

n2 1 1 + 2 2n

2n

i
i=1

(n + 1)2 1 . 2

Soluci on 3.39 Vea la contrarrec proca, esto es, x3 + y 3 > 2, implica que
y x2 + y 3 < x3 + y 4 . La desigualdad entre medias potenciales x + 3 x 2 implica que 3 x2 + y 2 (x3 + y 3 )2/3 2 < (x3 + y 3 )2/3 (x3 + y 3 )1/3 = x3 + y 3 .
2 2 3 +y 3

Luego x2 x3 < y 3 y 2 y 4 y 3 . La u ltima desigualdad se debe al hecho 2 2 que y (y 1) 0. Segunda Soluci on. Como (y 1)2 0, se tiene que 2y y 2 + 1, luego 3 4 2 2y y + y . Luego, x3 + y 3 x3 + y 4 + y 2 y 3 x2 + y 2 , ya que x3 + y 4 x2 + y 3 . Soluci on 3.40 La desigualdad es equivalente a ( x 0 x1 ) + 1 1 + (x1 x2 ) + + (xn1 xn ) + 2n. (x0 x1 ) (xn1 xn )

200 Soluci on 3.41 Como que


a+3b 4

Soluciones a los Ejercicios y Problemas 4 ab3 ,


b+4c 5

bc4 y
11

c+2a 3
17

ca2 , tenemos

(a + 3b)(b + 4c)(c + 2a) 60a 12 b 20 c 15 . Ahora, muestre que c 15 a 12 b 20 o, equivalentemente, que c8 a5 b3 . Soluci on 3.42 Se tiene una equivalencia entre las siguientes desigualdades, 7(ab + bc + ca) 2 + 9 abc
2 1 1

19

a2 b + a b2 + b2 c + b c2 + c2 a + c a2 2(a3 + b3 + c3 ). Para ver la validez de la u ltima desigualdad, use la desigualdad del reacomodo o la desigualdad de Tchebyshev. Soluci on 3.43 Sea E la intersecci on de AC y BD. Los tri angulos ABE y DCE son semejantes, entonces |AB | |AB CD| = . |AC BD| |AE EB |
|AB | Usando la desigualdad del tri angulo en ABE , se tiene que |AE EB | 1 y concluya entonces que |AB CD| |AC BD|. An alogamente, |AD BC | |AC BD|. +1) Soluci on 3.44 Muestre, que siempre se tiene que a1 + + aj j (j 2n an , para j n, de la siguiente forma. Primero pruebe que la desigualdad es v alida para a ; use el hecho de que 2( a + + an ) = j = n, es decir, a1 + + an n+1 n 1 2 a1 ++aj (a1 + an1 ) + + (an1 + a1 ) + 2an . Luego, pruebe que si bj = 1+ +j , an on que bj bj +1 , entonces b1 b2 bn n (para probar por inducci a +1 , lo cual se sigue de la primera parte para necesitamos probar que, bj jj+1 n = j + 1). +1) Damos otra demostraci on de a1 + + aj j (j 2n an , nuevamente usando inducci on. Es claro que

7(ab + bc + ca)(a + b + c) 2(a + b + c)3 + 9 abc

a1 a1 a1 + a2 a1 a1 a2 a2 a2 = + + + = a2 . 2 2 2 2 2 2

4.3 Soluciones a los problemas del cap tulo 3

201

Ahora bien, supongamos que la armaci on es v alida para n = 1, . . . , j , es decir, a1 a1 a2 a2 a1 + 2 . . . . . . aj a2 a1 + + + aj . 2 j Sumando todas las desigualdades anteriores se tiene ja1 + (j 1) aj a2 + + a1 + + aj . 2 j

Sumando de ambos lados la identidad a1 + 2 se tiene que (j +1) a1 + Luego, a1 +


a

aj a2 + + j = aj + + a1 2 j

aj a2 + + 2 j

(a1 + aj )+(a2 + aj 1 )+ +(aj + a1 ) jaj +1 .

aj j a2 + + aj +1 . 2 j j+1

+1 Finalmente, sumando jj+1 en ambos lados de la desigualdad, terminamos el u ltimo paso de la prueba por inducci on. Ahora,

a1 +

an a2 + + 2 n

1 (a1 + + an ) + n 1 n n(n + 1) an 2n +

n1 j =1 n1 j =1

1 1 j j+1

(a1 + + aj )

j (j + 1) 1 an = an . j (j + 1) 2n

202 Soluci on 3.45

Soluciones a los Ejercicios y Problemas

1in

xi

x2 i +2
1in

= 8 8

= 8

1in

2 x2 i x2 i

1i<j n

xi xj

1i<j n

xi xj

1in

1i<j n

xi xj (x2 1

1i<j n 1i<j n

+ + x2 n)

xi xj

2 xi xj (x2 i + xj ).

Para la primera desigualdad se aplica la desigualdad M G M A. Para determinar cu ando la igualdad ocurre, note que en el u ltimo paso dos de las xi deben ser diferentes de cero y las n 2 restantes iguales a cero; adem as, en el paso en el que se us o la desigualdad M G M A, las xi diferentes de cero deben ser iguales. Se puede comprobar que, en tal caso, la constante C = 1 8 es la m nima. Soluci on 3.46 Sea 3 a = x y 3 b = y , necesita mostrar que (x2 + y 2 )3 2(x3 + y 3 )2 , para x, y > 0. Por la desigualdad M G M A tenemos 3x4 y 2 x6 + x3 y 3 + x3 y 3 y con igualdad si y s olo si x6 = x3 y 3 = y 6 o, equivalentemente, si y s olo si x = y . Sumando estas dos desigualdades y sumando x6 + y 6 a ambos lados, obtendr a x6 + y 6 + 3x2 y 2 (x2 + y 2 ) 2(x6 + y 6 + 2x3 y 3 ). La igualdad ocurre cuando x = y , esto es, cuando a = b. Soluci on 3.47 Denote por S el lado izquierdo de la desigualdad. Como a b c y x y z , por la desigualdad del reacomodo obtiene bz + cy by + cz , entonces (by + cz )(bz + cy ) (by + cz )2 2((by )2 + (cz )2 ). 3x2 y 4 y 6 + x3 y 3 + x3 y 3 ,

4.3 Soluciones a los problemas del cap tulo 3 Dena = (ax)2 , = (by )2 , = (cz )2 , luego, a2 x2 a2 x2 = . (by + cz )(bz + cy ) 2((by )2 + (cz )2 ) 2( + ) Sumando las otras dos desigualdades similares, obtendr a S 1 2 + + + + + .

203

Para terminar la demostraci on use la desigualdad de Nesbitt. Soluci on 3.48 Si XM es una mediana en el tri angulo XY Z , entonces, 1 2 2 como resultado de utilizar el teorema de Stewart, XM 2 = 1 2 XY + 2 XZ 1 2 4 Y Z . Sustituya (X, Y, Z, M ) por (A, B, C, P ), (B, C, D, Q), (C, D, A, R) y (D, A, B, S ) en esta f ormula y sumando las cuatro ecuaciones obtenidas, se tiene una quinta ecuaci on. Multiplicando ambos lados de la quinta ecuaci on por 4, encontrar a que el lado izquierdo de la desigualdad deseada es igual a AB 2 + BC 2 + CD2 + DA2 + 4(AC 2 + BD2 ). Luego, es suciente mostrar que AC 2 + BD2 AB 2 + BC 2 + CD2 + DA2 . Esta desigualdad es conocida como la desigualdad del paralelogramo. Para demostrarla, sea O un punto arbitrario en el plano y, para cada punto X sea x el vector de O a X . Desarrolle cada uno de los t erminos en AB 2 + BC 2 + CD2 + DA2 AC 2 BD2 , por ejemplo, escribiendo AB 2 = |a b|2 = |a|2 2a b + |b|2 y encontrar a que esta expresi on es igual a |a|2 + |b|2 + |c|2 + |d|2 2(a b + b c + c d + d a a c b d) = |a + c b d|2 0, con igualdad si y s olo si a + c = b + d, esto es, si el cuadril atero ABCD es un paralelogramo. Soluci on 3.49 Dena A = x2 + y 2 + z 2 , B = xy + yz + zx, C = x2 y 2 + y 2 z 2 + z 2 x2 , D = xyz . Entonces 1 = A + 2B , B 2 = C + 2xyz (x + y + z ) = C + 2D, y x4 + y 4 + z 4 = A2 2C = 4B 2 4B + 1 2C = 2C 4B + 8D + 1. Entonces, la expresi on del centro es igual a 3 2A + (2C 4B + 8D + 1) = 2 + 2C + 8D 2, con igualdad si y s olo si dos de las variables x, y , z son cero.

204

Soluciones a los Ejercicios y Problemas

Ahora, la expresi on de la derecha es igual a 2 + B + D. Luego, se tiene que demostrar que 2C + 8D B + D o B 2B 2 3D 0. Utilice la desigualdad de Cauchy-Schwarz para obtener A B , luego B (1 2B ) = BA B 2 . Entonces es suciente demostrar que B 2 3D = C D 0. Pero C xyyz + yzzx + zxxy = D lo cual se deduce de la desigualdad de CauchySchwarz. Soluci on 3.50 Suponga que a = equivalente a x z 1+ y y
x y,

b =

y z

, c =

z x.

La desigualdad es

y x 1+ z z

z y 1, 1+ x x

que se reescribe como (x + z y )(x + y z )(y + z x) xyz. Esta u ltima es v alida si x, y , z son las longitudes de los lados de un tri angulo. Vea el ejemplo 2.2.3. Falta considerar el caso en que algunas de las variables u = x + z y , v = x + y z , w = y + z x sean negativas. Si una o tres de ellas son negativas entonces el lado izquierdo es negativo y la desigualdad es evidente. Si dos de los valores u, v , w son negativos, por ejemplo u y v , entonces tambi en u + v = 2x es negativa; pero x > 0, pero esta u ltima situaci on no es posible. Soluci on 3.51 Note primero que abc a + b + c implica que (abc)2 (a + b + c)2 3(a2 + b2 + c2 ), donde la u ltima desigualdad se sigue de la desigualdad (1.11). Por la desigualdad M G M A, a2 + b2 + c2 3 3 (abc)2 , luego (a2 + b2 + c2 )3 33 (abc)2 . Por lo tanto (a2 + b2 + c2 )4 32 (abc)4 . Soluci on 3.52 Por la desigualdad M G M A, (a + b)(a + c) = a(a + b + c) + bc 2 abc(a + b + c). Segunda Soluci on. Dena x = a + b, y = a + b, z = b + c, tendr a que x, y , z son las longitudes de los lados del tri angulo XY Z , ya que a, b, c son positivos. Entonces, por la f ormula del area del tri angulo de la secci on 2.2 la desigualdad xy es equivalente a 2 (XY Z ) . Ahora, use que el area del tri angulo con lados . de longitud x, y , z es menor o igual a xy 2 Soluci on 3.53 Como xi 0, entonces xi 1 1. Luego, puede usar la desigualdad de Bernoulli para toda i, y obtener (1 + (xi 1))i 1 + i(xi 1).

4.3 Soluciones a los problemas del cap tulo 3 Sumando estas desigualdades para 1 i n, obtendr a el resultado.

205

Soluci on 3.54 Restando 2, se obtiene que las desigualdades son equivalentes a (a + b c)(a b + c)(a + b + c) 1. 0< abc La desigualdad de la izquierda es ahora obvia. La desigualdad de la derecha es el ejemplo 2.2.3. a4/3 +a Soluci on 3.55 Si muestra que a2a , ser a claro como b4/3 +c4/3 +8bc obtener el resultado. Esta u ltima desigualdad es equivalente a a4/3 + b
4/3 4/3

+c

4/3

a2/3 (a2 + 8bc).

Aplique la desigualdad M G M A en cada factor de a4/3 + b


4/3

+c

4/3

a4/3

= b

4/3

+c

4/3

a4/3 + a4/3 + b

4/3

+c

4/3

Otra forma de resolver el problema es la siguiente, considere la funci on f (x) = 3 1 (x) = ; esta funci o n es convexa para x > 0 ( f > 0 ). Para 0 < a, b, 5 x c < 1, con a + b + c = 1, se tiene que
a x

esto a x = a2 + 8bc, y = b2 + 8ca y z = factor para tener la condici on a + b + c = 1), obtendr a

4 x c b + ax+1by+cz . Aplicando y z c2 + 8ab (antes multiplique por un

a b c 1 + + . a2 + 8bc b2 + 8ca c2 + 8ab a3 + b3 + c3 + 24abc

Use tambi en que, (a + b + c)3 = a3 + b3 + c3 + 3(a2 b + a2 c + b2 a + b2 c + c2 a + c2 b) + 6abc a3 + b3 + c3 + 24abc. Soluci on 3.56 Utilizando la desigualdad de Cauchy-Schwarz xi con ai = 1, bi = 1+x2 +x 2 ++x2 , se tiene que
1 2 i

ai bi b2 i.

a2 i

b2 i,

x1 x2 xn + + + n 2 2 2 2 2 1 + x 1 1 + x1 + x2 1 + x1 + + x n Luego, es suciente demostrar que 1


1 . x2 1

2 b2 i < 1. Para i = 1, use que b1

x2 1 1+x2 1

206 Observe que para i 2, b2 i = xi 2 1 + x1 + + x 2 i


2

Soluciones a los Ejercicios y Problemas

= =
x2

x2 i 2 (1 + x2 + + x2 1 i) x2 i 2 )(1 + x2 + + x2 ) (1 + x2 + + x 1 1 i1 i 1 1 2 2 . (1 + x2 (1 + x2 1 + + xi1 ) 1 + + xi )

1 1 Para i = 1, use que b2 1 1+x2 = 1 1+x2 . 1 1 Sume las desigualdades, la suma de la derecha es telesc opica, por lo que, n

b2 i =
i=1

xi 2 1 + x1 + + x 2 i

1+

x2 1

1 < 1. + + x2 n

Soluci on 3.57 Como s olo existen dos valores posibles para , , , los tres deben ser iguales, o dos iguales y uno diferente de los otros dos. Por lo tanto, hay dos casos a considerar. (1) = = . En este caso, se tiene que a + b + c = 0, y por lo tanto a3 + b3 + c3 abc
2

= =

a3 + b3 (a + b)3 ab(a + b)

(a + b)2 a2 + ab b2 ab

3ab ab

= 9.

(2) Sin p erdida de generalidad, puede suponer que = , = , entonces c=a+b y a3 + b3 + c3 abc = = a3 + b3 + (a + b)3 (a + b)2 + a2 ab + b2 = ab(a + b) ab 2 2 a b 2a + 2b + ab =2 + + 1. ab b a

Si a y b tienen el mismo signo, vea que esta expresi on no es menor que 5, y su cuadrado es, por lo tanto, no menor a 25. Si los signos de a y b no son el mismo, 2 a a b b b 9. se tiene a b + a 2, por lo tanto 2 b + a + 1 3 y 2 b + a + 1 Entonces, el valor posible m as peque no es 9.
1 Soluci on 3.58 Use la desigualdad M G M A, para ver que b(a1 +b) + c(b+c) + 3 3 1 abc, Y = 3 (a + b)(b + c)(c + a). Use M G M A a(c+a) XY , donde X =

4.3 Soluciones a los problemas del cap tulo 3 nuevamente, para obtener que X 27 1 2 (a+b+c)2 .
a+b+c 3 b+c y Y 2 a+3 . Luego,

207
3 XY

Soluci on 3.59 La desigualdad es equivalente a a4 + b4 + c4 a2 bc + b2 ca + c2 ab, que se sigue directamente del teorema de Muirhead, ya que [4, 0, 0] [2, 1, 1]. Segunda Soluci on. b3 c3 a3 + + bc ca ab a4 b4 c4 + + abc abc abc (a2 + b2 + c2 )2 3abc a + b + c 3 (a + b + c) (a + b + c)4 = 27abc 3 abc a+b+c (abc) = a + b + c. abc =

En las primeras dos desigualdades aplicamos la desigualdad (1.11), y en la u ltima desigualdad aplicamos la desigualdad M G M A.
2 x Soluci on 3.60 Considere a f (x) como f (x) = 1 x . Como f (x) = (1x)3 > 0, f (x) es convexa. Usando la desigualdad de Jensen se obtiene f (x) + f (y ) + y +z ). Pero f es creciente para x < 1, y la desigualdad M G M A f (z ) 3f ( x+3 y +z y +z 3 xyz , se tiene f ( x+3 ) f ( 3 xyz ). le ayudar a a probar que x+3

Soluci on 3.61 1 a + b+c 2 1 b + c+a 2 1 c + a+b 2 1

es equivalente a (2a + b + c)(2b + c + a)(2c + a + b) 8(b + c)(c + a)(a + b). Ahora, observe que (2a + b + c) = (a + b + a + c) 2 (a + b)(c + a). Soluci on 3.62 La desigualdad del problema es equivalente a la siguiente desigualdad (a + b c)(a + b + c) (b + c a)(b + c + a) (c + a b)(c + a + b) + + 9, c2 a2 b2
(b+c) (c+a) b) 12. Como (a + b)2 la cual a su vez es equivalente a (a+ c2 + a2 + b2 4ab, (b + c)2 4bc y (c + a)2 4ca, se tiene que
2 2 2

(a + b)2 (b + c)2 (c + a)2 4ab 4bc 4ca 3 (ab)(bc)(ca) + + 2 + 2 + 2 12 = 12. 2 2 2 c a b c a b c2 a2 b2

208

Soluciones a los Ejercicios y Problemas

Soluci on 3.63 Por la desigualdad M G M A, x2 + x + x 3x. Sumando desigualdades similares para y , z , se obtiene x2 + y 2 + z 2 + 2( x + y + z ) 3(x + y + z ) = (x + y + z )2 = x2 + y 2 + z 2 + 2(xy + yz + zx). 1 1 +1 Soluci on 3.64 Multiplicando por abc la igualdad 1 = a b + c , se tiene bc ca abc = ab c + ab c + a + b . Entonces, es suciente demostrar que c + ab ltima desigualdad es equivalente a c + ab c . Elevando al cuadrado la u ab 1 c + c + 2 ab o c + ab c + ab(1 a 1 b ) + 2 ab o a + b 2 ab. Soluci on 3.65 Como (1 a)(1 b)(1 c) = 1 (a + b + c) + ab + bc + ca abc y como a + b + c = 2, la desigualdad es equivalente a 1 . 27 Pero a < b + c = 2 a implica que a < 1 y, an alogamente, b < 1 y c < 1, entonces la desigualdad de la izquierda es verdadera. La otra desigualdad se sigue de la desigualdad M G M A. 0 (1 a)(1 b)(1 c) Soluci on 3.66 Es posible construir otro tri angulo AA1 M , de lados AA1 , A1 M , M A de longitud igual a las medianas ma , mb , mc .

C1

B1

A1

3 Adem as, (AA1 M ) = 4 (ABC ). Entonces la desigualdad que se tiene que demostrar es, 3 1 1 3 1 + . + ma mb mb mc mc ma 4 (AA1 M ) Ahora, la u ltima desigualdad ser a verdadera si la siguiente desigualdad se satisface para las longitudes de los lados de un tri angulo a, b, c y area S , es decir, 1 1 1 3 3 + + , ab bc ca 4S

4.3 Soluciones a los problemas del cap tulo 3 la cual es equivalente a

209

4 3S

9abc a+b+c

que a su vez es el ejemplo 2.4.6. Soluci on 3.67 Sustituya cd = desigualdad se convierte en


1 ab

y da =

1 bc ,

entonces el lado izquierdo de la

1 + ab 1 + bc 1 + bc 1 + ab + + + 1+a ab + abc 1+b bc + bcd = (1 + ab) 1 1 + 1 + a ab + abc


1 x

(4.8) .

+ (1 + bc)

1 1 + 1 + b bc + bcd

Ahora, use la desigualdad

1 y

4 x +y ,

para obtener

(Lado izquierdo) (1 + ab) = 4 = 4

4 4 + (1 + bc) 1 + a + ab + abc 1 + b + bc + bcd 1 + ab 1 + bc + 1 + a + ab + abc 1 + b + bc + bcd 1 + ab a + abc + = 4. 1 + a + ab + abc a + ab + abc + abcd

Soluci on 3.68 Por el teorema de Stewart se tiene que


2 la = bc 1

a b+c

1 bc ((b + c)2 a2 ) ((b + c)2 a2 ). 2 (b + c) 4

Por la desigualdad de Cauchy-Schwarz se sigue que


2 2 2 + lc ) (la + lb + lc )2 3(la + lb 3 ((a + b)2 + (b + c)2 + (c + a)2 a2 b2 c2 ) 4 3 = (a + b + c)2 . 4

Soluci on 3.69 Como

1 1a

1 b+c

la desigualdad es equivalente a

1 1 2 2 2 1 + + + + . b+c c+a a+b 2a + b + c 2b + a + c 2c + a + b

210 Ahora, use que 2


1 x

Soluciones a los Ejercicios y Problemas +


1 y

4 x +y ,

para ver que, 4 4 4 + + a + b + 2c b + c + 2a c + a + 2b

1 1 1 + + b+c c+a a+b

lo cual muestra la desigualdad. Soluci on 3.70 Podemos suponer que a b c. Entonces c < a + b y n n n 2 2 2 n = (a + b + c) > (2c) = 2cn n bn + cn . 2 2 2 Como a b, se tiene que b+ a 2
n

a = bn + nbn1 + otros t erminos positivos 2 n > bn + abn1 bn + an . 2

n n n An alogamente, ya que a c, se tiene (c + a 2 ) > c + a ; por lo tanto n 1 1 1 2 a a n n n n n n n n n +c+ < b+ + (a + b ) + (b + c ) + (c + a ) 2 2 2 n n 2 2 =1+ . = a+b+c+ 2 2

Segunda Soluci on. Recuerde que, a, b, c son las longitudes de los lados de un tri angulo si y s olo si existen n umeros positivos x, y , z con a = y + z , b = z + x, 1 c = x + y . Como a + b + c = 1, se tiene que x + y + z = 2 . Ahora, use la desigualdad de Minkowski
n
1 m

1 m

1 m

(xi + yi )
i=1

xm i
i=1

+
i=1

m yi

para obtener (an + bn ) n = ((y + z )n + (z + x)n ) n (xn + y n ) n + (2z n ) n < c +


1 1 1 1 1

2z.

An alogamente, (bn + cn ) n < a +

n 2y . Por lo tanto, n 1 1 1 2 n n n n n n n n n n (a + b ) + (b + c ) + (c + a ) < a + b + c + 2(x + y + z ) = 1 + . 2 2x y (cn + an ) n < b +


1

4.3 Soluciones a los problemas del cap tulo 3

211

Soluci on 3.71 Note primero que si restringe las sumas con i < j , entonces ellas suman la mitad. La suma de la izquierda est a elevada al cuadrado y la suma de la derecha no, entonces la desigualdad deseada con sumas restringidas a i < j , tiene (1/3), en lugar de (2/3), en el lado derecho. Considere las sumas de todos los |xi xj |, con i < j . El t ermino x1 aparece en (n 1) t erminos con signo negativo, x2 aparece en un t ermino con signo positivo y en (n 2) t erminos con signo negativo, etc. Entonces, se obtiene que (n 1)x1 (n 3)x2 (n 5)x3 + (n 1)xn = (2i 1 n)xi .

Ahora, puede usar la desigualdad Cauchy-Schwarz para mostrar que el cuadrado (2i 1 n)2 . de esta suma es menor que x2 i Analizando la suma del otro lado de la desigualdad deseada, observe inmediatamente que es n x2 xi )2 . Nos gustar a desaparecer el segundo t ermino, i ( pero esto es f acil, ya que si suma h a cada xi , las sumas en la desigualdad deseada no se afectan, ya que ellas usan s olo diferencias de xi . Luego, puede escoger h tal que xi sea cero. Luego, la demostraci on se termina si se puede n(n2 1) 2 , mostrar que (2i 1 n) = 3 (2i 1 n)2 = 4 = = = 2 n(n + 1)(2n + 1) 2n(n + 1)2 + n(n + 1)2 3 1 n(n + 1)(2(2n + 1) 6(n + 1) + 3(n + 1)) 3 1 n(n2 1). 3 i2 4(n + 1) i + n(n + 1)2

Con lo que se obtiene la desigualdad que se quer a. Segunda Soluci on. La desigualdad es del tipo de la desigualdad de CauchySchwarz, y como el problema pide que ser a igualdad cuando x1 , x2 , ... , xn sea una progresi on aritm etica, es decir, cuando xi xj = r (i j ), con r > 0, entonces considere la siguiente desigualdad, ya garantizada por la desigualdad de Cauchy-Schwarz, 2 (xi xj )2 (i j )2 .

i,j

|i j | |xj xj |

i,j

i,j

Aqu est a asegurado que la igualdad se da si y s olo si (xi xj ) = r (i j ), con r > 0.

212 Como
i,j

Soluciones a los Ejercicios y Problemas (i j )2 = (2n 2) 12 + (2n 4) 22 + + 2 (n 1)2 =


i,j n2 (n2 1) . 6

Bastar a mostrar que

|i j | |xi xj | =

n 2

i,j

|xi xj |. Para ver que esto

u ltimo sucede, compare el coeciente de xi en cada lado. En el lado izquierdo se tiene (i 1) + (i 2) + + (i (i 1)) ((i + 1) i) + ((i + 2) i) + + (n i)) = (i 1)i (n i)(n i + 1) n(2i n 1) = . 2 2 2 El coeciente de xi en el lado derecho es, n n n(2i n 1) 1+ 1 = ((i 1) (n i)) = . 2 2 2 =
i<j j>i

Como son iguales se termina la prueba.

Soluci on 3.72 Sean xn+1 = x1 y xn+2 = x2 . Dena ai = Es claro que xi xi+1


n

bi = xi + xi+1 + xi+2

i {1, ..., n}.

ai = 1,
i=1 i=1

bi = 3
i=1

xi = 3.

La desigualdad es equivalente a
n i=1

ai n2 . bi 3

Usando la desigualdad M G M A, puede deducir que 1 n


n i=1

bi

b1 b n

3 n

b1 b n

n 1 . 3 b1 b n

Por otra parte y usando nuevamente la desigualdad M G M A, se obtiene


n i=1

n a a an n n2 a1 ai 1 n n . n =n = n n bi b1 bn 3 b1 b n b1 b n

4.3 Soluciones a los problemas del cap tulo 3

213

1 2, con igualdad Soluci on 3.73 Para cualquier a n umero real positivo a + a si y s olo si a = 1. Como los n umeros ab, bc y ca son no-negativos, se tiene

P (x)P

1 x

1 1 +b +c x2 x 1 1 = a2 + b2 + c2 + ab x + + bc x + x x = (ax2 + bx + c) a

+ ca x2 +

1 x2

a2 + b2 + c2 + 2ab + 2bc + 2ca = (a + b + c)2 = P (1)2 . La igualdad se tiene si y s olo si x = 1 o ab = bc = ca = 0, que por la condici on a > 0, implica que b = c = 0. Consecuentemente, para cualquier n umero real positivo x, se tiene 1 P (x)P (P (1))2 , x con igualdad si y s olo si x = 1 o b = c = 0. Segunda Soluci on. Use la desigualdad de Cauchy-Schwarz para obtener P (x)P 1 x = (ax2 + bx + c) a = 1 1 +b +c 2 x x 2 2 2 a ( ax) + ( bx) + ( c) x a b ax + bx + c c x x
2

b x

2 + ( c)

= (a + b + c)2 = (P (1))2 .

Soluci on 3.74

a2 (b + c) + b2 (c + a) + c2 (a + b) 3 (a + b)(b + c)(c + a) 4

3 a2 b + a2 c + b2 c + b2 a + c2 a + c2 b 2 2 2 2 2 2 2abc + a b + a c + b c + b a + c a + c b 4 2 2 2 2 2 2 a b + a c + b c + b a + c a + c b 6abc [2, 1, 0] [1, 1, 1].

La u ltima desigualdad se sigue del teorema de Muirhead. Segunda Soluci on. Use la desigualdad (1.11) y la desigualdad de CauchySchwarz.

214

Soluciones a los Ejercicios y Problemas

Soluci on 3.75 Use la desigualdad M G M A en cada denominador, para obtener 1 1 1 1 1 1 + + + + . 1 + 2ab 1 + 2bc 1 + 2ca 1 + a2 + b2 1 + b2 + c2 1 + c2 + a2 Ahora, usando la desigualdad (1.11), se obtiene 1 1 1 (1 + 1 + 1)2 9 + + = = 1. 2 2 2 2 2 2 2 2 2 1+a +b 1+b +c 1+c +a 3 + 2(a + b + c ) 3+23 Soluci on 3.76 La desigualdad es equivalente a cada una de las siguientes desigualdades x4 + y 4 + z 4 + 3(x + y + z ) (x3 z + x3 y + y 3 x + y 3 z + z 3 y + z 3 x) x3 (x + y + z ) + y 3 (x + y + z ) + z 3 (x + y + z ) + 3(x + y + z ) 0 (x + y + z )(x3 + y 3 + z 3 3xyz ) 0. La identidad (1.9) muestra que la u ltima desigualdad es equivalente a 2 2 2 y + z ) ((x y ) + (y z ) + (z x)2 ) 0.
1 2 (x

Soluci on 3.77 Sean O y I el circuncentro y el incentro de un tri angulo acut angulo ABC , respectivamente. Los puntos O, M , X son colineales y, OCX y OM C son tri angulos rect angulos semejantes. De donde se tiene OM OC = . OX OC
OA Como OC = R = OA, se tiene OM = OX OA . Luego, OAM y OXA son semeOM AM jantes, entonces AX = R . Ahora es suciente mostrar que OM r . Compare los angulos OBM y IBM . Como ABC es acut angulo, O y I son puntos interiores de ABC . 1 Ahora, se tiene que OBM = 2 A = 2 (A + B + C ) A = 1 B 2 (B + C A) 2 = IBM , donde la desigualdad se sigue ya que C < A. An alogamente, se tiene que OCM ICM . Luego, el punto O es un punto interior de IBC , de donde OM r .

Soluci on 3.78 Dena a = x2 , b = y 2 , c = z 2 , la desigualdad es equivalente a x6 + y 6 + z 6 x4 yz + y 4 zx + z 4 xy. Esto se sigue del teorema de Muirhead, ya que [6, 0, 0] [4, 1, 1].

4.3 Soluciones a los problemas del cap tulo 3

215

Soluci on 3.79 Use la desigualdad de Cauchy-Schwarz para ver que xy + z = x y + z z x + z y + z = xy + z (x + y + z ) = xy + z . An alogamente, yz + x yz + x y zx + y zx + y. Por lo tanto, xy + z + yz + x + zx + y xy + yz + zx + x + y + z. Soluci on 3.80 Por el ejemplo 1.4.11, se tiene que a3 + b3 + c3 Ahora, a3 + b3 + c3 a+b+c 3 (a2 + b2 + c2 ) 3 abc(ab + bc + ca) ab + bc + ca, (a + b + c)(a2 + b2 + c2 ) . 3

donde se utilizaron la desigualdad M G M A y la desigualdad Cauchy-Schwarz. Soluci on 3.81 Use el ejemplo 1.4.11, obtendr a (a + b + c)(a2 + b2 + c2 ) 3 3 3 3(a + b + c ), pero si por hip otesis (a + b + c)2 3(a3 + b3 + c3 ) entonces a + b + c 1. Por otra parte, 4(ab + bc + ca) 1 a2 + b2 + c2 ab + bc + ca, por lo tanto 3(ab + bc + ca) 1. Como 1 3(ab + bc + ca) (a + b + c)2 1, se obtiene a + b + c = 1. Consecuentemente, a + b + c = 1 y 3(ab + bc + ca) = (a + b + c)2 , lo cual implica a = b = c = 1 3. Soluci on 3.82 Por la desigualdad de Cauchy-Schwarz se tiene (|a| + |b| + |c|)2 3(a2 + b2 + c2 ) = 9. Luego, |a| + |b| + |c| 3. De la desigualdad M G M A se tiene a2 + b2 + c2 3 3 (abc)2 por lo que |abc| 1, que implica abc 1. La desigualdad requerida es obtenida entonces por adici on. Soluci on 3.83 Note que OA1 OB OC BC (OBC ) 4R = . = AA1 (ABC ) 4R1 AB AC BC

216 Luego, se tiene que demostrar que

Soluciones a los Ejercicios y Problemas

OB OC BC + OA OB AB + OA OC AC AB AC BC. Considere las coordenadas complejas O(0), A(a), B (b), C (c) y obtenga |b| |c| |b c| + |a| |b| |a b| + |a| |c| |c a| |a b| |b c| |c a|. Es decir, |b2 c c2 b| + |a2 b b2 a| + |c2 a a2 c| |ab2 + bc2 + ca2 a2 b b2 c c2 a|, lo cual es obvio por la desigualdad del tri angulo. Soluci on 3.84 Sea S = {i1 , i1 + 1, ..., j1 , i2 , i2 + 1, ..., j2 , ..., ip , ..., jp } el orden de S , donde jk < ik+1 para k = 1, 2, ..., p 1. Dena Sp = a1 + a2 + + ap , S0 = 0. Entonces ai = Sjp Sip 1 + Sjp1 Sip1 1 + + Sj1 Si1 1 (ai + + aj )2 = (Si Sj )2 .

iS

y
1ij n 0ij n

Es suciente demostrar una desigualdad de la forma


p

(x1 x2 + + (1)

p+1

xp )

1i<j p

(xj xi ) +

x2 i,
i=1

(4.9)

ya que esto signica olvidar los mismos t erminos no-negativos en la expresi on de la derecha de la desigualdad dada. Entonces la desigualdad (4.9) se reduce a
p

4
1ij p j i par

xi xj (p 1)

x2 i.
i=1

Esto puede ser obtenido sumando las desigualdades de la forma 4xi xj 2(x2 i + y para i par, xi aparece x2 j ), i < j , j i =par (para i impar en la desigualdad, xi aparece
p 2 p 1 2

veces

1 veces).

Soluci on 3.85 Sean x = a + b + c, y = ab + bc + ca, z = abc. Entonces 2 2 a + b + c2 = x2 2y , a2 b2 + b2 c2 + c2 a2 = y 2 2xz , a2 b2 c2 = z 2 , luego, la

4.3 Soluciones a los problemas del cap tulo 3

217

desigualdad se convierte en z 2 + 2(y 2 2xz ) + 4(x2 2y ) + 8 9y , es decir, z 2 + 2y 2 4xz + 4x2 17y + 8 0. Ahora como a2 + b2 + c2 ab + bc + ca = y se obtiene x2 = a2 + b2 + c2 + 2y 3y . Tambi en, a2 b2 + b2 c2 + c2 a2 = (ab)2 + (bc)2 + (ca)2 = (a + b + c)abc = xz , luego, y 2 = a2 b2 + b2 c2 + c2 a2 + 2xz 3xz . De donde, z 2 + 2y 2 4xz + 4x2 17y + 8 = + z x 3
2

ab ac + bc ab + ac bc

35 2 (x 3y ) 0, 9

8 10 + (y 3)2 + (y 2 3xz ) 9 9

como era requerido. Segunda Soluci on. Desarrollando el lado izquiedo de la desigualdad, se obtiene la desigualdad equivalente (abc)2 + 2(a2 b2 + b2 c2 + c2 a2 ) + 4(a2 + b2 + c2 ) + 8 9(ab + bc + ca).
2 2 2 2 2 2 Como 3(a2 + b2 + c2 ) 3(ab + bc + ca) and 2(a b + b c + c a ) + 6 4(ab + bc + ca) (ya que por ejemplo, 2a2 b2 + 2 4 a2 b2 = 4ab), es suciente mostrar que (abc)2 + a2 + b2 + c2 + 2 2(ab + bc + ca).

La parte (i) del ejercicio 1.90 nos dice que es suciente probar que (abc)2 + 2 3 2 2 2 3 a b c , pero esto se sigue de la desigualdad M G M A.

Soluci on 3.86 Escriba 3 3 3


3

1 3 + 6(a + b + c) = 3 abc 3
3

1 + 6a2 bc + 6b2 ac + 6c2 ab = abc

3 = 3 3

1 + 3ab(ac + bc) + 3bc(ba + ca) + 3ca(ab + bc) , abc

y considere la condici on ab + bc + ca = 1, para obtener 3 3 3


3

1 + 3ab 3(ab)2 + 3bc 3(bc)2 + 3ca 3(ca)2 = abc


3

3 = 3 3

4 3((ab)2 + (bc)2 + (ca)2 ) . abc

218

Soluciones a los Ejercicios y Problemas

Es f acil ver que 3((ab)2 + (bc)2 + (ac)2 ) (ab + bc + ac)2 (use la desigualdad de Cauchy-Schwarz). Entonces es suciente probar que 3 3 3 que es equivalente a (abc)2 la desigualdad M G M A
1 27 .
3

1 3 , abc abc

Pero esta u ltima desigualdad es inmediata de


3

(abc)2 = (ab)(bc)(ca)

ab + bc + ca 3
1 . 3

1 . 27

La igualdad se da si y s olo si a = b = c =

Soluci on 3.87 Por simetr a, es suciente mostrar que t1 < t2 + t3 . Se tiene


n n

ti
i=1 i=1

1 =n+ ti

1i<j n

tj ti + tj ti

n + t1

1 1 + t2 t3

1 (t2 + t3 ) + t1

(i,j )=(1,2),(1,3)

tj ti + tj ti

Usando la desigualdad M G M A, se tiene 1 1 + t2 t3 tj 2 ti + 2, para todo i,j. , t2 + t3 2 t2 t3 , y tj ti t2 t3

Luego, si a = t1 / t2 t3 > 0 y utilizando la hip otesis se llega a


n n

n +1 >
i=1

ti
i=1

t2 t3 n2 n t1 2 1 n+2 +2 2 = 2a+ +n2 4. +2 ti t1 2 a t2 t3

2 De donde, 2a + a 5 < 0 y esto implica 1/2 < a = t1 / t2 t3 < 2, por lo que t1 < 2 t2 t3 . Una aplicaci on m as de la desigualdad M G M A, implica a. t1 < 2 t2 t3 t2 + t3 , como se quer Soluci on 3.88 Note que 1 + b c = a + b + c + b c = a + 2b 0. Entonces 3 a 1+bca 1 + 1 + (1 + b c) 3 =a+ ab ac . 3

4.3 Soluciones a los problemas del cap tulo 3 An alogamente, bc ba b3 1+ca b+ 3 ca cb 3 . c 1+ab c+ 3 Sume estas tres desigualdades, y obtendr a que a 3 1 + b c + b 3 1 + c a + c 3 1 + a b a + b + c = 1.

219

Soluci on 3.89 Si alguno de los n umeros es cero o si un n umero impar de n umeros son negativos, entonces x1 x2 x6 0 y la desigualdad es obvia. Por lo tanto, u nicamente podemos tener 2 o 4 n umeros negativos entre los n umeros involucrados en la desigualdad. Supongamos que ninguno de ellos es cero y que hay dos n umeros negativos (para el otro caso, cambie el signo de 2 + y 2 + + y 2 = 6, todos los n umeros). Si yi = |xi | entonces, es claro que y1 2 6 y1 + y2 = y3 + + y6 , and that x1 x2 x6 = y1 y2 y6 . De la desigualdad M G M A se obtiene y1 y2 y1 + y2 2
2

= A2 .

Adem as, la desigualdad M G M A garantiza que y3 y4 y5 y6 y3 + y4 + y5 + y6 4


4

y1 + y2 4

1 4 A . 24

1 6 Por lo tanto, y1 y2 y6 2 4A . Por otro lado, la desigualdad de Cauchy-Schwarz implica que, 2 2 2(y1 + y2 ) (y1 + y2 )2 = 4A2 2 2 2 2 4(y3 + y4 + y5 + y6 ) (y3 + y4 + y5 + y6 )2 = 4A2 .

2 + y 2 + + y 2 2A2 + A2 = 3A2 y entonces y y ...y Es decir, 6 = y1 1 2 6 6 2 3 2 1 1 6 . A = 4 4 2 2 2 b c Soluci on 3.90 Use la desigualdad de Cauchy-Schwarz con (1, 1, 1) y ( a b , c, a) para obtener 2 2 2

(1 + 1 + 1 )

a2 b2 c2 + + b2 c2 a2

a b c + + b c a

220

Soluciones a los Ejercicios y Problemas


a b

La desigualdad M G M A implica que c2 a2 b2 + + b2 c2 a2 An alogamente,


a c

b c

c a

33 .

abc bca

= 3, luego

a b c + + b c a

b a

c b

33

abc bca

= 3. Por lo tanto,

a2 b2 a b c c2 a b c + + + + + 3+ + + . b2 c2 a2 c a b b c a Sumando
a c

b a

c b

en ambos lados, se obtiene el resultado.

Soluci on 3.91 Note que (a2 a + 1) + (a + 1) a2 + 2 = 2 2 (a2 a + 1)(a + 1) = 1 + a3 .

Despu es de sustituir en la desigualdad se tiene que demostrar que b2 c2 1 a2 + 2 + 2 . 2 2 + 2)(b + 2) (b + 2)(c + 2) (c + 2)(a2 + 2) 3

(a2

Sea x = a2 , y = b2 , z = c2 , luego xyz = 64 y x y z 1 + + (x + 2)(y + 2) (y + 2)(z + 2) (z + 2)(x + 2) 3 si y s olo si 3[x(z + 2) + y (x + 2) + z (y + 2)] (x + 2)(y + 2)(z + 2). Ahora, 3(xy + yz + zx)+ 6(x + y + z ) xyz + 2(xy + yz + zx)+ 4(x + y + z )+ 8 si y s olo si xy + yz + zx + 2(x + y + z ) xyz + 8 = 72, pero por la desigualdad M G M A se tiene que x + y + z 12 y xy + yz + zx 48, lo cual concluye la demostraci on. Soluci on 3.92 Observe que x2 (y 2 + z 2 )(x3 1)2 x5 x2 x5 x2 = 0. x5 + y 2 + z 2 x3 (x2 + y 2 + z 2 ) x3 (x5 + y 2 + z 2 )(x2 + y 2 + z 2 )

4.3 Soluciones a los problemas del cap tulo 3 Luego x5 x2 x5 + y 2 + z 2 = x5 x2 x3 (x2 + y 2 + z 2 ) 1 1 x2 2 2 2 x +y +z x 1 (x2 yz ) 0. x2 + y 2 + z 2
1 x

221

La segunda desigualdad se sigue del hecho de que xyz 1, es decir, La u ltima desigualdad se sigue de la ecuaci on (1.8). Segunda Soluci on. Primero, observe que

yz .

x5 + y 2 + z 2 (x2 + y 2 + z 2 ) x2 + y 2 + z 2 x5 x2 = = 1 . x5 + y 2 + z 2 x5 + y 2 + z 2 x5 + y 2 + z 2 Ahora tiene que probar que 1 1 1 3 + + 2 . x5 + y 2 + z 2 x5 + z 2 + x2 x5 + x2 + y 2 x + y2 + z2 Usando la desigualdad de Cauchy-Schwarz se obtiene (x2 + y 2 + z 2 )2 (x2 x3 + y 2 + z 2 )(x2 y como xyz 1 entonces x2 por lo tanto, 1 5 x + y2 + z2 yz + y 2 + z 2 (x2 + y 2 + z 2 )2
y 2 +z 2 + y2 + z2 2 (x2 + y 2 + z 2 )2 1 x3

1 + y2 + z2 ) x3

1 x

yz , por lo que

(x2 + y 2 + z 2 )2 (x5 + y 2 + z 2 )(yz + y 2 + z 2 ) 3 . + y2 + z2

x2

Soluci on 3.93 Inicie observando que (1 + abc) = = 1 1 1 + + a(b + 1) b(c + 1) c(a + 1) +3

1 + abc + ab + a 1 + abc + bc + b 1 + abc + ca + c + + a(b + 1) b(c + 1) c(a + 1) b(c + 1) 1+b c(a + 1) 1+c a(b + 1) 1+a + + + + + 6. a(b + 1) (b + 1) b(c + 1) (c + 1) c(a + 1) (a + 1)

222

Soluciones a los Ejercicios y Problemas

La u ltima desigualdad se sigue despu es de aplicar la desigualdad M G M A para seis n umeros. Soluci on 3.94 Sea R el circunradio del tri angulo ABC. Como BOC = 2A, COA = 2B y AOB = 2C se tiene que (ABC ) = (BOC ) + (COA) + (AOB ) R2 (sen 2A + sen 2B + sen 2C ) 2 2A + 2B + 2C R2 3 sen 2 3 2 2 R 3 3R2 3 sen . = = 2 3 4 = La desigualdad se debe a que la funci on sen x es c oncava en [0, ]. Por otro lado, como BOC es is osceles, la mediatriz OA de BC es tambi en bisectriz del angulo BOC por lo que BOA = COA = A; an alogamente, COB = AOB = B y AOC = BOC = C. R En el tri angulo B OC la altura sobre el lado B C es R 2 y B C = 2 (tan B + 2 tan C ), por lo que el area del tri angulo B OC es (B OC ) = R 8 (tan B +tan C ). 2 R R2 An alogamente (C OA ) = 8 (tan C + tan A) y (A OB ) = 8 (tan A + tan B ). Luego, (A B C ) = (B OC ) + (C OA ) + (A OB ) = R2 (tan A + tan B + tan C ) 4 R2 3 tan 4

A+B+C 3 R2 3 3R 2 = = 3 tan . 4 3 4 La desigualdad ocurre por ser convexa la funci on tan x en [0, 2 ]. De donde se concluye que, 3 3R 2 (A B C ) (ABC ). 4 alogaSoluci on 3.95 Note primero que a2 + bc 2a2 bc = 2 ab ca. An 2 2 mente, b + ca 2 bc ab, c + ab 2 ca bc; luego se puede asegurar

4.3 Soluciones a los problemas del cap tulo 3 que 1 1 1 1 + + a2 + bc b2 + ca c2 + ab 2 1 1 1 + + ab ca bc ab ca bc

223

Usando la desigualdad de Cauchy-Schwarz de la siguiente manera 1 1 1 + + ab ca bc ab ca bc el resultado se concluye. Soluci on 3.96 Por la desigualdad de Cauchy-Schwarz se obtiene 2 2 n ai ai ai aj ai = (n 1)2 A2 . = (n 1) aj
i=j i=j i=j i=1 2

1 1 1 + + ab bc ca

1 1 1 + + ca ab bc

Por otro lado,

ai aj =
i=j i=1

ai

a2 i
i=1

= A2 A.

Soluci on 3.97 Sin p erdida de generalidad, suponga que a1 an . Sea dk = ak+1 ak para k = 1, ..., n. Entonces d = d1 + + dn1 . Para i < j , se tiene |ai aj | = aj ai = di + + dj 1 . Entonces, s=
i<j

|ai aj | = =

n j 1 j =2 i=1 n j =2 n1 k =1

(di + + dj 1 )

(d1 + 2d2 + + (j 1)dj 1 )

= (n 1)d1 + (n 2)2d2 + + 1 (n 1)dn1 = k(n k)dk .

Como k(n k) (n 1), (ya que (k 1)(n k 1) 0) y 4k(n k) n2 2 (por la desigualdad M G M A), se obtiene que (n 1)d s n4 d . Para ver cu ando la desigualdad del lado izquierdo es igualdad, observe que k(n k) = (n 1) n(k 1) = k2 1 k = 1 o k = n 1, entonces (n 1)d = s s olo si d2 = = dn2 = 0, esto es, a1 a2 = = an1 an .

224

Soluciones a los Ejercicios y Problemas

Para ver cu ando la segunda desigualdad es igualdad, observe que 4k(n k) = n2 k = n k. Si n es impar, la igualdad 4k(n k) = n2 se da u nicamente cuando dk = 0, para toda k, por lo tanto, a1 = = an = 0. Si n es par, es decir, n = 2k, la u nica dk puede ser distinta de cero y entonces a1 = = ak ak+1 = = a2k . Soluci on 3.98 Considere el polinomio P (t) = tb(t2 b2 ) + bc(b2 c2 ) + 2 ct(c t2 ). Este satisface las igualdades P (b) = P (c) = P (b c) = 0, luego P (t) = (b c)(t b)(t c)(t + b + c), ya que el coeciente de t3 es (b c). Por lo que |ab(a2 b2 ) + bc(b2 c2 ) + ca(c2 a2 )| = |P (a)|

= |(b c)(a b)(a c)(a + b + c)|.

El problema es, ahora, encontrar el menor n umero M que cumpla para todos los n umeros a, b, c, la desigualdad |(a c)(a b)(b c)(a + b + c)| M (a2 + b2 + c2 )2 . Pero si (a, b, c) cumple la desigualdad, tambi en, (a, b, c) la cumple para cualquier n umero real . Por lo que se puede suponer, sin perder generalidad, que a2 + b2 + c2 = 1. As el problema se reduce a encontrar el valor m aximo de P = |(a b)(a c)(b c)(a + b + c)| para n umeros reales a, b, c que satisfacen a2 + b2 + c2 = 1. Note que [3(a2 + b2 + c2 )]2 = [2(a b)2 + 2(a c)(b c) + (a + b + c)2 ]2 8|(a c)(b c)|[2(a b)2 + (a + b + c)2 ] 16 2|(a c)(b c)(a b)(a + b + c)| = 16 2P.

Las dos desigualdades se obtienen de aplicar la desigualad M G M A. , y el valor m , ya que la igualdad Lo que garantiza que P 169 aximo es 169 2 2 ocurre con a =
3 3+ 6, 6 2

b=

6 6 2

yc=

6 3 3 . 6 2

1 Soluci on 3.99 Para a = 2, b = c = 2 y n 3 la desigualdad no es v alida. 3 a+b+c Si n = 1, la desigualdad se reduce a abc 1, que se sigue de abc 3 = 1. Para el caso n = 2, sea x = ab + bc + ca; entonces como a2 + b2 + c2 = (a + b + c)2 2(ab + bc + ca) = 9 2x y x2 = (ab + bc + ca)2 3(a2 bc + ab2 c + abc2 ) =

4.3 Soluciones a los problemas del cap tulo 3

225

3abc(a + b + c) = 9abc, la desigualdad es equivalente a abc(9 2x) 3. Pero, bastar a demostrar que x2 (9 2x) 27. Esta u ltima desigualdad es a su vez 2 equivalente a (2x + 3)(x 3) 0. Soluci on 3.100 Primero use la desigualdad M G M A para ver que ca + c+ a 3 2 2 3 c a . Por lo que (a + 1)(b + 1)2 (a + 1)(b + 1)2 (a + 1)(b + 1)2 (b + 1)2 = = . 3 ca + c + a + 1 (c + 1)(a + 1) (c + 1) 3 c2 a2 + 1 An alogamente para los otros dos sumandos; y entonces se tiene (a + 1)(b + 1)2 (b + 1)(c + 1)2 (c + 1)(a + 1)2 + + 3 3 3 3 c2 a2 + 1 3 a2 b2 + 1 3 b2 c2 + 1 (c + 1)2 (a + 1)2 (b + 1)2 + + . (c + 1) (a + 1) (b + 1) Ahora aplique la desigualdad (1.11). Soluci on 3.101 Use la transformaci on de Ravi a = x + y , b = y + z , c = z + x, y +z 3 y xyz ( x+3 ) =1 as, obtendr a que x + y + z = 3 2 8 . Adem a2 + b2 + c2 + (a2 + b2 + c2 )(a + b + c) + 4abc 4abc = 3 3 2 2 2 2((y + z ) + (z + x) + (x + y ) )(x + y + z ) + 4(y + z )(z + x)(x + y ) = 3 4 3 = ((x + y + z ) xyz ) 3 13 3 3 1 4 = . 3 2 8 3
13 3 .

Por lo tanto el valor m nimo es

Soluci on 3.102 Aplique la transformaci on de Ravi, a = y + z , b = z + x, c = x + y , de tal forma que la desigualdad se puede reescribir como (2z )4 (2x)4 (2y )4 + + (z + x)(2x) (x + y )(2y ) (y + z )(2z ) (y + z )(z + x) + (z + x)(x + y ) + (x + y )(y + z ).

226

Soluciones a los Ejercicios y Problemas

Aplique la desigualdad (1.11) y la desigualdad del ejercicio 1.27, para ver que (2z )4 (2x)4 (2y )4 + + (z + x)(2x) (x + y )(2y ) (y + z )(2z ) 8(x2 + y 2 + z 2 )2 x2 + y 2 + z 2 + xy + yz + zx 8(x2 + y 2 + z 2 )2 . 2(x2 + y 2 + z 2 )

Por otro lado (y + z )(z + x) + (z + x)(x + y ) + (x + y )(y + z ) = 3(xy + yz + zx) + (x2 + y 2 + z 2 ); luego bastar a demostrar que 4(x2 + y 2 + z 2 ) 3(xy + 2 2 2 yz + zx) + (x + y + z ), que se reduce a ver que x2 + y 2 + z 2 xy + yz + zx.
a+b b+c c +a Soluci on 3.103 La sustituci on x = a b , y = bc , z = ca , tiene la propiedad de que xy + yz + zx = 1. Por la desigualdad de Cauchy-Schwarz, (x + y + z )2 3(xy + yz + zx) = 3, por lo tanto |x + y + z | 3 > 1.

Soluci on 3.104 Bastar a considerar el caso en que x y z. Entonces x = y a, z = y + b con a, b 0. Por un lado se tiene que, xz = 1 xy yz = 1 (y a)y y (y + b) = 1 2y 2 + ay by y por otro xz = (y a)(y + b) = y 2 ay + by ab. Al sumar ambas identidades se tiene que 2xz = 1 y 2 ab, luego 2xz 1 = y 2 ab 0. 1 Si 2xz = 1 entonces y = 0 y xz = 1 una contradicci on, luego xz < 2 . 1 1 1 yz= 2 (n n ), cumplen x y z y xy + yz + zx = 1. Los n umeros x = y = n 1 1 1 Sin embargo, xz = 21 n (n n ) = 2 2n2 puede estar tan cerca como se desee 1 de 1 2 , por lo que el valor 2 no puede mejorarse. Soluci on 3.105 Suponga que a = [x] y que r = {x}. Entonces, la desigualdad es equivalente a a a + 2r a a + 2r Que se reduce a 2 Pero como
r a

2a + r r r 2a + r

9 > . 2

r a + a r

a r + a + 2r 2a + r

5 > . 2

a r

2, bastar a ver que, a r 3 + < . a + 2r 2a + r 2

Pero a +2r a + r y 2a + r a + r ; m as a un, en las dos desigualdades anteriores las igualdades no se pueden dar simult aneamente (ya que eso implicar a que

4.3 Soluciones a los problemas del cap tulo 3 a = r = 0), por lo que a r a r 3 + < + =1< . a + 2r 2a + r a+r a+r 2 Soluci on 3.106 Aplique la desigualdad (1.11) para ver que a+b+c 32 1 1 1 + + , a b c a+b+c

227

b+c entonces a+3 a+3 a demostrar que a + b + c b+c . Entonces, bastar Como (x + y + z )2 3(xy + yz + zx), sucede que

3 abc .

(a + b + c)2

1 1 1 + + a b c

1 1 1 + + ab bc ca

3 (a + b + c), abc

y de aqu , es inmediato concluir. Soluci on 3.107 Utilizando la desigualdad de Cauchy-Schwarz se tiene que (a + b + 1)(a + b + c2 ) (a + b + c)2 . Entonces a2 + b + c a + b2 + c a + b + c2 + + (a + b + c)2 (a + b + c)2 (a + b + c)2 1 1 1 + + 1. a+b+1 b+c+1 c+a+1 Por lo tanto, 2(a + b + c) + (a2 + b2 + c2 ) (a + b + c)2 = a2 + b2 + c2 + 2(ab + bc + ca) de aqu el resultado. Soluci on 3.108 Para un punto P dentro de ABC , considere el punto Q sobre la mediatriz de BC tal que AQ = AP. Sea S la intersecci on de BP con la tangente a la circunferencia en Q. Entonces, SP + P C SC , por lo que BP + P C = BS + SP + P C BS + SC. Por otro lado, BS + SC BQ + QC, luego BP + P C es m nimo si P = Q. Sea T el punto medio del segmento M N. Como el tri angulo AM Q es is osceles y M T es una de sus alturas, entonces M T = ZQ donde Z es el pie de la altura de Q sobre AB. Luego M N + BQ + QC = 2(M T + QC ) = 2(ZQ + QC ) es

228

Soluciones a los Ejercicios y Problemas

m nimo cuando Z , Q, C sean colineales y esto signica que CZ es la altura. Por simetr a, BQ tambi en deber a ser altura y entonces P es el ortocentro. Soluci on 3.109 Sea H el ortocentro del tri angulo M N P y sean A , B , C las proyecciones de H sobre BC , CA, AB , respectivamente. Como el tri angulo M N P es acut angulo H est a en el interior y tambi en en el interior del tri angulo ABC , y entonces x HA + HB + HC HM + HN + HP 2X. La segunda desigualdad es clara, las otras dos son los siguientes dos lemas. Lema 1. Si H es un punto interior o sobre los lados de un tri angulo ABC , y si A , B , C son sus proyecciones sobre BC , CA, AB , respectivamente, entonces x HA + HB + HC , donde x es la longitud de la altura menor de ABC. Demostraci on. HA HB HC (BHC ) (CHA) (AHB ) HA + HB + HC + + = + + = 1. x ha hb hc (ABC ) (ABC ) (ABC ) Lema 2. Si M N P es un tri angulo acut angulo y H es su ortocentro, entonces HM + HN + HP 2X , donde X es la altura m as grande del tri angulo M N P. Demostraci on. Suponga que M N P, entonces N P P M M N y entonces pasa que X es igual a la altura M M . Debe ver que HM + HN + HP 2M M = 2(HM + HM ), o equivalentemente que HN + HP HM + 2HM . Sea H el punto sim etrico de H con respecto a N P ; como M N H P es un cuadril atero c clico, el teorema de Ptolomeo dice que H M N P = H N M P + H P M N H N N P + H P N P y entonces se obtiene que H N + H P H M = HM + 2HM . Soluci on 3.110 Suponga, sin p erdida de generalidad, que x y z. Por lo que x + y z + x y + z , xy zx yz , 2z 2 (x + y ) 2y 2 (z + x) 2x2 (y + 21 21 . Por la desigualdad del reacomodo, z ), 21
2z (x+y ) 2y (z +x) 2x (y +z )

aplicada dos veces, se tiene que

2yz 2x2 (y + z )

xy + zx 2x2 (y + z )

4.3 Soluciones a los problemas del cap tulo 3 2x2 2x2 (y + z ) 2x2 + 2yz 2x2 (y + z )

229

Ahora, al sumar obtiene

de ambos lados de la desigualdad anterior, se

2x2 + xy + zx 2x2 (y + z ) 2x2 + x(y + z ) 2x2 (y + z ) 2 2x3 (y + z )

2x2 (y + z ) = 2( x + y + z ) = 2. Segunda Soluci on. Note primero que, x2 + yz 2x2 (y + z) = = x2 x(y + z ) + yz 2x2 (y + z) (x y )(x z ) + + x(y + z ) 2x2 (y + z )

y+z 2 2x2 (y + z ) y+ z (x y )(x z ) + . 2 2x2 (y + z )

An alogamente para los otros dos sumandos; por lo que x2 + yz 2x2 (y + z) Luego, bastar a ver que (x y )(x z ) 2x2 (y + z ) + (y z )(y x) 2y 2 (z + x) + (z x)(z y ) 2z 2 (x + y ) 0. (x y )(x z ) 2x2 (y + z) + x+ y+ z.

xy )(xz ) Sin perder generalidad, suponga que x y z. Luego ( 0, y 2 2x (y +z )

(y z )(y x) 2y 2 (z + x)

(z x)(z y ) 2z 2 (x + y)

(x z )(y z ) 2z 2 (x

+ y) (x y )(y z ) 2z 2 (x + y ) 1 2y 2 (z

(y z )(x y )

2y 2 (z + x) (y z )(x y ) 2y 2 (z + x)

= (y z )(x y )

1 2z 2 (x + y)

+ x)

0.

230

Soluciones a los Ejercicios y Problemas

La u ltima desigualdad es consecuencia de y 2 (z + x) = y 2 z + y 2 x yz 2 + z 2 x = 2 z (x + y ). Soluci on 3.111 Por la desigualdad (1.11) b2 c2 (a + b + c)2 a2 + + . 2 + b + c2 2 + c + a2 2 + a + b2 6 + a + b + c + a2 + b2 + c2 Entonces, tiene que probar 6 + a + b + c + a2 + b2 + c2 12, pero como a2 + b2 + c2 = 3, es suciente mostrar que a + b + c 3. Pero se tiene tambi en que (a + b + c)2 = a2 + b2 + c2 + 2(ab + bc + ca) 3(a2 + b2 + c2 ) = 9. La igualdad se da si y s olo si a = b = c = 1. Soluci on 3.112 Note primero que 1 2bc 2bc 2bc a bc = = = . a + bc 1 b c + bc (1 b)(1 c) (c + a)(a + b) 2ca 2ab 3 2bc + + . (c + a)(a + b) (a + b)(b + c) (b + c)(c + a) 2 Simplique esta u ltima desigualdad para obtener que 4 [bc(b + c) + ca(c + a) + ab(a + b)] 3(a + b)(b + c)(c + a), que a su vez es equivalente a la desigualdad ab + bc + ca 9abc.
1 + Pero esta u ltima es consecuencia de (a + b + c)( a 1 b

Por lo que la desigualdad es equivalente a

Soluci on 3.113 Note que (x y + y z + z x)2 = x2 y + y 2 z + z 2 x +2(xy yz + yz zx + zx xy ). La desigualdad M G M A garantiza que xy yz = xyz por lo que, (x y + y z + z x)2 x2 y + y 2 z + z 2 x + xy 2 + yz 2 + zx2 + 3xyz. xy 2 xyz + xy 2 , 2

+1 c ) 9.

4.3 Soluciones a los problemas del cap tulo 3

231

Como (x + y )(y + z )(z + x) = x2 y + y 2 z + z 2 x + xy 2 + yz 2 + zx2 + 2xyz , se tiene que (x y + y z + z x)2 (x + y )(y + z )(z + x) + xyz 1 (x + y )(y + z )(z + x) + (x + y )(y + z )(z + x) 8 9 = (x + y )(y + z )(z + x). 8 Por lo tanto K 2
9 8 y entonces 3 , este valor 2 2

igualdad con K =

3 . 2 2

Como para x = y = z , se da la

es el m nimo.

Segunda Soluci on. Aplique la desigualdad de Cauchy-Schwarz de la siguiente forma: x y +y z +z x = x xy + y yz + z zx (x + y + z )(xy + yz + zx).

Despu es de esto, use la desigualdad M G M A varias veces para tener (x + y + z ) (xy + yz + zx) 3 xyz 3 3
3

x2 y 2 z 2 = xyz

(x + y ) (y + z ) (z + x) . 2 2 2

Soluci on 3.114 El lado izquierdo de la desigualdad se escribir tambi en como a2 b2 cd+ab2 c2 d+abc2 d2 +a2 bcd2 +a2 bc2 d+ab2 cd2 = abcd(ab+bc+cd+ac+ad+bd).
c +d 4 La desigualdad M G M A garantiza que a2 b2 c2 d2 ( a +b + ) = 1 4 4 , 1 por lo que abcd 16 . Para ver que el factor (ab + bc + cd + ac + ad + bd) es menor a 3 2 se pueden seguir los dos caminos siguientes: Uno es aplicar la desigualdad de Cauchy-Schwarz para tener que
2 2 2 2

(ab + bc + cd + ac + ad + bd + ba + cb + dc + ca + da + db) (a2 + b2 + c2 + d2 + a2 + b2 + c2 + d2 + a2 + b2 + c2 + d2 ) = 3. El otro consiste en aplicar la desigualdad M G M A como sigue: (ab + bc + cd + ac + ad + bd) a2 + b2 b2 + c2 c2 + d2 + + 2 2 2 + a2 + c2 a2 + d2 b2 + d2 3 + + = . 2 2 2 2

232

Soluciones a los Ejercicios y Problemas

Soluci on 3.115 (a) Desarrollando los cuadrados y agrupando los t erminos se tiene (1 + x + y )2 + (1 + y + z )2 + (1 + z + x)2 = 3 + 4(x + y + z ) + 2(xy + yz + zx) + 2(x2 + y 2 + z 2 ). Ahora, aplique la desigualdad M G M A para obtener (x + y + z ) 3 3 xyz 3

(xy + yz + zx) 3 3 x2 y 2 z 2 3

(x2 + y 2 + z 2 ) 3 3 x2 y 2 z 2 3.

Luego, (1 + x + y )2 + (1 + y + z )2 + (1 + z + x)2 3 + 4 3 + 2 3 + 2 3 = 27. La igualdad se da cuando x = y = z = 1. (b) Nuevamente, desarrollando los cuadrados la desigualdad es equivalente a 3 + 4(x + y + z ) + 2(xy + yz + zx) + 2(x2 + y 2 + z 2 ) 3(x2 + y 2 + z 2 ) + 6(xy + yz + zx) y tambi en a 3 + 4u u2 + 2v , donde u = x + y + z 3 y v = xy + yz + zx 3. Pero u 3 implica que (u 2)2 1. Luego, (u 2)2 + 2v 1 + 6 = 7. La igualdad se da cuando u = 3 y v = 3, es decir, cuando x = y = z = 1. Soluci on 3.116 Note que 1 1 1 = = . 1+ + c) 1 + a(ab + ac) 1 + a(3 bc) 3a + 1 abc 3 +ca La desigualdad M G M A implica que 1 = ab+bc a2 b2 c2 , entonces 3 abc 1. Luego, 1 1 1 = . 2 1 + a (b + c) 3a + 1 abc 3a a2 (b =
1 1+b2 (c+a)

An alogamente, 1 1 + a2 (b

1 3b

1 1+c2 (a+b)

1 3c .

Por lo tanto, =

+ c)

1 1+ b2 (c + a)

1 1+ c2 (a + b)

1 1 1 + + 3a 3b 3c 1 bc + ca + ab = . 3abc abc

Soluci on 3.117 La desigualdad es equivalente a (a + b + c) 1 1 1 + + a+b b+c c+a k + (a + b + c)k = (a + b + c + 1)k.

4.3 Soluciones a los problemas del cap tulo 3 Por otro lado, usando la condici on a + b + c = ab + bc + ca, se tiene que 1 1 1 + + a+b b+c c+a = = = Luego, (a + b + c) (a + b + c + 1) 1 1 1 + + a+b b+c c+a = (a + b + c)2 1, (a + b + c)2 abc

233

a2 + b2 + c2 + 3(ab + bc + ca) (a + b)(b + c)(c + a) 2 a + b2 + c2 + 2(ab + bc + ca) + (ab + bc + ca) (a + b)(b + c)(c + a) (a + b + c)(a + b + c + 1) . (a + b + c)2 abc

y como la igualdad se da si y s olo si abc = 0, se tiene que k = 1 es el valor m aximo. Soluci on 3.118 Multiplicando ambos lados de la desigualdad por el factor (a + b + c), se obtiene la desigualdad equivalente 9(a + b + c)(a2 + b2 + c2 ) + 27abc 4(a + b + c)3 , la cual es a su vez equivalente a 5(a3 + b3 + c3 ) + 3abc 3(ab(a + b) + ac(a + c) + bc(b + c)). Por la desigualdad de Sch ur con n = 1, ver el ejercicio 2.83 , se sigue que a3 + b3 + c3 + 3abc ab(a + b) + bc(b + c) + ca(c + a), y la desigualdad de Muirhead nos dice que 2[3, 0, 0] 2[2, 1, 0], que es equivalente a 4(a3 + b3 + c3 ) 2(ab(a + b) + ac(a + c) + bc(b + c)). Sume las u ltimas desigualdades y obtendr a el resultado.
1 1 1 4 Soluci on 3.119 Lema. Si a, b > 0, entonces (a b)2 + a2 + b2 ab . Demostraci on. (a2 +b2 3ab)2 1 1 1 4 Basta notar que (a +a . 2 + b2 ab = b)2 a2 b2 (ab)2 Sin perder generalidad suponga que z = m n{x, y, z }; ahora aplique el lema con a = (x z ) y b = (y z ), para tener

1 1 4 1 . + + (x y )2 (y z )2 (z x)2 (x z )(y z )

234

Soluciones a los Ejercicios y Problemas

Ahora, ser a suciente mostrar que xy + yz + zx (x z )(y z ); pero esto es equivalente a 2z (y + x) z 2 , que es inmediato. Soluci on 3.120 Para el caso (i) hay varias maneras de resolverlo. a 1 f orma. Puede mostrase que y2 z2 (yz + zx + xy 3)2 x2 + + 1 = . (x 1)2 (y 1)2 (z 1)2 (x 1)2 (y 1)2 (z 1)2
y x z 2a f orma. Con la sustituci on a = x 1 , b = y 1 , c = z 1 , la desigualdad por demostrar es a2 + b2 + c2 1, y la condici on xyz = 1 es equivalente a abc = (a 1)(b 1)(c 1) o (ab + bc + ca) + 1 = a + b + c. Con la ayuda de las identidades anteriores se puede obtener que

a2 + b2 + c2 = (a + b + c)2 2(ab + bc + ca) = (a + b + c 1)2 + 1 luego a2 + b2 + c2 = (a + b + c 1)2 + 1.

= (a + b + c)2 2(a + b + c 1)

El caso (ii) se puede demostrar dependiendo de como se lleg o a establecer el caso (i). Por ejemplo, si se demostr o de la 2a forma, la igualdad se tiene cuando a2 + b2 + c2 = 1 y a + b + c = 1. (En la 1a f orma la igualdad se tiene cuando xyz = 1 y xy + yz + zx = 3). De las ecuaciones se puede cancelar una variable, por ejemplo c (y como c = 1 a b, si se demuestra que a y b n umeros racionales el c ser a tambi en un n umero racional), para llegar a a2 + b2 + ab a b = 0. Esta identidad se puede trabajar como una ecuaci on
1a (1a)(1+3a)

, que ser an n umeros racionales cuadr atica en b con ra ces b = 2 k , entonces si (1 a) y (1 + 3a) son cuadrados de n umeros racionales. Si a = m sucede que m k y m + 3k son cuadrados de n umeros enteros, por ejemplo, si m = (k 1)2 + k, es claro que m k = (k 1)2 y m + 3k = (k + 1)2 . Luego, k +k 2 1 los n umeros racionales a = m , b = mk2 y c = 1 a b, cuando k varia m en los n umeros enteros, son n umeros racionales donde se alcanza la igualdad. Hay unas excepciones k = 0, 1, ya que los valores a = 0 o 1 no son valores permitidos.

Notaci on

Utilizamos la siguiente notaci on est andar:

N R R+ aA AB |x| {x} [x] [a, b] (a, b) f : [a, b] R f (x) f (x) det A n i=1 ai n i=1 ai i=j ai m ax{a, b, . . . } m n{a, b, . . . } x

los n umeros enteros positivos (n umeros naturales) los n umeros reales los n umeros reales positivos si y s olo si implica el elemento a pertenece al conjunto A A es un subconjunto de B valor absoluto del n umero real x la parte fraccionaria de un n umero real x la parte entera de un n umero real x el conjunto de n umeros reales x tal que a x b el conjunto de n umeros reales x tal que a < x < b la funci on f denida en [a, b] con valores en R la derivada de la funci on f (x) la segunda derivada de la funci on f (x) el determinante de la matriz A la suma a1 + a2 + + an el producto a1 a2 an el producto para toda a1 , a2 , . . . , an excepto aj el m aximo valor entre a, b, . . . el m nimo valor entre a, b, . . . la ra z del n umero real positivo x

236

Notaci on

x exp x = ex f (a, b, . . . )
c clica

la n esima ra z del n umero real positivo x la funci on exponencial representa la suma de la funci on f evaluada en todas las permutaciones c clicas de las variables a, b, . . .

Utilizamos la siguiente notaci on en la secci on del teorema de M uirhead:


!

F (x1 , . . . , xn )

(b) (a) [b] [a]

la suma de n! t erminos obtenidos cuando se eval ua F en todas las posibles permutaciones de (x1 , . . . , xn ) (a) mayoriza a (b) 1 1 bn an 1 b2 1 a2 xb xa 1 x2 x n n ! 1 x2 xn n!
! !

Utilizamos la siguiente notaci on geom etrica: A, B, C a, b, c A , B , C ABC A (ABC ) (ABCD...) ma , mb , mc ha , hb , hc la , lb , lc s r R I, O, H, G Ia , Ib , Ic los v ertices del tri angulo ABC las longitudes de los lados del tri angulo ABC los puntos medios de los lados BC, CA y AB el angulo ABC el angulo en el v ertice A o la medida del angulo en el v ertice A el area del tri angulo ABC el area del pol gono ABCD... las longitudes de las medianas del tri angulo ABC las longitudes de las alturas del tri angulo ABC las longitudes de las bisectrices internas del tri angulo ABC el semiper metro del tri angulo ABC el inradio del tri angulo ABC, el radio del inc rculo el circunradio del tri angulo ABC, el radio del circunc rculo el incentro, el circuncentro, el ortocentro y el centroide del tri angulo ABC los centros del exc rculo del tri angulo ABC

Utilizamos la siguiente notaci on referente a los problemas: IMO APMO (pa s, a no) Olimpiada Internacional de Matem aticas (por sus siglas en ingl es) Olimpiada de la Cuenca del Pac co (por sus siglas en ingl es) problema que corresponde a la olimpiada de matem aticas celebrada en ese pa s, en ese a no, en alguna de las etapas

Bibliograf a

[1] Altshiller, N., College Geometry: An Introduction to Modern Geometry of the Triangle and the Circle. Barnes and Noble, 1962. [2] Andreescu, T., Feng, Z., Problems and Solutions from Around the World, Mathematical Olympiads 1999-2000. MAA, 2002. [3] Andreescu, T., Feng, Z., Lee, G., Problems and Solutions from Around the World, Mathematical Olympiads 2000-2001. MAA, 2003. [4] Andreescu, T., Enescu, B., Mathematical Olympiad Treasures. Birkh auser, 2004. [5] Barbeau, E.J., Shawyer, B.L.R., Inequalities. A Taste of Mathematics, vol. 4, 2000. [6] Bulajich, R., G omez Ortega, J.A., Geometr a, Cuadernos de Olimpiadas de Matem aticas, Instituto de Matem aticas, UNAM, 2002. [7] Bulajich, R., G omez Ortega, J.A., Geometr a. Ejercicios y Problemas, Cuadernos de Olimpiadas de Matem aticas, Instituto de Matem aticas, UNAM, 2002. [8] Courant, R., Robbins, H., Qu e son las matem aticas? Fondo de Cultura Econ omica, 2002. [9] Coxeter, H., Greitzer, S., Geometry Revisited. New Math. Library, MAA, 1967. [10] Dorrie, H., 100 Great Problems of Elementary Mathematics. Dover, 1965.

238

Bibliography

[11] Engel, A., Problem-Solving Strategies. Springer-Verlag, 1998. [12] Fomin, D., Genkin, S., Itenberg, I., Mathematical Circles. Mathematical World, Vol. 7. American Mathematical Society, 1996. [13] Hardy, G.H., Littlewood, J.E., P` olya, G., Inequalities. Cambridge at the University Press, 1967. [14] Honsberger, R., Episodes in Nineteenth and Twentieth Century Euclidean geometry. New Math. Library, MAA, 1995. [15] Kazarino, N., Geometric Inequalities. New Math. Library, MAA. 1961. [16] Larson, L., Problem-Solving Through Problems. Springer-Verlag, 1990. [17] Mitrinovic, D., Elementary Inequalities. Noordho Ltd-Groningen, 1964. [18] Niven, I., Maxima and Minima Without Calculus. Expositions, MAA, 1981. The Dolciani Math.

[19] Shariguin, I., Problemas de geometr a. Editorial Mir, 1989. [20] Soulami, T., Les Olympiades de Math ematiques. Ellipses, 1999. [21] Spivak, M., Calculus. Editorial Benjamin, 1967.

Indice
Concavidad Funci on interpretaci on geom etrica, 30 Convexidad Funci on interpretaci on geom etrica, 30 Desigualdad u til, 40 Cauchy-Schwarz, 18 a la Engel, 41 de Bernoulli, 36 de Cauchy-Schwarz, 41 forma de Engel, 41 de Euler, 78 de H older, 32 de Jensen, 25 de Leibniz, 81 de Minkowski, 32 de Nesbitt, 19, 43, 70, 75 de Ptolomeo, 62 de Schur, 36 de Tchebyshev, 21 del reacomodo, 15 del tri angulo, 3 forma general, 3 Entre medias potenciales, 37 H older generalizada, 37 media arm onica- media geom etrica, 9 media cuadr atica-media aritm etica, 23, 42 Media geom etrica-media aritm etica con pesos, 31 media geom etrica-media aritm etica, 10, 55 Popoviciu, 38 Young, 31 Discrepancia, 54 Distancia entre dos puntos, 3 Funci on c oncava, 27 convexa, 24 cuadr atica, 5 Lema de Viviani, 102, 105 Mayor que, 2 Media aritm etica, 8, 10, 23, 36 arm onica, 9, 23 cuadr atica, 23 geom etrica, 8, 10, 23, 36 Menor o igual que, 2

240 Menor que, 2 Problema de Pompeiu, 62 Fagnano, 102, 108 Fermat-Steiner, 102 Her on, 106 con c rculo, 107 Punto de Fermat, 103, 105 Recta real, 1 Soluci on de Fej er Problema de Fagnano, 111 de Hofmann-Gallai Problema de Fermat-Steiner, 105 de Schwarz Problema de Fagnano, 110 de Shwarz Problema de Fagnano, 109 de Steiner Problema de Fermat-Steiner, 106 Problema Fermat-Steiner, 108 de Torricelli Problema de Fermat-Steiner, 102 Teorema de Erd os-Mordell, 9497, 101 de Euler, 77 de Leibniz, 80 de Muirhead, 51, 52 de Pappus, 93 Transformaci on de Ravi, 64, 85 Tri angulo ortico, 109, 112 pedal, 114 Valor absoluto, 3

Indice

Potrebbero piacerti anche